SUMA204
SUMA204
SUMA204
UNDERGRADUATE COURSE
B.Sc., MATHEMATICS
SECOND YEAR
FOURTH SEMESTER
TRANSFORM TECHNIQUES
WELCOME
Warm Greetings.
I invite you to join the CBCS in Semester System to gain rich knowledge leisurely at
your will and wish. Choose the right courses at right times so as to erect your flag of
success. We always encourage and enlighten to excel and empower. We are the cross
bearers to make you a torch bearer to have a bright future.
DIRECTOR
(i)
B.Sc., MATHEMATICS CORE PAPER - VII
SECOND YEAR - FOURTH SEMESTER TRANSFORM TECHNIQUES
COURSE WRITER
Dr. R. Sakthivel
Assistant Professor
Department of Mathematics
Pachaiyappa’s College
Chennnai - 600 030.
Prof. V. THANGARAJ
Former Director and Head
Ramanujan Institute for
Advanced Study in Mathematics
University of Madras,
Chennai - 600 005.
(ii)
B.Sc –Mathematics - Second Year- Fourth Semester
Core Paper 7 – TRANSFORM TECHNIQUES
Syllabus
UNIT II: Introduction, Properties of inverse Laplace transform, Problems (usual types);
Convolution Theorem - Inverse Laplace Transform using Convolution theorem.
Chapter 3: Section 3.1 to 3.11
UNIT III: Introduction, Expansions of periodic function of period 2π; expansion of even and
odd functions; half range cosine and sine series – Fourier series of change of interval.
Chapter 1: Section 1.1 to 1.11
UNIT IV: Introduction of Fourier transform - Properties of Fourier Transforms - Inverse Fourier
transform – Problems, Fourier sine and cosine transforms and their inverse Fourier transform –
Problems, Convolution theorem, Parseval’s identity and problems using Parseval’s identity.
Chapter 4: Section 4.1 to 4.12
UNIT V: Applications of Laplace transform to solution of first and second order linear differential
equations (constant coefficients) and simultaneous linear ordinary differential equations –
Application of Laplace transform to partial differential equations. Application of Laplace
Transform and Fourier transform to Initial and Boundary Value Problems.
Chapter 5: Section 5.1, 5.3, 5.7 to 5.11
v
Recommended Text Book:
Content and treatment as in:
S. Sreenath, S. Ranganatham, M.V.S.S.N. Prasad and V. Ramesh Babu, Fourier Series and Integral
Transforms, S.Chand Publishing, New Delhi, 2014.
2. P.Kandasamy and others, Engineering Mathematics,(Volume 3), S.Chand and Co., New
Delhi,2010.
3. Stanley I. Grossman and William R. Derrick, Advanced Engineering Mathematics, Harper and
Row, New York, 1988.
vi
Lessons
18 CONVOLUTION 413
18.1 Introduction . . . . . . . . . . . . . . . . . . . . . . . . . . . . . . . . . . . . . 413
18.2 Parseval’s Identity for Fourier Transforms . . . . . . . . . . . . . . . . . . . . . 414
18.3 Worked out Problems . . . . . . . . . . . . . . . . . . . . . . . . . . . . . . . . 416
18.4 Exercises . . . . . . . . . . . . . . . . . . . . . . . . . . . . . . . . . . . . . . 421
18.5 Worked out Problems Related to Finite Fourier Transform . . . . . . . . . . . . . 421
Transform Techniques B.Sc.(Mathematics)-IDE(UNOM)-II Year - IV Sem
xii LESSONS
Lesson 1
• define Laplace transform and identify the condition for the existence of Laplace
transform
1.1 Introduction
T
here are many transform techniques which are used in the analysis and design of
engineering systems. Some of the transform techniques were introduced in the beginning
by great individuals which were vigorously defined and developed by mathematician in later days.
The technique of Laplace transform is one such. In this chapter and subsequent chapters to follow,
we introduce the idea of the Laplace transform and develop some useful results. Subsequently, we
display how the Laplace transform techniques is used in solving a class of problems in differential
equations.
1.2 Definition
Let F(t) be a given function and defined for all positive values of t. Then the Laplace transform
of F(t) is denoted by L [ f (t)] or f (p) and is defined by
Z ∞
L [F(t)] = f (p) = e−pt F(t)dt (1.2.1)
0
provided the integral exists. Here the parameter p is a real or complex number.
The operation of multiplying F(t) by e−pt and integrating from 0 to ∞ is called Laplace
transformation.
The relation (16.2.1) can also be written as
F(t) = L −1 [ f (p)].
In such a case, the function F(t) is said to be Inverse Laplace Transform of f (p). The symbol
L which transform F(t) into f (p) can be called the Laplace transform operator. The symbol
L −1 which transform f (p) into F(t) can be called the Inverse Laplace transform operator.
While finding the Laplace transform of elementary functions, it can be noticed that the integral
exists under certain conditions, such as p > 0 or p > a etc. In general, the function F(t) must
satisfy the following conditions for the existence of the Laplace transform.
(i) The function F(t) must be piece - wise continuous or sectionally continuous in any limited
interval 0 < a ≤ t ≤ b.
(ii) The function F(t) is of exponential order.
1.4 Definitions
Definition 1.4.1 (Piece - wise continuous function). A function F(t) is said to be piece - wise
(or sectionally) continuous over the closed interval [a, b] it it is defined on that interval and is
such that the interval can be divided into a finite number of subintervals, in each of which F(t) is
continuous and has both right and left hand limits at every end point of the subinterval.
i.e.,for a given number T, there exists a real number M > 0 such that
or
|F(t)| < Meat , ∀ t ≥ T.
For example, F(t) = t2 , sinat, eat etc., are all of exponential order and also continuous. But
F(t) = et is not of exponential order and as such its Laplace transform does not exists.
2
Theorem 1.4.1 (Existence Theorem For Laplace Transform). If F(t) is a function which is
piece - wise continuous on every finite interval in the range t ≥ 0 and satisfies |F(t)| ≤ Meat for
all t ≥ 0 and for some constants a and M, then the Laplace transform of F(t) exists for all
p > a.
Z t0 Z ∞
L [F(t)] = e−pt
F(t)dt + e−pt F(t)dt (1.4.1)
0 t0
R t0
The integral 0
e−pt F(t)dt exists, since F(t) is piece - wise continuous on every finite interval
0 ≤ t ≤ t0 .
Now,
Z ∞ Z ∞
−pt
e F(t)dt ≤
|e−pt F(t)dt|
t0
Zt0∞
< e−pt Meat , ( since |F(t) ≤ Meat |)
Zt0∞
= Me−(p−a)t dt
t0
h e−(p−a)t i∞
= M
−(p − a) t0
e−(p−a)t0
= M , (p > a)
p−a
Z ∞ e−(p−a)t0
∴ e−pt F(t)dt ≤ M , (p > a).
t0 p−a
−(p−a)t0
But M e p−a
, (p > a) can be made as small as we please by taking t0 sufficiently large. Thus
from (1.4.1), we conclude that L [F(t)] exists for all p > a.
Note. 2 The condition of the theorem are sufficient but not necessary for the existence of the
Laplace transform. If the conditions are satisfied, the Laplace transform will exists. But if these
conditions are not satisfied, Laplace transform may or may not exists.
We know that
1
lim+ = lim+ √ = ∞.
t→0 t→0 t
Thus the function F(t) = 1
√
t
is not piece - wise continuous on every finite interval in the range
t ≥ 0. But F(t) is integrable from 0 to any positive value t0 .
Also,
F(t) ≤ Meat ∀ t > 1
with M = 1, a = 0.
Now Z ∞ Z ∞
1
L [F(t)] = e−pt
F(t)dt = e−pt √ dt
0 0 t
This converges for p > 0.
Put
√dt 2
pt = x ⇒ √ = √ dx
t p
√
π π
Z ∞ r
2 x2 2
∴ L [F(t)] = √ e dx = √ . = ,
p 0 p 2 p
√
π
R∞
e−x dx = .
2
since 0 2
General Properties of Laplace Transform: A very important property is that the Laplace
transform is a linear operator, just as differentiation and integration.
Proof. By definition,
Z ∞
L [c1 F(t) + c2G(t)] = e−pt [c1 F(t) + c2G(t)]dt
0
Z ∞ Z ∞
= e c1 F(t)dt +
−pt
e−ptG(t)dt
0 0
Z ∞ Z ∞
= c1 e F(t)dt + c2
−pt
e−ptG(t)dt
0 0
Elementary functions include algebraic and transcendental functions. From the definition of
Laplace transform, we obtain the following results:
1. L [k] = k
p
(p > 0), where k is a constant.
Proof. By definition,
Z ∞
L [F(t)] = e−pt F(t)dt
Z0 ∞
L [k] = e−pt kdt
0
e−pt ∞
= k
−p 0
k
= − (0 − 1)
p
k
L [k] = , if p > 0.
p
2. L [t] = 1
p2
Proof. By definition,
Z ∞
L [F(t)] = e−pt F(t)dt
Z0 ∞
L [t] = e−pt .tdt
0
3. L [tn ] = n!
pn+1
, where n is a positive integer.
Proof. By definition,
Z ∞
L [F(t)] = e−pt F(t)dt
Z0 ∞
L [tn ] = e−pt .tn dt
0
Z ∞ u n du
L [t ] =
n
e−u
0 p p
Z ∞
1
= n+1
e−u un du
p
Z0 ∞
1
= e−u u(n+1)−1 du
pn+1 0
Z ∞
1
= n+1
Γ(n + 1) since Γ(n) = e−x xn−1 dx, n > 0
p 0
Γ(n + 1)
L [tn ] = , if p > 0 and n + 1 > 0.
pn+1
n!
∴ L [tn ] = , if p > 0 and n is a positive integer.
pn+1
Definition 1.6.1. Gamma Function. If n > 0 then the Gamma function Γ(n) is defined by
Z ∞
Γ(n) = e−x xn−1 dx.
0
∞ ∞
Γ(n + 1)
Z Z
1 1
L [t ] =
n
e x dx =
−x n
e−x x(n+1)−1 dx = ,
pn+1 0 pn+1 0 pn+1
when n > −1.
If n is a positive integer, Γ(n + 1) = n! in particular.
Hence
Γ(n + 1) n!
L [tn ] = n+1
= n+1 ,
p p
where n is a positive integer.
4. L [eat ] = 1
p−a
, (p − a) > 0
Proof.
Z ∞
We have L [F(t)] = e−pt F(t)dt
Z0 ∞
L [eat ] = e−pt eat dt
0
Z ∞
= e−(p−a)t dt
0
h e−(p−a)t i∞
=
−(p − a) 0
1
L [eat ] = , if p > a
p−a
5. L [e−at ] = 1
p+a
, (p + a) > 0.
6. L [sinh at] = a
p2 −a2
, if p > |a|
Proof.
h eat − e−at i
L [F(t)] = L
2
1h i
= L [e ] − L [e−at ] , by linear property
at
2
1h 1 1 i
= −
2 p−a p+a
a
= 2
p − a2
7. L [cosh at] = p
p2 −a2
, if Re(p) > a. Here for the existence of Laplace Transform, we require
p > a or more precisely Re(p) > a
Proof.
h eat + e−at i
L [F(t)] = L
2
1h i
= L [eat ] + L [e−at ] , by linear property
2
8. L [sin at] = a
p2 +a2
, if p > 0.
9. L [cos at] = p
p2 +a2
, if p > 0.
1 p + ia
L [eiat ] = =
p − ia (p − ia)(p + ia)
p + ia
L [cos at + i sin at] =
p2 + a2
Equating the real and imaginary parts on both sides, we have
L [cos at] = p
p2 +a2
and L [sin at] = a
p2 +a2
.
Solution. (i)
4! 2! 1
L [(t2 + 1)2 ] = 5
+2 3 +
p p p
24 4 1
= 5+ 3+
p p p
1
L [(t2 + 1)2 ] = 5 (p4 + 4p2 + 24), p > 0.
p
(ii)
h e−at − 1 i 1 h −at i
L = L e −1
a a
1h i
= L [e−at ] − L [1] by linear property
a
(iii)
1h
L [cosh2 2t] = L [1] + L [cosh 4t]
2
1h 1 p i
= + 2
2 p p − 16
p2 − 8
L [cosh2 2t] =
p(p2 − 16)
1 3
cos3 2t = cos 6t + cos 2t
4 4
1 3
L [cos3 2t] = L [cos 6t] + L [cos 2t]
4 4
1 p 3 p
= +
4 p2 + 36 4 p2 + 4
ph 1 3 i
= +
4 p2 + 36 p2 + 4
ph 4p2 + 112 i
=
4 (p2 + 36)(p2 + 4)
(iii)
(sin t + cos t)2 = sin2 t + cos2 t + 2 sin t cos t = 1 + sin 2t
(i)
1
cos t cos 2t cos 3t = cos t(2 cos 2t cos 3t)
2
1
= cos t(cos 5t + cos t)
2
1
= (cos t cos 5t + cos2 t)
2
1h 1 + cos 2t i
= (cos 6t + cos 4t) + 2
4 2
1h i
= cos 6t + cos 4t + 1 + cos 2t
4
1h p p 1 p i
L [cos t cos 2t cos 3t] = + + +
4 p2 + 36 p2 + 16 p p2 + 4
(ii)
et − e−t
sinh t =
2
(iii)
h e3t − e−3t i2
sinh2 3t =
2
1 9t
= [e + e−9t − 2]
4
1
L [sinh2 3t] = L [e9t + e−9t − 2]
4
1h 1 1 2i
= + −
4 p−9 p+9 p
Solution. (i)
L [e2t + 4t3 − 2 sin 3t + 3 cos 3t] = [e2t ] + 4[t3 ] − 2[ sin 3t] + 3[ cos 3t]
1 3! 3 p
=
+ 4. 4 − 2. 2 +3 2
p−2 p p +9 p +9
1 24 6 3p
= + 4− 2 + 2
p−2 p p +9 p +9
1 24 3(p − 2)
L [e2t + 4t3 − 2 sin 3t + 3 cos 3t] = + 4+ 2 .
p−2 p p +9
(ii)
(sin t − cos t)3 = sin3 t − cos3 t − 3 sin2 t cos t + 3 sin t cos2 t
(sin t − cos t)3 = sin3 t − cos3 t − 3(1 − cos2 t) cos t + 3 sin t(1 − sin2 t)
1 3
cos 3A = 4 cos3 A − 3 cos A ⇒ cos3 A = cos 3A + cos A
4 4
3 1
sin 3A = 3 sin A − 4 sin3 A ⇒ sin3 A = sin A − sin 3A
4 4
h1 3 i h3 1 i
(sin t − cos t)3 = 3 sin t − 3 cos t + 2 cos 3t +
cos t − 2 sin t − sin 3t
4 4 4 4
3 3 1 1
= sin t − cos t + cos 3t + sin 3t
2 2 2 2
h3 3 1 1 i
L [(sin t − cos t)3 ] = L sin t − cos t + cos 3t + sin 3t
2 2 2 2
3 1 3 p 1 p 1 3
= − + +
2 p + 1 2 p + 1 2 p + 9 2 p2 + 9
2 2 2
1 h 3(1 − p) p+3i
L [(sin t − cos t)3 ] = +
2 p2 + 1 p2 + 9
Thus,
Z ∞
L [ f (t)] = f (t)e−pt dt,
0
Z 1 Z ∞
= 2e dt +
−pt
2te−pt dt,
0 1
e−pt 1 h e−pt e−pt i∞
= 2 +2 t − ,
−p 0 −p p2 1
2 h e−p e−p i
= − (e−p − 1) + 2 0 − − − 2 ,
p p p
2 2 2 2
= − e−p + + e−p + 2 e−p ,
p p p p
−p
2 e
L [ f (t)] = 1+ .
p p
Solution. (i)
1h 5 1 i
L [sin 2t cos 3t] = − ,
2 p2 + 25 p2 + 1
1 h 5(p2 + 1) − (p2 + 25) i
= ,
2 (p2 + 25)(p2 + 1)
2(p2 − 5)
L [sin 2t cos 3t] = .
(p2 + 25)(p2 + 1)
(iii) We have
Solution.
1 1 p 3 1
L [7e2t + 9e−2t + 5 cos t + 7t3 + 5 sin 3t + 2] = 7. + 9. + 5. 2 + 5. 2 + 2. .
p−2 p+2 p +1 p +9 p
Solution.
L [(5e2t − 3)2 ] = L [25e4t + 9 − 30e2t ]
1 1 1
= 25 + 9. − 30. .
p−4 p p−2
Solution.
lim e−3t f (t) = lim e−3t t2
t→∞ t→∞
t2 ∞
= lim form
t→∞ e3t ∞
2t
= lim 3t , by L’Hôpital Rule
t→∞ 3e
2
= lim 3t , by L’Hôpital Rule
t→∞ 93
2
= ,
∞
lim e−3t f (t) = 0, a finite quantity.
t→∞
3
Example 1.7.9. Prove that the Laplace transform of et does not exist.
Solution.
3 3 −at)
lim e−at f (t) = lim e−at et = lim e(t =∞
t→∞ t→∞ t→∞
Solution. By definition,
Z ∞
L [ f (t)] = e−pt f (t)dt,
0
Z 1 Z ∞
= −pt
e f (t)dt + e−pt f (t)dt,
0 1
Solution. By definition,
Z ∞
L [ f (t)] = e−pt f (t)dt,
Z0 π Z ∞
= e −pt
f (t)dt + e−pt f (t)dt,
Z0 π Zπ ∞
= e−pt sin tdt + e−pt .0 dt,
0 π
h e−pt iπ
= (−p sin t − cos t) ,
p2 + 1 0
1 + e−πp
L [ f (t)] = .
p2 + 1
eat
eat sin btdt =
R
Note: a2 +b2
(a sin bt − b cos bt).
Solution. By definition,
Z ∞
L [ f (t)] = e−pt f (t)dt,
0
Z 5 Z ∞
= f (t)dt +
e−pt
e−pt f (t)dt,
0 5
Z 5 Z ∞
L [ f (t)] = e e dt +
−pt t
e−pt 3dt,
0 5
Z 5 Z ∞
L [ f (t)] = e −(p−1)t
dt + e−pt 3dt,
0 5
h e−(p−1)t i5 e−pt ∞
= +3
−(p − 1) 0 −p 5
1 3
= − (e−5(p−1) − 1) − (0 − e−5p )
p−1 p
−5(p−1)
1−e 3
= + e−5p .
p−1 p
0<t<2
1, if
2<t<4
2, if
f (t) =
4<t<6
3, if
t>6
0, if
Solution. By definition,
Z ∞
L [ f (t)] = e−pt f (t)dt,
0
Z 2 Z 4 Z 6 Z ∞
= e .1dt +
−pt
e .2 dt +
−pt
e .3 dt +
−pt
e−pt .0 dt,
0 2 4 6
0, 0 < t < 1
f (t) =
t, 1 < t < 2
0, t > 2
Solution. By definition,
Z ∞
L [ f (t)] = e−pt f (t)dt,
0
Z 1 Z 2 Z ∞
= e .0 dt +
−pt
e .t dt +
−pt
e−pt .0 dt,
0 1 2
Z 2
L [ f (t)] = e−pt .tdt
1
h e−pt e−pt i2
= t − 1. 2
−p p 1
1 1
= − (2e−2p − e−p ) − 2 (e−2p − e−p )
p p
2 −2p 1 −p 1 −2p 1 −p
= − e + e − 2e + 2e
p p p p
1 −p
L [ f (t)] = (e − e−2p − 2pe−2p + pe−p ).
p2
Solution. By definition,
Z ∞
L [ f (t)] = e−pt f (t)dt,
Z0 a Z ∞
= e .0 dt +
−pt
e−pt .et−a dt,
Z0 ∞ a
Solution.
h 1 i 1 1
L √ = √ [t− 2 ]
πt π
1 Γ( 2 + 1)
−1
= √
π p− 12 +1
1 Γ( 2 )
h 1 i 1
L √ = √
πt π p 12
√
1 π 1 √
= √ √ , Γ( ) = π
π p 2
h 1 i 1
L √ = √ .
πt p
x3 x5 x7
Example 1.7.17. Using the expression sin x = x − 3!
+ 5!
− 7!
+ · · · , show that
√
√ π 1
L [sin t] = 3
e− 4p .
2p 2
Solution. √ √ √
√ √ ( t)3 ( t)5 ( t)7
sin t = t − + − + ···
3! 5! 7!
3 5 7
√ 1 t 2 t 2 t 2
sin t = t 2 − + − + · · ·
3! 5! 7!
√ 1
h t 32 i h t 52 i h t 72 i
L [sin t] = L [t 2 ] − L +L −L + ···
3!
5! 7!
Γ 32 1Γ 2
5
1Γ 2
7
= 3
− + + ···
p2 3! p 25 5! p 27
1
2
Γ( 1
2
.
3 1
1 2 2 2 Γ 1
. .
5 3 1
1 2 2 2 2 Γ 1
= 3
− + 7
+ · · · , Γ(n + 1) = nΓ(n)
p2 3! p 52 5! p2
√ h
π 3 5.3 7.5.3 i
= 3
1− + 2
− 3
+ ···
2p 2 2.3!p 2.2.5!p 2.2.2.7!p
√ h
π 1 1 1 i
= 3
1 − + − + · · ·
2p 2 22 p (22 p)2 2! (22 p)3 3!
√
√ π − 221 p
L [sin t] = 3
e
2p 2
√
√ π − 4p1
L [sin t] = 3
e
2p 2
1.8 Exercises
0, 0 < t < 1
f (t) =
1, 1 < t < 2
2, t > 2
Answers
A.(1) p23 + a
p2
+ bp ; (2) 6
p4
+ 5p
p2 +1
; (3) p+9
p2 −9
;
1 1 2
(4) p−log 2
; (5) p2 +4
; (6) p(p2 +4);
2a2 p +2
2 3p−20
(7) p(p2 +4a2 )
; (8) p(p2 +4)
; (9) p2 −25
;
2
2a3
(10) p(pp2−8
−16)
; (11) 8
p(p2 −16)
; (12) p4 −a4
;
(13) p cosp2θ−ω
+ω2
sin θ
; (14) p2 +7p
(p2 +1)(p2 +9)
; (15) p(p2 +63)
(p2 +9)(p2 +81)
;
48 48 12p
(16) (p2 −4)(p2 −36)
; (17) (p2 +4)(p2 +36)
; (18) (p2 +1)(p2 +25)
;
√ h
2 p(p +29)
2 π 3 √ i
(20) 2(p −12)
(p2 +36)(p2 +4)
; (21) (p2 +9)(p2 +49)
; (22) 4 5 −
6
3 + 12
√
p
+8 p ;
p 2 p2
√ h
π 3 √ i
5 + + −8 p .
6 12
(23) 4 3 √
p
p2 p2
(B). 2
p2
(1 − e−5p ) − 9p e−5p . (C) 1p (e−p + e−2p ).
p(1−e−2πp )
(D). 1
1−p
(e1−p − 1). (D) 1+p2
.
Lesson 2
I
n this lesson, we study first and second shifting theorems.
Proof. By definition,
Z ∞
L [F(t)] = e−pt F(t)dt
Z0 ∞
L [eat F(t)] = e−pt eat F(t)dt
Z0 ∞
= e−(p−a)t F(t)dt
0
Z ∞
L [F(t)] = e−(p−a)t F(t)dt
Z0 ∞
= e−ut F(t)dt, where u = p − a
0
= f (u)
L [eat f (t)] = f (p − a)
Corollary 2.1.1. Using the above theorem, we have [e−at F(t)] = f (p + a), p + a > 0.
Working Rule:
Solution. (i)
(ii)
(iii)
Solution. (i)
(ii)
Solution.
h 1 i h 1 i
L et (cos 2t + sinh 2t) = L cos 2t + sinh 2t
2 2 p→p−1
h p 1 2 i
= + . 2
p + 4 2 p − 4 p→p−1
2
p−1 1
= +
(p − 1) + 4 (p − 1)2 − 4
2
h 1 i p−1 1
L et (cos 2t + sinh 2t) = 2 + 2 .
2 p − 2p + 5 p − 2p − 3
2a2 p
Example 2.2.4. Show that L [sinh at sin at] = p4 +4a4
.
Solution.
h eat − e−at i
L [sinh at sin at] = L sin at
2
1
= L [eat sin at − e−at sin at]
2
1h i
= L [sin at] p→p−a − L [sin at] p→p+a
2
1h a i h a i
= −
2 p2 + a2 p→p−a p2 + a2 p→p+a
1 h a a i
= −
2 (p − a)2 + a2 (p + a)2 + a2
a h (p + a)2 + a2 − ((p − a)2 + a2 ) i
=
2 (p − a)2 + a2 )(p + a)2 + a2 )
ah 4ap i
=
2 (p2 − 2ap + 2a2 )(p2 + 2ap + 2a2 )
2a2 p
=
(p2 + 2a2 )2 − (2ap)2
2a2 p
L [sinh at sin at] =
p4 + 4a4
Solution.
h eat − e−at i
L [sinh at cos at] = L cos at
2
1
= L [eat cos at − e−at cos at]
2
1h i
L [sinh at cos at] = L [cos at] p→p−a − L [cos at] p→p+a
2
1h p i h p i
L [sinh at cos at] = −
2 p2 + a2 p→p−a p2 + a2 p→p+a
1h p−a p+a i
= −
2 (p − a)2 + a2 (p + a)2 + a2
1 h (p − a)((p + a)2 + a2 ) − (p + a)((p − a)2 + a2 ) i
=
2 (p − a)2 + a2 )(p + a)2 + a2 )
1h 4p2 a − 2a(p2 + 2a2 ) i
=
2 (p2 − 2ap + 2a2 )(p2 + 2ap + 2a2 )
2p2 a − ap2 − 2a3
=
(p2 + 2a2 )2 − (2ap)2
a(p2 − 2a2 )
L [sinh at sin at] = .
p4 + 4a4
Solution.
h eat + e−at i
L [cosh at cos at] = L cos at
2
1
= L [eat cos at + e−at cos at]
2
1h i
= L [cos at] p→p−a + L [cos at] p→p+a
2
1h p i h p i
= +
2 p2 + a2 p→p−a p2 + a2 p→p+a
Solution.
L [(1 + te−t )3 ] = L [1 + 3te−t + 3t2 e−2t + t3 e−3t ]
1 h1i h 2! i h 3! i
= + 3. 2 + 3. 3 + 4
p p p→p+1 p p→p+2 p p→p+3
1 3 6 6
L [(1 + te−t )3 ] = + + + .
p (p + 1)2 (p + 2)3 (p + 3)4
The function
0, if t < a
u(t − a) (or) H(t − a) (or) ua (t) =
1, if t > a
e−pt ∞
=
−p a
1
= − (e−∞ − e−ap )
p
−ap
e
=
p
Proof. By definition,
Z ∞
L [F(t)] = e−pt F(t)dt
Z0 ∞
L [g(t)] = e−pt g(t)dt
0
Z a Z ∞
= −pt
e .0 dt + e−pt F(t − a) dt
0 a
= e−ap L [F(t)]
Proof.
Z ∞
L [F(t)] = e−pt F(t)dt
Z0 ∞
L [F(t − a)u(t − a)] = e−pt F(t − a)u(t − a)dt
0
= e−ap L [F(t)]
p
∴ L [ f (t)] = L [cos t] = = F(p).
p2 +1
Now
π π π π
f (t − 3 ), if t > if t >
cos(t − 3 ),
3 3
g(t) = =
π π
if t < if t <
0,
3
0,
3
p
∴ L [ f (t)] = L [cos t] = = F(p).
p2 +1
Now
π
if t > if t >
2π
2π 2π
f (t − ), cos(t − ),
3 3 3 3
g(t) = =
π
if t < if t < 2π
0,
3
0,
3
2πp
− 2πp
h p i pe− 3
L [g(t)] = e 3 = 2 .
p2 + 1 p +1
Example 2.5.3. Find the Laplace transform of (i) (t − 2)3 u(t − 2); (ii) e−3t u(t − 2).
6
L [(t − 2)3 u(t − 2)] = e−2p . .
p4
(ii)
L [e−3t u(t − 2)] = L [e−3(t−2) e−6 u(t − 2)].
1 e−2(p+3)
L [e−3t u(t − 2)] = e−6 L [e−3t u(t − 2)] = e−6 e−2p . = .
p+3 p+3
(ii) Replace a by t
a
in (i), we get L [F( at )] = a f (ap).
Solution.
1 − cos 2at
sin2 at =
2
1
L [sin2 at] = L [(1 − cos 2at)]
2
1 1 1
= . − L [cos 2at]
2 p 2
p
1 p
L [cos 2at] = 2a
= 2 .
2a ( 2a ) + 1 p + 4a2
p 2
Hence
1 1 p 2a2
L [sin2 at] = − . 2 = .
2p 2 p + 4a2 p(p2 + 4a2 )
Solution.
1 − 1p
L [F(t)] = e
p
By the change of scale of property, we have
1 p 1 3 − 3p 1 − 3p
L [F(3t)] = f = . e = e .
3 3 3 p p
Now,
h1 3 i
L [e−t F(3t)] = e− p ,
p p→p+1
9p2 −12p+15
Solution. Given L [F(t)] = (p−1)3
= f (p).
By change of scale of property,
p2 −p+1
Example 2.7.4. Applying change of scale of property, if L [F(t)] = (2p+1)2 (p−1)
then show that
p2 −2p+4
L [F(2t)] = 4(p+1)2 (p−2)
.
p2 −p+1
Solution. Given L [F(t)] = (2p+1)2 (p−1)
= f (p).
1 p 1 ( 2 )2 − ( 2 ) + 1
p p
1 (p2 − 2p + 4)
L [F(2t)] = f = = .
2 2 2 (2 2p + 1)2 ( 2p − 1) 4 (p + 1)2 (p − 2)
2.8 Exercises
Answers
6 3p−17 16−5p
A. (1) (p+3)4; (2) p2 +2p+26
; (3) p2 −4p
;
1 b p +2p+3
2
(4) (p+a)n
; (5) (p+a)2 −b2
; (6) (p+1)(p2 +2p+5)
;
p3 p2 −13
h i
(7) p4 +4a4
; (8) 3 1
2 p2 −9
+ p4 −10p 2 +169 ; (9) 1
p
+ 3
(p+1)2
+ 6
(p+2)3
+ 6
(p+3)4
;
1 3 1 6e−p
(10) p2 +2p+5
; (11) p2 −2p+37
− p2 −2p+5
; (12) p4
;
1−e−(p+2)
(13) p+2
.
3πp −πp
e− 4
B. p2 +1
. C. e 3
p2 +1
.
D. (i) 60−4p
p2 −12p+80
; (ii) 4(9−p)
p2 −6p+73
.
Lesson 3
LAPLACE TRANSFORM OF
DERIVATIVES AND INTEGRALS
Learning Objectives
Upon completion of this lesson, students will be able to
I
n this Lesson, we study the Laplace transform of derivatives, initial and final value theorems
with worked out sample problems.
Theorem 3.1.1. If F(t) is continuous and of exponential order and if F 0 (t) is sectionally
continuous then the Laplace transform of F 0 (t) is given by L [F 0 (t)] = p f (p) − F(0), where
L [F(t)] = f (p).
Proof. By definition,
Z ∞
L [F(t)] = e−pt F(t)dt
Z0 ∞
L [F (t)] =
0
e−pt F 0 (t)dt
0
Z q
= lim e−pt d(F(t))
q→∞ 0
h q Z q i
L [F (t)] = lim e F(t) −
0 −pt
(−pe−pt )F(t)dt
q→∞ 0
0
h Z q i
= lim e F(t) − F(0) + p
−pq
e−pt F(t)dt .
q→∞ 0
= −F(0) + pL [F(t)]
Note. The Laplace transform of the second order derivative F 00 (t) is similarly obtained.
Similarly we can prove that L [F n (t)] = pn f (p) − pn−1 F(0) − pn−2 F 0 (0) − .... − F n−1 (0).
Theorem 3.2.1. Let F(t) be a continuous for all t ≥ 0 and be of exponential order as t → ∞. If
F 0 (t) is of class A then limt→0 F(t) = lim p→∞ pL [F(t)] = lim p→∞ p f (p).
Note. If F(t) fails to be continuous at t = 0, but limt→0 F(t) exists then the result still holds.
Theorem 3.3.1. Let F(t) be a continuous for all t ≥ 0 and be of exponential order as t → ∞. If
F 0 (t) is of class A then limt→∞ F(t) = lim p→0 L [F(t)] = lim p→0 p f (p).
Z ∞
lim e−pt F 0 (t)dt = lim p f (p) − F(0)
p→0 0 p→0
Z ∞
lim e−pt F 0 (t)dt = lim p f (p) − F(0)
0 p→0 p→0
Z ∞
F 0 (t)dt = lim p f (p) − F(0)
0 p→0
Note. If F(t) fails to be continuous at t = 0, but limt→0 F(t) exists then the result still holds.
Example 3.4.1. Using the theorem on transform of derivatives, find the Laplace transform of the
following functions:
(i) eat ; (ii) cos at; (iii) t sin at.
Solution. (i) Let F(t) = eat . Then F 0 (t) = aeat and F(0) = 1.
Now
L [aeat ] = pL [eat ] − 1
aL [eat ] − pL [eat ] = −1
(a − p)L [eat ] = −1
1
L [eat ] = −
a− p
1
L [eat ] = .
p−a
(ii) Let F(t) = cos at. Then F 0 (t) = −a sin at, F 00 (t) = −a2 cos at.
L [−a2 cos at] = p2 L [cos at] − p.1 − 0, since F(0) = 1 and F 0 (0) = 0.
(iii) Let F(t) = t sin at. Then F 0 (t) = sin at + at cos at, F 00 (t) = a cos at + a[cos at − at sin at] =
2a cos at − a2 t sin at.
F(0) = 0 and F 0 (0) = 0.
√ √ √
π 1
√
Example 3.4.2. Evaluate L [ cos√t t ] (or) If L [sin t] = 3 e
− 4p
, find L [ cos√t t ].
2p 2
√ √
Solution. Let F(t) = sin t. Then F 0 (t) = 1√
2 t
cos t. Also F(0) = 0.
Now
h q i
Example 3.4.3. If L 2 πt = 1
3 show that L [ √1tπ ] = √1 .
p
p2
q
Solution. Let F(t) = 2 t
π
. Then F 0 (t) = √2 . 1√
π 2 t
= √1
πt
and F(0) = 0.
= √
π p 32
1
2p 2 2Γ 1
= √
π p 32
√
p π
= √ 3
πp 2
h 1 i 1
L √ = √ .
πt p
3.5 Exercises
Rt
Proof. Let G(t) = 0
F(u)du. Then
Z t Z 0
dh i
g (t) =
0
F(u)du = F(t) and G(0) = f (u)du = 0.
dt 0 0
But
L [G0 (t)] = pL [G(t)] − G(0) = pL [G(t)] − 0
pL [G(t)] = L [F(t)]
1
L [G(t)] =
L [F(t)]
p
hZ t i 1 1
L F(u)du = L [F(t)] = f (p).
0 p p
hR t R t i
Note. Similarly if L [F(t)] = f (p) then L 0 0 F(u)dudu = 1
p2
f (p).
hR t R t R t i
In general, L 0 0 .... 0 F(u)dudu.....du = p1n f (p).
h p i p+1
L [F(t)] = L [e−t cos t] = L [cos t] p→p+1 = = = f (p)
p2 + 1 p→p+1 (p + 1)2 + 1
hZ t
i 1
L F(u)du = f (p)
0 p
hZ t i 1 p+1
L e−u cos u du = . 2 .
0 p p + 2p + 2
hR t R t i
Example 3.7.2. Find L 0 0
cosh au du du .
p
L [F(t)] = L [cosh at] = = f (p).
p2 − a2
hZ t i 1 p
L cosh au du = . 2
0 p p − a2
hZ t i 1
L cosh au du =
0 p2 − a2
hZ t Z t i 1 1 1
Applying again, L cosh au dudu = . 2 = .
0 0 p p − a2 p(p2 − a2 )
integration and differentiation i.e., first differentiate f (x, α) partially with respect to α and then
integrate it.
∂
Leibnitz’s Rule: If f (x, α) and ∂α
[ f (x, α)] be continuous functions of x and α, then
b b
∂
Z Z
dh i
f (x, α)dx = [ f (x, α)]dx.
dx a a ∂x
1. Multiplication by t.
Theorem 3.9.1. If F(t) is sectionally continuous and of exponential order and if L [F(t)] = f (p)
then L [tF(t)] = − ddp f (p) = − f 0 (p).
Proof. We have Z ∞
L [F(t)] = f (p) = e−pt F(t)dt.
0
Then by Leibnitz’s rule for differentiation under the integral sign, we have
Z ∞
df d
= e−pt F(t)dt
dp dp 0
∂ −pt
Z ∞
= e F(t)dt
∂p
Z0 ∞
df
= −te−pt F(t)dt
dp 0
Z ∞
df
= − e−pt [tF(t)]dt
dp 0
= −L [tF(t)]
df
Thus L [tF(t)] = − = − f 0 (p).
dp
2. Multiplication by tn
Theorem 3.9.2. If F(t) is sectionally continuous and of exponential order and if L [F(t)] = f (p)
n
then L [tn F(t)] = (−n)n ddpn f (p), where n = 1, 2, 3, ...
= −L [tF(t)]
d
T hus L [tF(t)] = (−1) [ f (p)],
dp
∞
dm
Z
e−pt [tm F(t)]dt = (−1)m [ f (p)].
0 d pm
d h ∞ −pt m dm+1
Z i
e [t F(t)]dt = (−1)m m+1
dp 0 dp
∂ −pt m
Z ∞
dm+1
[e ][t F(t)]dt = (−1)m m+1 , by Leibnitz’s rule
0 ∂p dp
∞
dm+1
Z
[−te−pt ][tm F(t)]dt = (−1)m
0 d pm+1
Z ∞
dm+1
− e−pt [tm+1 F(t)]dt = (−1)m m+1
dp
Z0 ∞
dm+1
e−pt [tm+1 F(t)]dt = (−1)m+1 m+1
0 dp
This shows that, if the result is true for n = m, it is also true for n = m + 1. Thus, by the method
of mathematical induction the result is true for all values of n, where n is a positive integer.
dn
∴ L [tn F(t)] = (−n)n f (p).
d pn
Solution. (i)
d
L [t sin at] = − L [sin at]
dp
d h a i
= −
d p p2 + a2
h 1 i
= −a − 2 (2p)
(p + a2 )2
2ap
L [t sin at] = .
(p + a2 )2
2
(ii)
d
L [t cos at] = − L [cos at]
dp
d h p i
= −
d p p2 + a2
h (p2 + a2 )(1) − p(2p) i
= −
(p2 + a2 )2
a2 − p2
L [t cos at] = − 2
(p + a2 )2
p2 − a2
L [t cos at] = .
(p2 + a2 )2
Example 3.10.2. Find (i) L [sin at − at cos at] (ii) L [t sin 3t cos 2t].
Solution. (i)
(ii)
d
L [t sin 3t cos 2t] = − L [sin 3t cos 2t]
dp
d h sin 5t + sin t i
= − L
dp 2
1 d h 5 1 i
= − +
2 d p p2 + 25 p2 + 1
1h 1 1 i
= − 5. − 2 (2p) − (2p)
2 (p + 25)2 (p2 + 1)2
5p p
L [t sin 3t cos 2t] = + 2 .
(p + 25)
2 2 (p + 1)2
Example 3.10.3. Find (i) L [t2 cos 3t]; (ii) L [t2 sin 2t].
Solution. (i)
d2
L [t2 cos 3t] = L [cos 3t]
d p2
d2 h p i
=
d p2 p2 + 9
(ii)
d2
L [t2 sin 2t] = L [sin 2t]
d p2
d2 h 2 i
=
d p2 p2 + 4
d h 1 i
= 2 − 2 (2p)
dp (p + 4)2
h (p2 + 4)2 (1) − p(2(p2 + 4)2p) i
= −4
(p2 + 4)4
h (p2 + 4)[(p2 + 4) − 4p2 ] i
= −4
(p2 + 4)4
2
h 4 − 3p i
= −4 2
(p + 4)3
4(3p2 − 4)
L [t2 sin 2t] = .
(p2 + 4)3
Solution. (i)
d
L [t(3 sin 2t − 2 cos 2t)] = − L [3 sin 2t − 2 cos 2t]
dp
d h 6 2p
= − − ]
d p p2 + 4 (p2 + 4)
d 6 − 2p i
L [t(3 sin 2t − 2 cos 2t)] = −
d p p2 + 4
(ii)
L [(t2 − 3t + 2) sin 3t] = L [t2 sin 3t] − 3L [t sin 3t] + 2L [sin 3t] (3.10.1)
3
L [sin 3t] = .
p2 +9
d
L [t sin 3t] = − L [sin 3t]
dp
d h 3 i
= −
d p p2 + 9
h 1 i
= −3 − 2 (2p)
(p + 9)2
6p
L [t sin 3t] = .
(p + 9)2
2
d2
L [t2 sin 3t] = L [sin 3t]
d p2
d2 h 3 i
=
d p2 p2 + 9
d h 6p i
= − 2
dp (p + 9)2
(p2 + 9)2 (−6) + 6p.2(p2 + 9)2p
=
(p2 + 9)4
(p2 + 9)[(p2 + 9)(−6) + 24p2 ]
=
(p2 + 9)4
−6p2 − 54 + 24p2
=
(p2 + 9)3
18p2 − 54
L [t2 sin 3t] = .
(p2 + 9)3
Using L [sin 3t], L [t sin 3t] and L [t2 sin 3t] in (3.10.1), we get
18p2 − 54 6p 3
L [(t2 − 3t + 2) sin 3t] = + 2 + 2 .
(p + 9)
2 3 (p + 9) 2 p +9
(i)
L [te−t cosh t] = L [t cosh t] p→p+1 , by first shifting theorem (3.10.2)
d h p i
L [t cosh t] = −
d p p2 − 1
(p2 − 1)(1) − p(2p)
= −
(p2 − 1)2
p +1
2
L [t cosh t] =
(p2 − 1)2
h p2 + 1 i (p + 1)2 + 1 p2 + 2p + 2
(3.10.2) ⇒ L [te−t cosh t] = = = .
(p2 − 1)2 p→p+1 ((p + 1)2 − 1)2 (p2 + 2p)2
Example 3.10.6. Find the Laplace transform of the following functions:
(i) (1 + te−t )2 ; (ii) te−t sin t.
Solution. (i)
(1 + te−t )2 = 1 + 2te−t + t2 e−2t
(ii)
L [te−t sin t] = L [t sin t] p→(p+1) (3.10.3)
d
L [t sin t] = − L [sin t]
dp
d h 1 i
= −
d p p2 + 1
h 1 i
= − − 2 (2p)
(p + 1)2
2p
L [t sin t] =
(p + 1)2
2
h 2p i 2)(p + 1) 2(p + 1)
(3.10.3) ⇒ L [te−t sin t] = = = 2 .
(p + 1) p→(p+1) ((p + 1) + 1)
2 2 2 2 (p + 2p + 2)2
Example 3.10.7. Find the Laplace transform of the following functions:
(i) t2 e−2t cos t; (ii) t3 e2t sin t.
Solution. (i)
L [t2 e−2t cos t] = L [t2 cos t] p→p+2 (3.10.4)
d2
L [t2 cos t] = L [cos t]
d p2
d2 h p i
=
d p2 p2 + 1
d h (p2 + 1)(1) − p(2p) i
=
dp (p2 + 1)2
d h 1 − p2 i
=
d p (p2 + 1)2
(p2 + 1)2 (−2p) − (1 − p2 )2(p2 + 1)(2p)
=
(p2 + 1)4
(p + 1)[(p + 1)(−2p) − 4p(1 − p2 )]
2 2
=
(p2 + 1)4
−2p − 2p − 4p + 4p3
3
=
(p2 + 1)3
2p3 − 6p
L [t2 cos t] =
(p2 + 1)3
(ii)
L [t3 e2t sin t] = L [t3 sin t] p→p−2 (3.10.5)
d3
L [t3 sin t] = (−1)3 L [sin t]
d p3
d3 h 1 i
= −1
d p3 p2 + 1
d2 h 2p i
= −1 2 − 2
dp (p + 1)2
d h (p2 + 1)2 (1) − p.2(p2 + 1)(2p) i
= 2
dp (p2 + 1)4
d h (p + 1)[p + 1 − 4p2 ] i
2 2
= 2
dp (p2 + 1)4
d h 1 − 3p2 i
= 2
d p (p2 + 1)3
h (p2 + 1)3 (−6p) − (1 − 3p2 )3(p2 + 1)2 (2p) i
= 2
(p2 + 1)6
h (p2 + 1)2 [(p2 + 1)(−6p) − 6p(1 − 3p2 )] i
= 2
(p2 + 1)6
h −6p3 − 6p − 6p + 18p3 i
= 2
(p2 + 1)4
h 12p3 − 12p i
= 2
(p2 + 1)4
24p(p2 − 1)
L [t3 sin t] =
(p2 + 1)4
h 24p(p2 − 1) i
(3.10.5) ⇒ L [t3 e2t sin t] =
(p2 + 1)4 p→p−2
24(p − 2)((p − 2)2 − 1)
=
((p − 2)2 + 1)4
24(p − 2)(p2 − 4p + 3)
L [t3 e2t sin t] = .
(p2 − 4p + 5)4
Solution.
L [teat sin bt] = L [t sin at] p→p−a (3.10.6)
d
L [t sin bt] = − L [sin bt]
dp
d h b i
= −
d p p2 + b2
h 1 i
= − −b 2 (2p)
(p + b2 )2
2bp
L [t sin bt] =
(p + b2 )2
2
h 2bp i
(3.10.6) ⇒ L [teat sin bt] =
(p2 + b2 )2 p→p−a
2b(p − a)
L [teat sin bt] =
((p − a)2 + b2 )2
hR t i
Example 3.10.9. Find L 0
te−t sin 4tdt .
Solution.
hZ t i 1
L te−t sin 4tdt = .L [te−t sin 4t] (3.10.7)
0 p
hZ t i 1 8(p + 1)
(3.10.7) ⇒ L te−t sin 4tdt = . 2
0 p (p + 2p + 17)2
h t 8(p + 1)
Z i
L te−t sin 4tdt = .
0 p(p + 2p + 17)2
2
hR t i
Example 3.10.10. Find L 0
te−t sin 2tdt .
Solution.
hZ t i 1
L te−t sin 2tdt = .L [te−t sin 2t] (3.10.9)
0 p
d
L [t sin 2t] = − L [sin 2t]
dp
d h 2 i
= −
d p p2 + 4
h 1 i
= − −2 2 (2p)
(p + 4) 2
4p
L [t sin bt] =
(p + 4)2
2
h 4p i
(3.10.10) ⇒ L [te−t sin 2t] =
(p2 + 4)2 p→p+1
4(p + 1)
=
((p + 1)2 + 4)2
8(p + 1)
L [te−t sin 2t] =
(p + 2p + 5)2
2
hZ t i 1 4(p + 1)
(3.10.9) ⇒ L te−t sin 2tdt = . 2
0 p (p + 2p + 5)2
hZ t i 4(p + 1)
L te−t sin 2tdt = .
0 p(p + 2p + 5)2
2
√
1 π 1
Example 3.10.11. If L [t 2 ] = 3 then find L [t− 2 ].
2p 2
1
Solution. Let F(t) = t− 2 .
1 d
L [t.t− 2 ] = − ( f (p))
dp
1 d
L [t 2 ] = − ( f (p))
dp
√
π d
3
= − ( f (p))
2p 2 dp
√
d π
( f (p)) = − 3
dp 2p 2
√ Z
π dp
f (p) = − 3
2 p2
√
π 2
=
2 p 12
π
r
L [F(t)] =
p
π
r
1
i.e., L [t− 2 ] = .
p
3.11 Division by t
h F(t) i R∞
Theorem 3.11.1. If L [F(t)] = f (p) then L t
= 0
f (p)d p, provided the integral exists.
Interchanging the order of integration in the repeated integrals as p and t are independent
variables, we have
Z ∞ Z ∞ Z ∞
f (p)d p = F(t) e−pt d pdt
p 0 p
Z ∞ hZ ∞ i
= F(t) e−pt d p dt
0 p
Z ∞ Z ∞ h e−pt i∞
f (p)d p = F(t) dt
p 0 −t p
Z ∞ Z ∞ h 0 − e−pt i
f (p)d p = F(t) dt
p 0 −t
Z ∞
e−pt
= F(t) dt
0 t
Z ∞ h F(t) i
= e−pt dt
0 t
Z ∞ h F(t) i
f (p)d p = L
p t
h F(t) i Z ∞
Hence L = f (p)d p.
t p
(i)
h sin t i Z ∞
L = L [sin t]dt
t p
Z ∞
1
= dt
p p2 +1
= [tan−1 p]∞
p
(ii)
h sin at i h sin at i
L = aL (3.12.1)
t at
Let F(t) = sin t
t
. Then
h sin t i
L [F(t)] = L = cot−1 (p),
t
by using (i).
h sin at i 1 −1 p
L = cot ,
at a a
by change of scale of property.
h sin at i 1 p p
∴ (3.12.1) ⇒ L = a. cot−1 = cot−1 .
t a a a
h i
e−at −e−bt
Example 3.12.2. Find L t
.
h e−at − e−bt i Z ∞
L = L [e−at − e−bt ]d p
t p
Z ∞ h 1 1 i
= − dp
p p+a p+b
h i∞
= log(p + a) − log(p + b)
p
h p + a i∞
= log
p+b p
h p(1 + ap ) i∞
= log
p(1 + bp ) p
h (1 + ap ) i∞
= log
(1 + bp ) p
(1 + ap )
= log 1 − log[ ]
(1 + bp )
h p + ai
= − log
p+b
h e−at − e−bt i h p + bi
L = log .
t p+a
h i
Example 3.12.3. Show that L cos at
t
does not exist.
Solution.
h cos at i Z ∞
L = L [cos at]d p
t p
Z ∞
p
= dp
p p2 + a2
1
= log(p2 + a2 )∞
p
2
h cos at i 1
L = [log(∞) − log(p2 + a2 )],
t 2
Solution.
h sin 3t cos t i Z ∞
L = L [sin 3t cos t]d p
t p
Z ∞
1
= L [sin 4t + sin 2t]d p
p 2
1 ∞h 4
Z
2 i
= + dp
2 p p2 + 16 p2 + 4
1 h 1 −1 p 1 p i∞
= 4. tan + 2. tan−1
2 4 4 2 2 p
1 h p p i
= tan−1 (∞) + tan−1 (∞) − tan−1 + tan−1
2 4 2
1 h π π −1 p p i
= + − tan + tan−1
2 2 2 4 2
h sin 3t cos t i π 1 −1 p p
L = − tan + tan−1 .
t 2 2 4 2
h i
Example 3.12.5. Find L cos 2t−cos 3t
t
.
Solution.
h cos 2t − cos 3t i Z ∞
L = L [cos 2t − cos 3t]d p
t p
1h 1 + p2 i
4
= log 1 − log
2 1 + p2
9
1h p2 + 4 i
= 0 − log 2
2 p +9
1 p + 9
2
= log 2
2 p +4
h cos 2t − cos 3t i p2 + 9 12
L = log 2 .
t p +4
h i
1−et
Example 3.12.6. Find L t
.
Solution.
h 1 − et i Z ∞
L = L [1 − et ]d p
t p
Z ∞ h1
1 i
= dp −
p p p−1
h i∞
= log p − log(p − 1)
p
h p i∞
= log
p−1 p
h p i∞
= log
p(1 − 1p ) p
h 1 i∞
= log
(1 − 1p ) p
1
= log 1 − log
1 − 1p
p
= 0 − log
p−1
h 1 − et i p − 1
L = log .
t p
hR t i
1−e−t
Example 3.12.7. Find L 0 t
dt .
Solution.
h Z t 1 − e−t i 1 h 1 − e−t i
L dt = .L (3.12.2)
0 t p t
h 1 − e−t i Z ∞
L = L [1 − e−t ]d p
t p
Z ∞ h1 1 i
= − dp
p p p+1
h i∞
= log p − log(p + 1)
p
h p i∞
= log
p+1 p
h p i∞
= log
p(1 + 1p ) p
h 1 i∞
= log
(1 + 1p ) p
1
= log 1 − log
1 + 1p
p
= 0 − log
p+1
h 1 − e−t i p + 1
L = log
t p
h Z t 1 − e−t i 1 p + 1
∴ (3.12.2) ⇒ L dt = log .
0 t p p
hR t i
et sin t
Example 3.12.8. Find L 0 t
dt .
Solution.
h Z t et sin t i 1 h et sin t i
L dt = .L (3.12.3)
0 t p t
h et sin t i h sin t i
L =L (3.12.4)
t t p→(p−1)
h sin t i Z ∞
L = L [sin t]d p
t p
h sin t i h i∞
L = tan−1 (p)
t p
Solution.
h 1 − cos at i Z ∞
L = L [1 − cos at]d p
t p
h p i
= 0 − log p
p2 + a2
h p2 + a2 i
p
h 1 − cos at i
L = log .
t p
h i
Example 3.12.10. Find L 1−cos t
t2
.
Solution.
h 1 − cos at i Z ∞ Z ∞
L = L [1 − cos t]d p
t2 p p
h p2 + 1 i
Z ∞ p
= log , putting a = 1 in the above problem
p p
Z ∞ p
= [log p2 + 1 − log p]d p
p
Z ∞
1
= [ log(p2 + 1) − log p]d p
p 2
Z ∞
1
= [log(p2 + 1) − 2 log p]d p
2 p
h 1 − cos at i 1 h ∞ Z ∞ 2p 2 i
L = log((p + 1) − 2 log p).p −
2
p. 2 − dp
t2 2 p
p p +1 p
hp p + 1 i∞ 1
2 Z ∞
1
= log − −2. 2 dp
2 p 2 p 2 p p +1
hp p2 (1 + p12 ) i∞ Z ∞ 1
= log + dp
2 p2 p
p p2 + 1
p 1
= − log 1 + 2 + (tan−1 p)∞ p
2 p
p p2 + 1
= − log + tan−1 (∞) − tan−1 (p)
2 p2
p p2 π
= log 2 + − tan−1 (p)
2 p +1 2
h 1 − cos at i p p +1
2
L = log + cot−1 (p).
t2 2 p2
h i
e−3t sin 2t
Example 3.12.11. Find L t
.
Solution.
h e−3t sin 2t i h sin 2t i
L =L . (3.12.5)
t t p→p+3
h sin 2t i Z ∞
L = L [sin 2t]d p
t p
h sin 2t i Z ∞
2
L = dp
t p +4 p2
h1 p i∞
= 2 tan−1
2 2 p
p
= tan−1 (∞) − tan−1
2
π −1 p
= − tan
2 2
h sin 2t i p
L = cot−1
t 2
h e−3t sin 2t i h p i
∴ (3.12.5) ⇒ L = cot−1
t 2 p→p+3
he −3t
sin 2t i p + 3
L = cot−1 .
t 2
h i
Example 3.12.12. Find L cos 4t sin 2t
t
.
Solution.
h cos 4t sin 2t i Z ∞
L = L [cos 4t sin 2t]d p
t p
Z ∞
1
= L [sin 6t − sin 2t]d p
p 2
1 ∞h 6
Z
2 i
= − dp
2 p p2 + 36 p2 + 4
Z ∞h
3 1 i
= − dp
p p2 + 36 p2 + 4
h 1 p 1 p i∞
= 3. tan−1 − tan−1
6 6 2 2 p
1 −1 1 −1 1 p 1 p
= tan (∞) − tan (∞) − tan−1 − tan−1
2 2 2 6 2 2
1 h p p i
= tan−1 − tan−1 .
2 2 6
h Rt i
Example 3.12.13. Find L e−3t 0 sin t
t
dt .
Solution. Z t
h sin t i h Z t sin t i
L e −3t
dt = L dt . (3.12.6)
0 t 0 t p→p+3
h Z t sin t i 1 h sin t i
L dt = L
0 t p t
Z ∞
1
= L [sin t]d p
p p
1 ∞ 1
Z
= dp
p p p2 + 1
1 π
h i
= − tan−1 (p)
p 2
1
= cot−1 (p)
p
Z t
h sin t i h1 i
∴ (3.12.6) ⇒ L e−3t
dt = cot−1 (p)
t p p→p+3
Z0 t
h sin t i 1
L e−3t dt = cot−1 (p + 3).
0 t p + 3
h i
Example 3.12.14. Find L sinh at
t
.
Solution.
h sinh at i Z ∞
L = L [sinh at]d p
t p
Z ∞
a
= dp
p − a2p2
1 p − a ∞
= a. log
2a p+a p
1 1 − a ∞
p
= log
2 1+ p a p
1h 1 − ap i
= log 1 − log
2 1 + ap
1h p − a i
= 0 − log
2 p+a
h sinh at i 1 p + a
L = log .
t 2 p−a
h i
Example 3.12.15. Show that L cosh at
t
does not exist.
Solution.
h cosh at i Z ∞
L = L [cosh at]d p
t p
Z ∞
p
= dp
p p2 − a2
1
= log[p2 − a2 ]∞
p
2
h cosh at i 1
L = [log(∞) − log(p2 − a2 )].
t 2
Sometimes, evaluation of improper integrals can be done easily by using Laplace transform
technique.
Z ∞ Z ∞
te dt =
−3t
e−pt tdt
0 0 p=3
= L [t] p=3
h1i 1
= 2
= .
p p=3 9
(ii)
Z ∞ Z ∞
e −4t
sin 3tdt = e−pt sin 3tdt
0 0 p=4
Z ∞
3 i h
e−4t sin 3tdt =
p2 + 9 p=4
Z0 ∞
3
e−4t sin 3tdt = .
0 25
R∞ sin 2t
Example 3.14.2. Evaluate 0 t
dt.
Solution. Z ∞ Z ∞
sin 2t sin 2t
dt = e−pt dt
0 t 0 t p=0
Z ∞
sin 2t h sin 2t i
dt = L (3.14.1)
0 t t p=0
h sin 2t i Z ∞
L = L [sin 2t]d p
t p
Z ∞
2
= dp
p +4 p2
h 1 p i
= 2. tan−1
2 2
π p
= − tan−1
2 2
∞
π
Z
sin 2t p
∴ (3.14.1) dt = − tan−1
t 2 2 p=0
Z0 ∞
sin 2t π
dt = .
0 t 2
R∞ R∞
Example 3.14.3. Show that (i) 0
te−3t sin tdt = 3
50
; (ii) 0
t3 e−t sin tdt = 0.
Solution. (i) Z ∞ Z ∞
te −3t
sin tdt = e−pt [t sin t]dt = L [t sin t] p=3 . (3.14.2)
0 0 p=3
d
L [t sin t] = − L [sin t]
dp
d h 1 i
= −
d p p2 + 1
h 2p i
= − − 2
(p + 1)2
2p
L [t sin t] =
(p + 1)2
2
Z ∞
2p i h
∴ (3.14.2) ⇒ te−3t sin tdt =
(p2 + 1)2 p=3
Z0 ∞
3
te−3t sin tdt = .
0 50
(ii) Z ∞ Z ∞
t e sin tdt =
3 −t
e−pt
[t sin t]dt = L [t3 sin t] p=1 .
3
(3.14.3)
0 0 p=1
d3
L [t3 sin t] = (−1)3 L [sin t]
d p3
d3 h 1 i
= − 3 2
dp p + 1
d2 h 2p i
= − 2 − 2
dp (p + 1)2
d h (p2 + 1)2 (1) − p.2(p2 + 1)2p i
= 2
dp (p2 + 1)4
d h (p + 1)[p2 + 1 − 4p2 ] i
2
= 2
dp (p2 + 1)4
d h 1 − 3p2 i
= 2
d p (p2 + 1)3
h (p2 + 1)3 (−6p) − (1 − 3p2 ).3(p2 + 1)2 (2p) i
= 2
(p+ 1)6
h (p2 + 1)2 [(p2 + 1)(−6p) − 6p(1 − 3p2 )] i
= 2
(p2 + 1)6
h (p2 + 1)(−6p) − 6p(1 − 3p2 ) i
= 2
(p2 + 1)4
h −6p3 − 6p − 6p + 18p3 i
= 2
(p2 + 1)4
h 12p3 − 12p i
= 2
(p2 + 1)4
24p(p2 − 1)
=
(p2 + 1)4
Z ∞ h 24p(p2 − 1) i
∴ (3.14.3) ⇒ t3 e−t sin tdt = = 0.
0 (p2 + 1)4 p=1
R∞ e−at −e−bt
R∞ e−3t −e−6t
R∞ e−t −e−2t
Example 3.14.4. Evaluate (i) 0 t
dt; (ii) 0 t
dt; (iii) 0 t
dt.
Solution. (i)
∞ ∞
e−at − e−bt h e−at − e−bt i h e−at − e−bt i
Z Z
dt = e−pt dt = L . (3.14.4)
0 t 0 t p=0 t p=0
h e−at − e−bt i h p + bi
L = log , by Example 3.12.2.
t p+a
Z ∞
e−at − e−bt h p + bi b
∴ (3.14.4) ⇒ dt = log = log .
0 t p + a p=0 a
(ii) Put a = 3, b = 6 in (i), we get
∞
e−3t − e−6t
Z 6
dt = log = log 2.
0 t 3
∞
e−1t − e−2t
Z 2
dt = log = log 2.
0 t 1
R∞ cos at−cos bt
Example 3.14.5. Evaluate 0 t
dt, by using Laplace transform.
Solution.
Z ∞ Z ∞
cos at − cos bt h cos at − cos bt i h cos at − cos bt i
dt = e−pt dt = L . (3.14.5)
0 t 0 t p=0 t p=0
h cos at − cos bt i Z ∞
L = L [cos at − cos bt]d p
t p
Z ∞ h p p i∞
= −
p p2 + a2 p2 + b2 p
1
= [log(p2 + a2 ) − log(p2 + b2 )]∞
p
2
1 p +b
2 2
= log
2 (p2 + a2 )
Z ∞
cos at − cos bt 1 p2 + b2
∴ (3.14.5) ⇒ dt = log
0 t 2 (p2 + a2 ) p=0
1 b2
= log 2
2 a
Z ∞
cos at − cos bt b
dt = log .
0 t a
R∞ e−at sin2 t
Example 3.14.6. Using Laplace transform, evaluate 0 t
dt.
Solution.
∞ ∞
e−at sin2 t h sin2 t i h sin2 t i
Z Z
dt = −pt
e dt = L
. (3.14.6)
0 t 0 t p=a t p=a
h sin2 t i Z ∞
L = L [sin2 t]d p
t p
Z ∞ h 1 − cos 2t i
= L dp
p 2
1 ∞ h1
Z
p i
= − 2 dp
2 p p p +4
1h 1 i∞
= log p − log(p2 + 4)
2 2 p
1 h p i ∞
= log p
2 p +4 p
2
1h p i
= log 1 − log p
2 p2 + 4
p2 + 4
p
1
= log
2 p
p2 + 4
∞
p
e−at sin2 t
Z
1
∴ (3.14.6) ⇒ dt = log
0 t 2 p p=a
√
Z ∞
e−at sin2 t 1 a2 + 4
dt = log .
0 t 2 a
Solution. Z ∞ Z ∞
te sin tdt =
−t
e−pt [t sin t]dt = L [t sin t] p=1 . (3.14.7)
0 0 p=1
d
L [t sin t] = − L [sin t]
dp
d h 1 i
L [t sin t] = −
d p p2 + 1
h 2p i
=
(p2 + 1)2
2p
=
(p + 1)2
2
Z ∞ h 2p i
∴ (3.14.7) ⇒ te−t sin tdt =
(p2 + 1)2 p=1
Z0 ∞
1
te−t sin tdt = .
0 2
3.15 Exercises
Answers
2 +4) 54(p2 +12)
1. (i) p2(3p
2 (p2 +4)2 ; (ii) p2 (p2 +36)2
; (iii) 2ap
(p2 −a2 )2
;
2p3 −6p 2
2a(3p −a ) 2
−6
(iv) (p2 +1)3
; (v) (p2 +a2 )3
; (vi) (p+3)4
;
(vii) 6(p (p+a2 +a)−6a 3p +a2
4 4 2 p2 2
2 )4 ; (viii) 1
(p2 +a2 )2
[(p2 − a2 ) cos b − 2ap sin b]; (ix) p2 (p2 +a2 )2
.
6. (i) 1p log 1 − 1p ; (ii) 1p log 1 + 2p .
7. (i) b
p(p−a)2 −b2
; (ii) p−1
p2 (p−2)
; (iii) 2(p+1)
p(p2 +2p+2)2
.
8. (i) 1
p(p2 +a2 )
; (ii) a
p2 (p2 +a2 )
.
Lesson 4
LAPLACE TRANSFORM OF
SOME SPECIAL FUNCTIONS AND
PERIODIC FUNCTIONS
Learning Objectives
Upon completion of this lesson, students will be able to
I
n this Lesson, we study laplace Transform of some special functions.
0, for t < a
f (t − a) =
, for a ≤ t ≤ a +
1
ε
0, for t > a
In other words,
ε , for a ≤ t ≤ a +
1
f (t − a) =
0, otherwise
As ε > 0 it is clear from figure that as ε → 0, the height of shaded strip increases indefinitely
and the width decreases in such a way that its area is always unity.
The limit of fε (t − a) as ε → 0 is denoted by δ(t − a) and is called Dirac delta function
(sometimes the unit impulse function).
Thus, the unit impulse function δ(t − a) is defined as follows:
∞, for t = a
δ(t − a) =
0, t , a
e−ap (1 − e−εp )
L [ fε (t − a)] = .
εp
= e−ap .1
√ 3p + 8
L [terf(2 t)] = .
+ 4) 2
3
p2 (p
2 h 1 1 3 1 5 1 7
∴ L [erf(t)] = √ L [t 2 ] − L [t 2 ] + L [t 2 ] − L [t 2 ] + · · ·
π 3 5.2! 7.3!
2 h Γ( 2 ) 1 Γ( 2 ) 1 Γ( 2 ) 1 Γ( 2 )
3 5 7 9 i Γ(n + 1)
= √ − + − + · · · since L [tn ] =
π p2 3
3 p25
5.2! p 27
7.3! p 29
pn+1
√ √ √ √
2 h π 1 π 1 1 π 1 3 1 π 1 3 5 1 i
L [erf(t)] = √ . 3 − . . 5 + . . . 7 − . . . 9 + · · · , Γ(n + 1) = nΓ(n)
π 2 p2 2 2 p2 2 2 4 p2 2 2 4 6 p2
√ h
2 π 1 1 1 3 1 1 3 5 1 i
L [erf(t)] = √ . 3 1 − . + . 2 − . . 3 + · · ·
π 2p 2 2 p 2 4p 2 4 6p
1
1h 1 i− 2
= 3 1+
p2 p
1
∴ L [erf(t)] = p .
p p+1
p
Using change of scale of property, L [F(at)] = 1a f a
, where L [F(t)] = f (p). We have
√ √
L [erf(2 t)] = L [erf( 4t)]
1 1
= . q
4 1 p +1
4 4
√ 2
L [erf(2 t)] =
p p+4
p
√ d h 2 i
L [t erf( 4t)] = −
dp p p + 4
p
h 1 1 i
= + p + 4.1
p
−2. − 2 p. p
p (p + 4) 2 p + 4
2 p + 2(p + 4)
=
p2 (p + 4) 2 p + 4
p
√ 3p + 8
L [t erf( 4t)] = .
p2 (p + 4) 2
3
3. Bessel Function.
Bessel function of order n is defined by
tn h t2 t4 i
Jn (t) = 1 − + − · · ·
2n Γ(n + 1) 2.(2n + 2) 2.4(2n + 2)(2n + 4)
∞ r
X (−1) t n+2r
Jn (t) = .
r=0
r!Γ(n + r + 1) 2
∞
X (−1)r 1 n+2r
1. Jn (t) =
r=0
r!Γ(n + r + 1) 2
∞
X (−1)r t 2r
∴ J0 (t) =
r=0
(r!)2 2
t2 t4 t6
=1− + − + ··· .
22 22 .42 22 .42 .62
1 1 1
∴ L [J0 (t)] = L [1] − 2 L [t2 ] + 2 2 L [t4 ] − 2 2 2 L [t6 ] + · · · .
2 2 .4 2 .4 .6
1 1 2! 1 4! 1 6!
= − 2. 3 + 2 2. 5 − 2 2 2. 7 + ··· .
p 2 p 2 .4 p 2 .4 .6 p
1 h 1 1 1.3 1 2 1.3.5 1 3
i
= 1− . 2 + − + · · ·
p 2 p 2.4 p2 2.4.6 p2
1h 1 i− 12
= 1+ 2
p p
1
= q
2
p 1+p p2
1
L [J0 (t)] = p .
1 + p2
1 p
L [F(at)] = f
a a
1 1
L [J0 (at)] = q
a p2
1+ a2
1
L [J0 (at)] = p .
a2 + p2
d
3. L [tJ0 (t)] = − L [J0 (at)]
dp
d h 1 i
=−
d p a2 + p2
p
h 1 1 i
L [tJ0 (t)] = − − 1 2 . .2p
a + p2 2 a2 + p2
p
p
L [tJ0 (t)] = .
(p2 + a2 ) 2
3
= −L [J00 (t)]
(ii)
d
L [tJ1 (t)] = − L [J1 (t)]
dp
d h p i
L [tJ1 (t)] = − 1− p
dp p2 + 1
p2 + 1 − p. √1 2 2p
p
2 p +1
=
p2 +1
1
L [tJ1 (t)] = .
(p2 + 1) 2
3
Definition 4.3.1. Periodic Function. A function F(t) is said to be periodic if and only if
F(t + T ) = F(t) for some value of T and for every value of t. The smallest positive value of
T for which this equation is true for all every value of t is called the period of the function.
Example 4.4.1. Find the Laplace transform of the square - wave function of period 2a defined as
k, when 0 < t < a
F(t) =
−k, when a < t < 2a
k(1 − e−ap )
=
p(1 + e−ap )
−ap ap ap
ke 2 (e 2 − e− 2 )
= ap ap ap
pe− 2 (e 2 + e− 2 )
k ap
L [F(t)] = tanh .
p 2
Example 4.4.2. Find L [F(t)], where F(t) is a periodic function of period 2π and it is given by
sin t, 0 < t < π
F(t) =
0, π < t < 2π
Example 4.4.3. If F(t) = t2 , 0 < t < 2 and F(t + 2) = F(t), find L [F(t)].
Z π
1
L [F(t)] = −πp
e−pt | sin t|dt
1−e
Z0 π
1
= −πp
e−pt sin tdt since | sin t| = sin t in 0 < t < π
1−e 0
1 h e−pt iπ
= (−p sin t − cos t)
1 − e−πp p2 + 1 0
−πp
1 h e 1 i
= (0 + 1) − (0 − 1)
1 − e−πp p2 + 1 p2 + 1
1 + e−πp
L [F(t)] = .
(1 + p2 )(1 − e−πp )
Example 4.4.6. Find the Laplace transform of the saw - toothed wave of period T, given
k
F(t) = t, when 0 < t < T.
T
Example 4.4.7. Find the Laplace transform of the rectified semi - wave function defined by
π
sin ωt, 0 < t <
ω
F(t) =
Z 2π
1 ω
L [F(t)] = e−pt F(t)dt
− 2πp
1−e ω 0
Z π
1 ω
= e−pt sin ωt dt
− 2πp
1−e ω 0
1 h e−pt i ωπ
= (−p sin ωt − ω cos ωt)
1 − e− ω p + ω
2πp 2 2 0
πp
1 h e− ω 1 i
= (0 + ω) − (0 − ω)
1 − e− ω p + ω p2 + ω2
2πp 2 2
πp
1 h ωe− ω + ω i
=
1 − e− ω p + ω
2πp 2 2
πp
ω(1 + e− ω )
L [F(t)] = πp πp
(1 + e− ω )(1 − e− ω )(p2 + ω2 )
ω
L [F(t)] = − πp
.
(1 − e ω )(p2 + ω2 )
4.5 Exercises
1. If
3t, 0 < t < 2
F(t) =
6, 2 < t < 4
with period T = a.
4. Find the Laplace transform of the function of period T defined as F(t) = Tt , 0 < t < T.
Answers. h
k ap
ap
3
1. p2 (1−e −4p ) 1 − e
−2p
(1 + 2e2p ). 2. 2p
e2 sec h 2
e−pT
3. 1p tanh pa 4
. 4. 1
p2 T
− p(1−e−pT )
.
Lesson 5
5.1 Introduction
I
n the first Unit we have considered Laplace transform of some functions F(t). Let us now
consider the converse namely, given f (p) how to determine F(t) such that L [F(t)] =
f (p).
The inverse Laplace transform is useful in solving differential equations without finding the
general solution and arbitrary constants.
Definition 5.1.1. If f (p) is the Laplace transform of a function F(t), then F(t) is called the
inverse Laplace transform of f (p) and is denoted by L −1 [ f (p)] i.e,
F(t) = L −1 [ f (p)].
Note. Inverse Laplace transform of a given function f (p) can be obtained either by use of the
above standard results or by splitting the given function into its partial fractions and then applying
the above results.
Linear Property. If f1 (p) and f2 (p) are Laplace transform of F1 (t) and F2 (t) respectively, then
Solution.
h 3p i h p i
L −1 = 3L −1
= 3 cosh 5t.
p2 − 25 p2 − 25
Example 5.2.2. Find the inverse Laplace transform of the following functions:
(i) 2p−5
p2 −4
; (ii) 2p+1
p(p+1)
; (iii) 3p−8
4p2 +25
.
h 2p − 5 i h 2p 5 i
Solution. (i) L −1 = L −1− (ii)
p2 − 4 p2 − 4 p2 − 4
h p i h 1 i
= 2L −1 2 − 5L −1 2
p −4 p −4
h 2p − 5 i 1
L −1 2 = 2 cosh 2t − 5. sinh 2t.
p −4 2
h 2p + 1 i h p + (p + 1) i
L −1 = L −1 (iii)
p(p + 1) p(p + 1)
h 1 1i
= L −1 +
p+1 p
h 1 i h1i
= L −1 + L −1
p+1 p
h 2p + 1 i
L −1 = e−t + 1.
p(p + 1)
h 3p − 8 i h 3p 8 i
L −1 = L −1
−
4p2 + 25 4p2 + 25 4p2 + 25
3 h p i 8 h 1 i
= L −1 2 − L −1
4 p + ( 25 )2 4 p2 + ( 25 )2
3 5t 2 5t
= cos −2 sin
4 2 5 2
h 3p − 8 i 3 5t 4 5t
L −1 = cos − sin .
4p2 + 25 4 2 5 2
h i
Example 5.2.3. Find L −1 3p−2 5 − p+2 7
.
p2
Solution.
h 3p − 2 7 i h pi h1i 7 h 1 i
L −1 − = 3L −1 5 − 2L −1 5 − L −1
p2
5
p+2 p2 p2 3 p + 23
1 3
t2 t2 7 2
= 3 3 − 2 5 − e− 3 (t) .
Γ( 2 ) Γ( 2 ) 3
h 2p+1 i
Example 5.2.4. Find L −1 p2 −4
.
Solution.
h 2p + 1 i p i h h 1 i
L −1 = 2L −1 + L −1
p2 − 4 p2 − 4 p2 − 4
1
= 2 cosh(2t) + sinh(2t).
2
h p2 −3p+4 i h 3(p2 −2)2 i
Example 5.2.5. Find (i) L −1 p3
. (ii) L −1 2p5
.
Solution.
h p2 − 3p + 4 i h p2 p 4i
(i) L −1 = L −1 + −3
p3 p3 p3 p3
h1 1 1i
= L −1 − 3. 2 + 4 3
p p p
2
t
= 1 − 3t + 4.
2!
= 1 − 3t + 2t2 .
Solution.
h 2p − 5 4p − 18 i −1 2p − 5 −1 4p − 18
h i h i
(i) L −1 + = L + L
4p2 + 25 9 − p2 4p2 + 25 9 − p2
h p2 + 9p − 9 i h p2 − 9 + 9p i
(ii) L −1 = L −1
p3 − 9p p(p2 − 9)
h p2 − 9 i h 9p i
= L −1 + L −1
p(p2 − 9) p(p2 − 9)
h1i h 1 i
= L −1 + 9L −1 2
p p −9
1
= 1 + 9. sinh 3t
3
h p2 + 9p − 9 i
L −1 = 1 + 3 sinh 3t.
p3 − 9p
h √ p−1 2 i
Example 5.2.7. Find L −1 5
p2
+ p
− 7
3p+2
.
Solution.
√ p − 1 2 h p − 2 √p + 1
−1 5 7 i −1 1
h h i h 1 i
L + − = 5L + L −1
− 7L −1
p2 p 3p + 2 p2 p2 3p + 2
h1i h1i h1i 7 h 1 i
= 5t + L −1 − 2L −1 3 + L −1 2 − L −1
p p2 p 3 p + 23
1
t2 7 2
= 5t + 1 − 2 3 + t − e− 3 t
Γ( 2 ) 3
h 5 √ p − 1 2 7 i 2t 2
1
7 − 23 t
L −1
+ − = 6t + 1 − − e .
p2 p 3p + 2 Γ( 23 ) 3
√ i
6−30 p
h
Example 5.2.8. Find L −1 3
p2 −3
+ 3p+2
p3
− 3p−27
p2 +9
+ p4 .
Solution. √
h 3 3p + 2 3p − 27 6 − 30 p i h 3 i
−1 3p + 2
h i
−1 3p − 27
h i
L −1
+ − + = L −1
+ L − L
p2 − 3 p3 p2 + 9 p4 p2 − 3 p3 p2 + 9
√
h 6 − 30 p i
+ L −1
p4
h 1 i h 3p i h2i
= 3L −1 2 + L −1 3 + L −1 3
p −3 p p
h p i h 1 i h1i
− 3L −1 2 + 27L −1 2 + 6L −1 4
p +9 p +9 p
h1i
− 30L −1 7
p2
1 √ t2
= 3. √ sinh( 3t) + 3t + 2. − 3 cos 3t
3 2!
5
1 t3 t2
+ 27. sin 3t + 6. − 30 7
3 3! Γ( 2 )
√
h 3 3p + 2 3p − 27 6 − 30 p i √ √
L −1 2 + − + = 3 sinh 3t + 3t + t2 − 3 cos 3t + 9 sin 3t
p −3 p 3 p +9
2 p 4
5
t2
+ t − 30 7 .
3
Γ( 2 )
h i
Example 5.2.9. Find L −1 6
2p−3
− 3+4p
9p2 −16
+ 8−6p
16p2 +9
.
h 6 3 + 4p 8 − 6p i
Solution. L −1 − 2 +
2p − 3 9p − 16 16p2 + 9
h 3 + 4p i
6 i −1 8 − 6p
h h i
= L −1 − L −1 + L
2p − 3 9p2 − 16 16p2 + 9
6 h 1 i h 3 i h p i h 8 i h p i
= L −1 − L −1
− 4L −1
+ L −1
− 6L −1
2 p − 32 9p2 − 16 9p2 − 16 16p2 + 9 16p2 + 9
3 3 h 1 i 4 h p i 8 h 1 i 6 h p i
= 3e 2 t − L −1 2 16 − L −1 2 16 + L −1 2 9 − L −1 2 9
9 p − 9 9 p − 9 16 p + 16 16 p + 16
3 1 3 4 4 4 1 4 3 3 3
= 3e 2 t − . sinh t − cosh t + . sin t − cos t
3 4 3 9 3 2 3 4 8 4
h 6 3 + 4p 8 − 6p i 3 1 4 4 4 2 3 3 3
L −1 − 2 + = 3e 2t − sinh t − cosh t + sin t − cos t.
2p − 3 9p − 16 16p2 + 9 4 3 9 3 3 4 8 4
Solution.
p2 p4 p6
S incecos p = 1 − + − + .....
2! 4! 6!
1 1 1h 1 1 2 1 1 4 1 1 6 i
cos = 1− + − + ...
p p p 2! p 4! p 6! p
1 1 1 1 1 1 1
= − 3
+ 5
− + ....
p 2! p 4! p 6! p7
h1 1 i h1i 1 h1i 1 h1i 1 h1i
L −1 cos = L −1 − L −1 3 + L −1 5 − L −1 7
p p p 2! p 4! p 6! p
2 4 6
1 t 1 t 1 t
= 1− + − + ....
2! 2! 4! 4! 6! 6!
h1 1 i t2 t4 t6
L −1 cos = 1− + − + .....
p p (2!)2 (4!)2 (6!)2
h i
t3 t5 t7
Example 5.2.11. Show that L −1 1p . sin 1p = t − (3!)2
+ (5!)2
− (7!)2
+ ....
Solution.
p3 p5 p7
S incesin p = p − + − + .....
3! 5! 7!
1 1 1h 1 1 3 1 1 5 1 1 7 i
sin = p− + − + ...
p p p 3! p 5! p 7! p
1 1 1 1 1 1 1
= 2− + − + ....
p 3! p4 5! p6 7! p8
h1 1 i h1i 1 h1i 1 h1i 1 h1i
L −1 sin = L −1 2 − L −1 4 + L −1 6 − L −1 8
p p p 3! p 5! p 7! p
3 5 7
1 t 1 t 1 t
= t− + − + ....
3! 3! 5! 5! 7! 7!
h1 1 i t3 t5 t7
L −1 sin = t− + − + .....
p p (3!)2 (5!)2 (7!)2
Lesson 6
6.1 Introduction
I
f f (p) is given in the form g(p)
h(p)
, where g and h are polynomial in p, then F(t) can be
obtained by resolving f (p) into partial fractions and manipulating term by term.
Solution. Let
p p A B
= = + (6.2.1)
p2 −a 2 (p − a)(p + a) p − a p + a
Then
p = A(p + a) + B(p − a) (6.2.2)
p 1 1
= +
−ap2
2 2(p − a) 2(p + a)
h p i h 1 1 i
L −1 2 = L −1
+
p − a2 2(p − a) 2(p + a)
1 −1 h 1 i 1 −1 h 1 i
= L + L
2 p−a 2 p+a
1 at 1 −at
= e + e
2 2
1 at
= (e + e−at )
2
p
= cosh at.
p2 − a2
Solution.
h 4 i h 1 i
L −1 = 4L −1
(p + 1)(p + 2) (p + 1)(p + 2)
h 1 1 i
= 4L −1 − (Resolving into partial f ractions)
p+1 p+2
h 1 i h 1 i
= 4L −1 − 4L −1
p+1 p+2
h 4 i
L −1 = 4e−t − 4e−2t
(p + 1)(p + 2)
h i
Example 6.2.3. Find L −1 3p+1
p2 −2p−3
.
Solution. Let
3p + 1 3p + 1 A B
= = + (6.2.3)
p2 − 2p − 3 (p + 1)(p − 3) p + 1 p − 3
Then
3p + 1 = A(p − 3) + B(p + 1)
3p + 1 1 5
= +
p2 − 2p − 3 2(p + 1) 2(p − 3)
h 3p + 1 i 1 h 1 i 5 h 1 i
L −1 2 = L −1 + L −1
p − 2p − 3 2 p+1 2 p−3
h 3p + 1 i 1 5
L −1 2 = e−t + e3t .
p − 2p − 3 2 2
h i
Example 6.2.4. Find L −1 p−2
p2 +5p+6
.
Solution. Let
p−2 p−2 A B
= = + (6.2.4)
p2 + 5p + 6 (p + 2)(p + 3) p + 2 p + 3
Then
p − 2 = A(p + 3) + B(p + 2)
p−2 4 5
= − +
+ 5p + 6p2 p+2 p+3
h p−2 i h 4 5 i
L −1 2 = L −1 − +
p + 5p + 6 p+2 p+3
h 1 i h 1 i
= −4L −1 + 5L −1
p+2 p+3
h p−2 i
L −1 2 = −4e−2t + 5e−3t .
p + 5p + 6
h i
Example 6.2.5. Find L −1 1
p(p+1)(p+2)
.
Solution. Let
1 A B C
= + + (6.2.5)
p(p + 1)(p + 2) p p + 1 p + 2
Then
1 = A(p + 1)(p + 2) + Bp(p + 2) + C p(p + 1)
Put p = 0, we get 1 = 2A ⇒ A = 1
2
1 1 1 1
= − +
p(p + 1)(p + 2) 2p p + 1 2(p + 2)
h 1 i 1 h1i h 1 i 1 h 1 i
L −1 = L −1 − L −1 + L −1
p(p + 1)(p + 2) 2 p p+1 2 p+2
1 1
= − e−t + e−2t
2 2
1
= (1 − 2e + e−2t )
−t
2
h 1 i 1
L −1
= (1 − e−t )2 .
p(p + 1)(p + 2) 2
p2
h i
Example 6.2.6. Evaluate L −1 (p+1)(p+2)(p+3)
Solution. Let
p2 A B C
= + + (6.2.6)
(p + 1)(p + 2)(p + 3) p + 1 p + 2 p + 3
Then
p2 = A(p + 2)(p + 3) + B(p + 1)(p + 3) + C(p + 2)(p + 1)
p2 1 4 9
= − +
(p + 1)(p + 2)(p + 3) 2(p + 1) p + 2 2(p + 3)
2
h p i 1 h 1 i h 1 i 9 h 1 i
L −1 = L −1 − 4L −1 + L −1
(p + 1)(p + 2)(p + 3) 2 p+1 p+2 2 p+3
2
h p i 1 9
L −1 = e−t − 4e−2t + e−3t
(p + 1)(p + 2)(p + 3) 2 2
Solution. Let
1 A Bp + C
= + 2 (6.2.7)
(p + 1)(p + 1) p + 1
2 p +1
Then
1 = A(p2 + 1) + (Bp + C)(p + 1) = (A + B)p2 + (B + C)p + (A + C)
1 1 p 1
= − +
(p + 1)(p + 1)
2 2(p + 1) 2(p + 1) 2(p + 1)
2 2
h 1 i 1 −1 h 1 i 1 −1 h p i 1 −1 h 1 i
L −1 = L − L + L
(p + 1)(p2 + 1) 2 p+1 2 p2 + 1 2 p2 + 1
h 1 i 1 −t 1 1
L −1 = e − cos t + sin t.
(p + 1)(p2 + 1) 2 2 2
p2 +p−2
h i
Example 6.2.8. Find the inverse Laplace transformation of p(p+3)(p−2)
Solution. Let
p2 + p − 2 A B C
= + + (6.2.8)
p(p + 3)(p − 2) p p + 3 p − 2
Then
p2 + p − 2 = A(p + 3)(p − 2) + Bp(p − 2) + C p(p + 3)
p2 + p − 2 1 4 2
= + +
p(p + 3)(p − 2) 3p 15(p + 3) 5(p − 2)
h p + p−2 i
2
1 h1i 4 h 1 i 2 h 1 i
L −1 = L −1 + L −1 + L −1
p(p + 3)(p − 2) 3 p 15 p+3 5 p−2
h p + p−2 i
2
1 4 2
L −1 = + e−3t + e2t .
p(p + 3)(p − 2) 3 15 5
p2 +2p−4
h i
Example 6.2.9. Evaluate L −1 (p2 +9)(p−5)
Solution. Let
p2 + 2p − 4 A Bp + C
= + (6.2.9)
(p2 + 9)(p − 5) p − 1 p2 + 9
Then
p2 + 2p − 4 = A(p2 + 9) + (Bp + C)(p − 5)
p2 + 2p − 4 31 3p + 83
= +
(p2 + 9)(p − 5) 34(p − 5) 34(p2 + 9)
h p2 + 2p − 4 i 31 3 h p i 83 h 1 i
L −1 2 = + L −1 2 + L −1 2
(p + 9)(p − 5) 34(p − 5) 34 p +9 34 p +9
h p2 + 2p − 4 i 31 5t 3 83 sin 3t
L −1 2 = e + cos 3t +
(p + 9)(p − 5) 34 34 34 3
h p2 + 2p − 4 i 1 h 5t 83 i
L −1 2 = 31e + 3 cos 3t + sin 3t .
(p + 9)(p − 5) 34 3
p2
h i
Example 6.2.10. Evaluate L −1 (p2 +4)(p2 +25)
Solution. Let
p2 Ap + B C p + D
= + 2 (6.2.10)
(p2 + 4)(p2 + 25) p2 + 4 p + 25
Then
p2 = (Ap + B)(p2 + 25) + (C p + D)(p2 + 4)
Comparing the coefficients of p3 , and p2 , p and the constant term on both sides, we get
p2 −4 25
= +
(p2 + 4)(p2 + 25) 21(p2 + 4) 21(p2 + 25)
2
h p i 4 −1 h 1 i 25 −1 h 1 i
L −1 2 = − L + L
(p + 4)(p2 + 25) 21 p2 + 4 21 p2 + 25
4 sin 2t 25 sin 5t
= − +
21 2 21 5
2 5
= − sin 2t + sin 5t
21 21
h p2 i 1h i
L −1 2 = 5 sin 5 − 2 sin 2t .
(p + 4)(p2 + 25) 21
h i
Example 6.2.11. Evaluate L −1 p
(p2 +1)(p2 +9)(p2 +25)
h i
Solution. We have 1
(p2 +1)(p2 +9)
= 1 1
8 p2 +1
− 1
p2 +9
, by resolving into partial fractions.
1 1h 1 1 i
= −
(p2 + 1)(p2 + 9)(p2 + 25) 8 (p2 + 1)(p2 + 25) (p2 + 9)(p2 + 25)
1h 1 1 1 1 1 1 i
= − − −
8 24 p2 + 1 p2 + 25 16 p2 + 9 p2 + 25
1h 1 1 1 1 1 1
= . 2 − . 2 + . 2
8 24 p + 1 16 p + 9 48 p + 25
1 h 16 24 8 i
= − +
8 × 24 × 16 p2 + 1 p2 + 9 p2 + 25
p 1 h 16p 24p 8p i
= − +
(p2 + 1)(p2 + 9)(p2 + 25) 3092 p2 + 1 p2 + 9 p2 + 25
h p i 1 h p p p i
L −1 2 = 16.L −1
− 24.L −1
+ 8.L −1
(p + 1)(p2 + 9)(p2 + 25) 3092 p2 + 1 p2 + 9 p2 + 25
1
= 16 cos t − 24 cos 3t + 8 cos 5t .
3092
Solution. Let
2p + 3 2p + 3 A B C
= = + + (6.2.11)
p3 − 6p + 11p − 6 (p − 1)(p − 2)(p − 3) p − 1 p − 2 p − 3
2
Then
2p + 3 = A(p − 2)(p − 3) + B(p − 1)(p − 3) + C(p − 1)(p − 2)
Put p = 1, we have A = 5
2
Put p = 2, we have B = −7
Put p = 3, we get C = 29 .
Substituting these values of A and B in (6.2.11), we get
2p + 3 5 7 9
= − +
− 6p + 11p − 6
p3 2 2(p − 1) p − 2 2(p − 3)
h 2p + 3 i 5 h 1 i h 1 i 9 h 1 i
L −1 3 = L −1
− 7L −1
+ L −1
p − 6p2 + 11p − 6 2 p−1 p−2 2 p−3
h 2p + 3 i 5 9
L −1 3 = et − 7e2t + e3t .
p − 6p + 11p − 6
2 2 2
6.3 Exercises
Answers
1. 15 (3 sin 3t − 2 sin 2t); 2. 1
768
(4 cos t − 6 cos 3t + 2 cos 5t); 4. 31 (e2t − e−t );
5. 21 (e−t − e−3t ); 6. e3t − e2t ; 7. 4e3t − e−t ;
et 3t2 3t4
8. 1
2
+ 54 e3t − 3 −2t
10
e ; 9. 1 + 2
− 32 e−t ; 10. 3
2
− 2
+ 48
;
11. 1
2
cos 2t; 12. 1
2
cosh 2t; 13. et (cos t + 2 sin t);
t
14. 21 (sinh t − sin t); 15. cos t + cosh t; 16. 2e−t + 5e2t − 23 e 2 ;
17. et + e−t 23 sin 2t − cos 2t ; 18. sin t sinh t.
Lesson 7
7.1 Introduction
T
his Lesson contains the first shifting theorem, the second shifting theorem, change of scale
property and how to apply these ideas to find out the inverse Laplace Transform.
Theorem 7.1.1. (First Shifting Theorem) If L −1 [ f (p)] = F(t), then L −1 [ f (p − a)] = eat F(t).
Solution.
h 1 i h 1 i
L −1 = L −1
p2 + 2p + 5 (p + 1)2 + 4
since p2 + 2p + 5 = (p + 1)2 − 1 + 5 = (p + 1)2 + 4 by completing square method
h 1 i h 1 i
L −1 2 = e−t L −1 2 , by first shifting theorem
p + 2p + 5 p + 22
sin 2t
= e−t .
2
h i
Example 7.2.2. Find L −1 p+3
p2 −10p+29
.
Solution.
h p+3 i h p+3 i
L −1 = L −1
p2 − 10p + 29 (p − 5)2 + 4
h p − 5 + 5 + 3i
= L −1
(p − 5)2 + 4
h p−5 i h 8 i
= L −1 + L −1
(p − 5)2 + 4 (p − 5)2 + 4
h p i h 1 i
= e5t L −1 2 + 8e5t L −1 2 , by first shifting theorem
p +4 p +4
sin 2t
= e5t cos 2t + 8e5t
2
h p + 3 i
L −1 2 = e5t cos 2t + 4 sin 2t .
p − 10p + 29
Solution.
h p i h p i
L −1 = L −1
p2 + 4p + 5 (p + 2)2 + 1
h p+2−2 i
= L −1
(p + 2)2 + 1
h p i h p+2 i h 2 i
L −1 = L −1
− L −1
p2 + 4p + 5 (p + 2)2 + 1 (p + 2)2 + 1
h p i h 1 i
= e−2t L −1 2 − 2e−2t L −1 2 , by first shifting theorem
p +1 p +1
= e−2t cos t − 2e−2t sin t
h p i
L −1 = e−2t cos t − 2 sin t .
p2 + 4p + 5
h i h i
Example 7.2.4. Find (i) L −1 1
(p+2)2 +16
; (ii) L −1 √ 1 ;
2p−3
Solution. (i)
h 1 i h 1 i
−2t sin 4t
L −1 = e −2t
L −1
= e .
(p + 2)2 + 16 p2 + 16 4
(ii)
1
h 1 i h 1 i 1 3t h1i 1 3t t 2 −1 1 3t
L −1 p = L −1 q = √ e 2 L −1 1 = √ e 2 1 = √ e 2 .
2p − 3 2(p − 32 ) 2 p2 2 Γ( 2 ) 2πt
h i h i
Example 7.2.5. Find (i) L −1 2p+3
p2 +2p+2
; (ii) Find (i) L −1 1
(p+a)n
.
Solution.
h 2p + 3 i h 2p + 3 i
L −1 = L −1
p2 + 2p + 2 (p + 1)2 + 1
h 2(p + 1) − 2 + 3 i
= L −1
(p + 1)2 + 1
h 2(p + 1) i h 1 i
= L −1 + L −1
(p + 1)2 + 1 (p + 1)2 + 1
h p i h 1 i
= 2e−t L −1 2 + e−t L −1 2 , by first shifting theorem
p +1 p +1
= 2e−t cos t + e−t sin t
h 2p + 3 i
L −1 = e−t 2 cos t + sin t .
p2 + 2p + 2
(ii)
n−1
1 i −1 1
h i
h
−at t
L =e L
−at
−1
=e .
(p + a)n pn Γ(n)
h 3p−2 i
Example 7.2.6. Find the inverse Laplace transform of L −1 p2 −4p+20 .
Solution.
h 3p − 2 i h 3p − 2 i
L −1 = L −1
p2 − 4p + 20 (p − 2)2 + 16
3p − 2 i −1 3(p − 2) + 6 − 2
h h i
L −1 = L
p2 − 4p + 20 (p − 2)2 + 16
h 3(p − 2) i h 4 i
= L −1 + L −1
(p − 2)2 + 16 (p − 2)2 + 16
h p i h 1 i
= 3e2t L −1 2 + 4e2t L −1 2 , by first shifting theorem
p + 16 p + 16
= 3e2t cos 4t + e2t sin 4t
h 3p − 2 i
L −1 = e2t 3 cos 4t + sin 4t .
p2 − 4p + 20
h i
Example 7.2.7. Find the inverse Laplace transform of L −1 1
(p+1)2 (p2 +4)
.
Solution. Let
1 A B Cp + D
= + + (7.2.1)
(p + 1)2 (p2 + 4) p + 1 (p + 1)2 p2 + 4
Then
A + C = 0, A + B + 2C + D = 0, 4A + C + 2D = 0 and 4A + 4B + D = 1
2 1 2 3
A= , B= ,C=− , D=− .
25 5 25 25
1 2 1 2 3
= + − −
(p + 1)2 (p2 + 4) 25(p + 1) 5(p + 1) 2 25(p + 4) 25(p2 + 4)
2
h 1 i 2 −1 h 1 i 1 −1 h 1 i 2 −1 h p i 3 −1 h 1 i
L −1 = L + L − L − L
(p + 1)2 (p2 + 4) 25 p+1 5 (p + 1)2 25 p2 + 4 25 p2 + 4
2 −t −1 h 1 i 1 −t −1 h 1 i 2 3 sin 2t
= e L + e L 2
− cos 2t −
25 p 5 p 25 25 2
2 −t 1 2 3 sin 2t
= e .1 + e−t .t − cos 2t −
25 5 25 25 2
h 1 i 2 −t 1 −t 2 3
L −1 = e + e t− cos 2t − sin 2t.
(p + 1)2 (p2 + 4) 25 5 25 50
h i h i
Example 7.2.8. Find (i) L −1 p+2
(p−2)3
; (ii) Find (i) L −1 p
(p+3)2
.
Solution.
h p+2 i
−1 p − 2 + 2 + 2
h i
L −1 = L
(p − 2)3 (p − 2)3
h p−2 i h 1 i
= L −1 + 4L −1
(p − 2)3 (p − 2)3
h 1 i
−1 1
h i
= L −1 + 4e 2t
L , by first shifting theorem
(p − 2)2 p3
t2
= e2t .t + 4e2t
2!
h p+2 i
L −1 = te2t 1 + 2t .
(p − 2)3
(ii)
p i −1 p + 3 − 3
h h i
L −1 = L
(p + 3)2 (p + 3)2
h p+3 i h 3 i
= L −1 − L −1
(p + 3)2 (p + 3)2
h 1 i h1i
= L −1 − 3e−3t L −1 2
p+3 p
h1i
= e−3t L −1 − 3e−3t .t
p
= e−3t .1 − 3e−3t t
h p i
L −1 = e−3t (1 − 3t).
(p + 3)2
h i
Example 7.2.9. Find the inverse Laplace transform of L −1 4p+5
(p−1)2 (p+2)
.
Solution. Let
4p + 5 A B C
= + + (7.2.2)
(p − 1) (p + 2) p − 1 (p − 1)
2 2 p+2
Then
4p + 5 = A(p − 1)(p + 2) + B(p + 2) + C(p − 1)2 (7.2.3)
= A(p2 + p − 2) + Bp + 2B + C(p2 − 2p + 1)
1
A+C = 0 ⇒ A = .
3
Substituting these values of A, B, C and D in (7.2.2), we get
4p + 5 1 3 1
= + −
(p − 1) (p + 2)
2 3(p − 1) (p − 1) 2 3(p + 2)
h 4p + 5 i 1 −1 1h i h 1 i 1 −1 h 1 i
L −1 = L + 3L −1
− L
(p − 1)2 (p + 2) 3 p−1 (p − 1)2 3 p+2
1 t h1i 1
= e + 3et L −1 2 − e−2t
3 p 3
h 4p + 5 i 1 1
L −1 = et + 3et .t − e−2t .
(p − 1) (p + 2)
2 3 3
h i
Example 7.2.10. Find L −1 3p−4
p2 −4p+8
Solution.
p2 − 4p + 8 = (p − 2)2 − 4 + 8 = (p − 2)2 + 4,
3p − 4 i
h h 3p − 4 i
L −1 = L −1
p2 − 4p + 8 (p − 2)2 + 4
h 3(p − 2) + 6 − 4 i
= L −1
(p − 2)2 + 4
h 3(p − 2) i h 1 i
= L −1 + 2L −1
(p − 2)2 + 4 (p − 2)2 + 4
h p i h 1 i
= 3e2t L −1 2 + 2e2t L −1 2
p +4 p +4
sin 2t
= 3e−2t cos 2t + 2e2t
2
h 3p − 4 i
L −1 2 = 3e−2t cos 2t + e2t sin 2t.
p − 4p + 8
p2
h i
Example 7.2.11. Prove that L −1 (p+2)3
= e−2t (1 − 4t + 2t2 ).
Solution.
p2 i −1 (p + 2) − 4p − 4)
h h 2 i
L −1 = L
(p + 2)3 (p + 2)3
h (p + 2)2 i h p+1 i
= L −1 − 4L −1
(p + 2)3 (p + 2)3
h 1 i h p + 2 − 1i
= L −1 − 4L −1
p+2 (p + 2)3
h p2 i h p+2 i
−1 1
h i h 1 i
L −1 = e−2t
L − 4L −1
+ 4L −1
(p + 2)3 p (p + 2)3 (p + 2)3
h p2 i h 1 i
−1 1
h i
L −1 = e−2t
.1 − 4L −1
+ 4e−2t
L
(p + 2)3 (p + 2)2 p3
h1i t2
= e−2t − 4e−2t L −1 2 + 4e−2t
p 2!
= e−2t − 4e−2t .t + 2e−2t t2
h p2 i
L −1 = e−2t (1 − 4t + 2t2 ).
(p + 2)3
h i
Example 7.2.12. Find L −1 p+2
p2 −2p+5
.
p+2 i
h h p+2 i
L −1 = L −1
p2 − 2p + 5 (p − 1)2 + 4
h p − 1 + 1 + 2i
= L −1
(p − 1)2 + 4
h p−1 i h 1 i
= L −1 + 3L −1
(p − 1)2 + 4 (p − 1)2 + 4
h p i h 1 i
= et L −1 2 + 3et L −1 2
p +4 p +4
sin 2t
= et cos 2t + 3et
2
h p+2 i 3
L −1 2 = et cos 2t + sin 2t .
p − 2p + 5 2
Solution.
p p
=
+ 4a
p4 4 (p + 2a − 2ap)(p2 + 2a2 + 2ap)
2 2
1h 1 1 i
= −
4a p2 − 2ap + 2a2 p2 + 2ap + 2a2
1h 1 1 i
= −
4a (p − a)2 + a2 (p + a)2 + a2
h p i 1 −1 h 1 i h 1 i
L −1 4 = L − L −1
p + 4a4 4a (p − a)2 + a2 (p + a)2 + a2
h p i 1 at −1 h 1 i h 1 i
L −1 = e L − e −at
L −1
p4 + 4a4 4a p2 + a2 p2 + a2
1 h at sin at sin at i
= e − e−at
4a a a
1
= sin at(eat − a−at )
4a2
1
= sin at. 2 sinh at
4a2
h p i 1
L −1 4 = sin at sinh at.
p + 4a 4 2a2
h i
Example 7.2.14. Find L −1 p
5
(p+1) 2
Solution.
p p+1−1
=
(p + 1) (p + 1) 2
5 5
2
p+1 1
= −
(p + 1) (p + 1) 2
5 5
2
1 1
= −
(p + 1) 2 (p + 1) 2
3 5
h p i h 1 i h 1 i
L −1 = L −1 − L −1
(p + 1) (p + 1) (p + 1)
5 3 5
2 2 2
h1i h1i
= e−t L −1 3
− e−t L −1
5
p 2 p2
h p i h t 21 3
t2 i
L −1
= e −t
− .
(p + 1) 2
5
Γ( 32 ) Γ( 25 )
7.3 Exercises
Answers
1. 12 (cos ht + cos t − 2); 2. e−3t cos 2t − 12 sin 2t ; 3. 17 e4t + 3e4t t − 71 e−3t ;
q √
e−t t √2 3t
4. πt − 2e
√ −t
π
; 5. 3
sin 2
sinh 2t ; 6. 3t (et − e−2t );
7. 21 e(t−3) sin 2(t − 3)H(t − 3); 8. cosh 3(t − a)H(t − a).
Proof. By definition,
Z ∞
L [G(t)] = e−ptG(t)dt
Z0 a Z a
= G(t)dt +
e −pt
e−ptG(t)dt
0 0
Z a Z ∞
= e .0 dt +
−pt
e−pt F(t − a)dt
0 a
= e L [F(t)]
ap
= e−ap f (p)
or
L −1 [e−ap f (p)] = F(t − a)H(t − a),
Solution.
h 1 i t3
L −1 =
p4 3!
3
h 1 i 2t t
L −1 = e = F(t),
(p − 2)4 6
by first shifting theorem.
3
e2(t−5)(t−5)
> 6 , if t > 5
F(t − 5), if t 5
1
h i
L −1 e−5p = =
(p − 2)4
if t < 5 if t < 5
0,
0,
h e−5p i 1
L −1 = e2(t−5) (t − 5)3 H(t − 5),
(p − 2)4 6
where
if t > 5
1,
H(t − 5) =
if t < 5
0,
h i
e−3p
Example 7.5.2. Find L −1 p3
.
Solution.
h 1 i t2
S ince L −1 = = F(t),
p3 2!
by first shifting theorem. Using second shifting theorem, we have
h e−3p i (t − 3)2
L −1 = F(t − 3)H(t − 3) = H(t − 3),
p3 2
Solution. Since
3
h 1 i −1 1
h i
3t t
L −1 = e3t
L = e = F(t),
(p − 3)4 p4 6
by first shifting theorem. Using second shifting theorem, we have
Solution. Since
h p i
L −1 = cos 5t = F(t).
p2 + 25
Using second shifting theorem, we have
h pe −uπp
5 i uπ uπ uπ uπ
L −1
= F t − H t − = cos 5 t− H t−
p2 + 25 5 5 5 5
uπ
where H t − is the Heaviside unit step function.
5
h −2p i
Example 7.5.5. Find L −1 p2 +4p+5
e
.
Solution. Since
h 1 i h 1 i h 1 i
L −1 = L −1
= e−2t
L −1
== e−2t sin t = F(t),
p2 + 4p + 5 (p + 2)2 + 1 p2 + 1
h e−2p i
L −1
= F(t − 2)H(t − 2) = e−2(t−2) sin(t − 2)H(t − 2),
p + 4p + 5
2
Solution. Let
3 + 5p 3 5
f (p) = 2
= 2+
p p p
h3 5i
L −1 [ f (p)] = L −1 + = 3t + 5 = F(t),
p2 p
by first shifting theorem. Using second shifting theorem, we have
h 3 + 5p i h e−2p (3 + 5p) i
L −1 = L −1 = F(t − 2)H(t − 2) = (3(t − 2) + 5)H(t − 2),
p2 e2p p2
Solution. Let
p+1
f (p) =
p2 + p+1
Then
p+1 p+1
=
p2 + p+1 (p + 21 )2 + 3
4
p + 21 + 1
= 2
(p + 21 )2 + 3
4
p+ 1 1
= 2
+ 2
(p + ) + 34
1 2
2
(p + 1 2
2
) + 34
h p+1 i h p + 12 i 1 h 1 i
L −1 = L −1 + L −1
p2 + p+1 (p + 12 )2 + 43 2 (p + 12 )2 + 34
1
h p i 1 1 h 1 i
= e− 2 t L −1 2 3 + e− 2 t L −1 2 3
p +4 2 p +4
√
√
3 1 1 sin 23 t
− 12 t
= e cos t + e− 2 t √
2 2 3
2
√ √
3 1 3 i
− 12 t
h
= e cos t + √ sin t
2 3 2
Solution.
h 1 i h1i
L −1 = e−4t L −1
(p + 4) 2
5 5
p2
3
t2
= e −4t
Γ( 52 )
3
t2
= e−4t 3
. 1 Γ( 12 )
2 2
3
4 t2
= e−4t √
3 π
= F(t)
3
h e4−3p i 4 (t − 3) 2
L −1
= e F(t − 3)H(t − 3) = e−4(t−3) √ H(t − 3),
4
(p + 4) 2
5
3 π
Solution.
h 1 + e−πp i 1 i h −πp i
−1 e
h
L −1 = L −1 + L
p2 + 1 p2 + 1 p2 + 1
= sin t + sin(t − π)H(t − π), by second shi f ting theorem
Proof. By definition,
Z ∞
f (p) = e−pt F(t)dt
Z0 ∞
f (ap) = e−apt F(t)dt
0
Put at = u, then dt = du
a
.
When t = 0, u = 0 and t = ∞, u = ∞.
Z ∞ h u i du
f (ap) = e−pu F
0 a a
Z ∞
1 hui
= e−pu F du
a 0 a
Z ∞
1 h ti
= e−pt F dt, on changing the dummy variable u by t
a 0 a
1 h t i
= L
a a
h 1 t i
f (ap) = L F
a a
1 t
Hence L −1 [ f (ap)] = F .
a a
Solution. Given,
h p i 1
L −1 = t sin t.
(p2 + 1)2 2
h ap i 1 1 t t
L −1 = . . . sin ,
((ap)2 + 1)2 2 a a a
Putting a = 2, we get
h 2p i t t
L −1 = sin
(4p2 + 1)2 8 2
8p i t h t
∴ L −1 = sin .
(4p2 + 1)2 2 2
h − 1p i √ h − ap i
Example 7.7.2. If L −1 e 1 = cos√2πt t , find L −1 e 1
p2 p2
Solution. Given, √
h e− 1p i cos 2 t
L −1 = √ .
πt
1
p2
Replace p by ap, we get q
t
1
− ap
i 1 cos 2
−1 e
h a
L 1
= q ,
(ap) 2 a π at
q
t
1 1 cos1
2
− ap
−1 e
h i a
√ L = √ √ ,
πt
1
a p2 a
Putting a = a1 , we get
√
h e− ap i cos 2 at
L −1 √ = √ .
p πt
Example 7.7.3. For a > 0, prove that L −1 [ f (p)] = F(t) implies that
1 −bt t
L −1 [ f (ap + b)] = e a F .
a a
Solution. By definition,
Z ∞
f (p) = L [F(t)] = e−pt F(t)dt
0
Z ∞
f (ap + b) = e−(ap+b)t F(t)dt
Z0 ∞
= e−apt e−bt F(t)dt
0
Put at = x, so that dt = dx
a
.
Also t = 0 ⇒ x = 0 and t = ∞ ⇒ x = ∞.
x Z ∞
bx
f (ap + b) = dx e−px e− a F
a
Z0 ∞ h 1 bt t
= e−pt e− a F dt
0 a a
h 1 bt t i
f (ap + b) = L e− a F
a a
1 −bt
t
Hence L −1 [ f (ap + b)] = e a F .
a a
7.8 Exercises
h i h i
1. Given L −1 p
(p2 +1)2
= 2t sin t, show that L −1 32p
(16P2 +1)2
= 4t sin t
4
.
p2 −1 9p2 −1
h i h i
2. Given L −1 (p2 +1)2
= t cos t, show that L −1 (9P2 +1)2
= 9t cos t
3
.
Lesson 8
8.1 Introduction
T
his Lesson focuses on finding Inverse Laplace Transform of derivatives, integrals,
multiplication and division by powers of p.
dn
Theorem 8.2.1. If L −1 [ f (p)] = F(t), then L −1 [ f n (p)] = (−1)n tn F(t) where f n (p) = d pn
[ f (p)].
dn
L [tn F(t)] = (−1)n ( f (p))
d pn
= (−1)n f n (p)
= L −1 [ f n (p)]
Solution.
1
L −1 [ f (p)] = − L −1 [ f 0 (p)] (8.3.1)
t
h 1i h p2 + 1 i
Let f (p) = log 1 + 2 = log 2
, f (p) = log(p2 + 1) − log p2
p p
2p 2
f 0 (p) = −
p2 + 1 p
2h p 1 i
(4.2.1) ⇒ L −1 [ f (p)] = − L −1 2 − L −1 2
t p +1 p
2
= − [cos t − 1]
t
h 1 i 2
L −1 log 1 + 2 = (1 − cos t).
p t
Solution.
h p + 3i
Let f (p) = log = log(p + 3) − log(p + 4).
p+4
1
f 0 (p) = −
p+3 p+4
1h 1 1 i
(4.2.1) ⇒ L −1 [ f (p)] = − L −1 − L −1
t p+3 p+4
1
= − [e−3t − e−4t ]
t
h p + 3 i 1
L −1 log = [e−4t − e−3t ].
p+4 t
h p+1 i
Example 8.3.3. Find the inverse Laplace transform of log p−1
.
Solution.
h p + 1i
Let f (p) = log = log(p + 1) − log(p − 1).
p−1
1 1
f 0 (p) = −
p+1 p−1
1h 1 1 i
(4.2.1) ⇒ L −1 [ f (p)] = − L −1 − L −1
t p+1 p−1
1
= − [e−t − et ]
t
h p + 1 i 1 t
L −1 log = [e − e−t ]
p−1 t
h p + 1 i 2
i.e., L −1 log = sinh t.
p−1 t
h 1+p i
Example 8.3.4. Find the inverse Laplace transform of log p2
.
Solution.
h1 + pi
Let f (p) = log = log(p + 1) − log(p2 ),
p2
f (p) = log(p + 1) − 2 log p
1 1
f 0 (p) = −2
p+1 p
1 h 1 1 i
(4.2.1) ⇒ L −1 [ f (p)] = − L −1 − 2L −1
t p+1 p
1
= − [e−t − 2]
t
h 1 + p i 1
L −1 log = [2 − e−t ]
p2 t
p2 +1
h i
Example 8.3.5. Find the inverse Laplace transform of log (p−1)2
.
Solution.
h p2 + 1 i
Let f (p) = log = log(p2 + 1) − log(p − 1)2 , f (p) = log(p2 + 1) − 2 log(p − 1)
(p − 1)2
2p 1
f 0 (p) = −2
+1p2 p−1
1 h 2p 1 i
(4.2.1) ⇒ L −1 [ f (p)] = − L −1 2 − 2L −1
t p +1 p−1
1
= − [2 cos t − 2et ]
t
h p2 + 1 i 2
L −1 log 2
= [et − cos t]
(p − 1) t
h i
Example 8.3.6. Find L −1 tan−1 ap + cot−1 bp .
Solution.
a p
Let f (p) = tan−1 + cot−1 .
p b
Then
1 −a 1 1
f 0 (p) = − . .
1+ a2
p2
p2 1+ p2
b2
b
Solution.
p + a
Let f (p) = cot−1 .
b
Then
1 1
f 0 (p) = − (p+a)2
. .
1+ b2
b
−b
f 0 (p) = −
(p + a)2 + b2
1 h −1 b i
(4.2.1) ⇒ L −1 [ f (p)] = − L −
t (p + a)2 + b2
1 h b i
= e−at L −1 2
t p + b2
h p + a i e −at
L −1 cot−1 = sin bt.
b t
h i
Example 8.3.8. Find L −1 tan−1 (p + 1) .
1 h −1 1 i
(4.2.1) ⇒ L −1 [ f (p)] = − L
t (p + 1)2 + 1
1 h 1 i
= − e−t L −1 2
t p +1
h p + a i e −t
L −1 cot−1 = − sin t.
b t
Solution.
1
Let f (p) = .
p2 − 25
Then
1
f 0 (p) = − .2p
(p2 − 25)2
1
Since L −1 [ f (p)] = − L −1 [ f 0 (p)]
t
i.e., L −1 [ f 0 (p)] = −tL −1 [ f (p)]
h −2p i h 1 i
L −1 2 = −tL −1
(p − 25)2 p2 − 25
t h 1 i
= L −1 2
2 p − 25
h p i t
L −1 2 2
= sinh 5t.
(p − 25) 10
hZ ∞ Z ∞
i1 h i
L −1
f (p)d p = L −1 [ f (p)] or L −1 [ f (p)] = tL −1 f (p)d p
p t p
Solution.
2p
Let f (p) =
(p2 − 4)2
hZ ∞ i
L [ f (p)] = tL
−1 −1
f (p)d p
p
hZ ∞
h 2p i 2p i
L −1 = tL −1 d p
(p − 4)2
2 2
p (p − 4)
2
h −1 ∞ i
= tL −1 2
p −4 p
h 1 i
= tL −1 0 + 2
p −4
sinh 2t
= t
2
h 2p i t
L −1 = sinh 2t
(p2 − 4)2 2
h i
Example 8.5.2. Find L −1 p+2
((p2 +4p+5)2
.
Solution.
p+2
Let f (p) =
(p2 + 4p + 5)2
hZ ∞ i
L [ f (p)] = tL
−1 −1
f (p)d p
p
h p+2 i hZ ∞ p+2 i
L −1 = tL −1
d p
(p2 + 4p + 5)2 p (p + 4p + 5)
2 2
∞
2p + 4
h1 Z i
= tL −1 d p
2 p (p2 + 4p + 5)2
t −1 h 1 ∞ i
L − 2 =
2 p + 4p + 5 p
t h 1 i
= L −1
2 (p + 2)2 + 1
t h 1 i
= e−2t L −1 2
2 p +1
h p + 2 i t
L −1 2 = e−2t sin t.
(p + 4p + 5)2 2
h i
Example 8.5.3. Find L −1 p
(p2 −a2 )2
.
Solution.
p
Let f (p) =
(p2 − a2 )2
hZ ∞ i
L [ f (p)] = tL
−1 −1
f (p)d p
p
h p i hZ ∞ p i
L −1
= tL −1
d p
(p2 − a2 )2 2
p (p − a )
2 2
h1 Z ∞
2p i
= tL −1 d p
2 p (p2 − a2 )2
t h −1 ∞ i
= L −1 2
2 p − a2 p
t h 1 i
= L −1 0 + 2
2 p − a2
t sinh at
=
2 a
h p i t
L −1 2 2 2
= sinh at.
(p − a ) 2a
hR ∞ i
Example 8.5.4. Find L −1 p
log u−1
u+1
du .
Solution.
u − 1
Let f (u) = log = log(u − 1) − log(u + 1)
u+1
1 1
f 0 (u) = −
u−1 u+1
1
L −1 [ f (u)] = − L −1 [ f 0 (u)]
t
1 h 1 1 i
L −1 [log(u − 1) − log(u + 1)] = − L −1 −
t u−1 u+1
1 −1 h 1 1 i
= − L −
t u−1 u+1
1 t
= − (e − e−t )
t
1 −t
L [log(u − 1) − log(u + 1)] = (e − et ) = F(t)
−1
t
hZ ∞ i F(t)
L −1
f (u)du = .
p t
∞
hZ u − 1 i 1 1 e−t − et
∴L −1
log du = (e−t − et ) = .
p u+1 t t t2
h i h i
Note. (1) L −1 2ap
(p2 −a2 )
= t sinh at; (2) L −1 p
(p2 +a2 )
= t
2a
sin at.
8.6 Exercises
hR ∞ i
B. Find L −1 1
u
− 1
u−1
du .
h R p∞ i
C. Find L −1 p
a
u2 −a2
b
− u2 −b 2 du .
hR ∞ i
D. Find L −1 p u2 +a u
2
u
− u2 +b 2 du .
Answers
t e−bt −e−at
A. 1. e −1
t
; 2. t
; 3. 2
at
sinh at;
4. 2t (1 − cos at); 5. 2t (1 − cosh at); 6. 2t (1 − cosh t);
7. 1t (1 + e−t − 2 cos t); 8. 1t (2 cos 2t − e−t − 1); 9. sin 2t
t
;
e−t sin t 2 t
10. t
; 11. t
sinh t sin t; 12. 2a
sinh at.
1−et
B. t
.
C. 1t (sinh at − sinh bt).
D. 1t (cos at − cos bt).
or
L −1 [p f (p)] = F 0 (t).
Solution. Let
1
f (p) =
2p2 −1
h p i h 1 i
Thus L −1 = L −1
p.
2p2 − 1 2p2 − 1
= L −1 [p f (p)]
= F 0 (t)
1 t 1
= √ cosh √ . √
2 2 2
h p i 1 t
L −1 = cosh √ .
2p2 − 1 2 2
h i
Example 8.8.2. Find L −1 p
(p−4)5
.
Solution. Let
1
f (p) =
(p − 4)5
Clearly F(0) = 0.
L −1 [p f (p)] = F 0 (t)
h p i d h t4 e4t i
L −1 =
(p − 4)5 dt 24
1 4 4t
= [t e + e4t 4t3 ]
24
h p i t3 e4t (t + 1)
L −1 = .
(p − 4)5 6
p2
h i
Example 8.8.3. Find L −1 (p−3)3
.
Solution. Let
1
f (p) =
(p − 3)3
p2
h i
Example 8.8.4. Find L −1 (p2 +a2 )2
.
Solution. Let
p
f (p) =
(p2 + a2 )2
hZ ∞
p i
L [ f (p)] = tL
−1 −1
d p
p (p + a )
2 2 2
h1 ∞Z
2p i
= tL −1 d p
2 p (p2 + a2 )2
h1 1 ∞ i
= tL −1 − 2
2 p + a2 p
t −1 h 1 i
= L
2 p2 + a2
t
L −1 [ f (p)] = sin at.
2a
Clearly F(0) = 0.
L −1 [p f (p)] = F 0 (t)
h p2 i h p.p i
L −1 2 = L −1
(p + a2 )2 (p2 + a2 )2
dh t i
=
sin at
dt 2a
1
= [t.a cos at + sin at]
2a
h p2 i 1
L −1 2 = [at cos at + sin at].
(p + a )
2 2 2a
8.9 Division by p
h f (p) i Rt
Theorem 8.9.1. If L −1 [ f (p)] = F(t), then L −1 p
= 0
F(u)du.
h f (p) i RtRt
Note.If L −1 [ f (p)] = F(t), then L −1 p2
= 0 0
F(u)dudu.
h f (p) i Z t
Since L −1
= F(u)du
p 0
Z t
h 1 i
L −1
= e−2t dt
p(p + 2) 0
h f (p) i Z t
Since L −1
= F(u)du
p 0
Z t
h 1 i sin at
L −1
= dt
p(p2 + a2 ) 0 a
1 − cos at t
=
a a 0
1
(cos at − 1) = −
a2
h 1 i 1
L −1 = 2 (1 − cos at).
p(p + a )
2 2 a
h 1 i 1
L −1 = 2 (1 − cos at).
p(p2 + a2 ) a
h f (p) i Z t
Since L −1
= F(u)du
p 0
Z t
h 1 1i
L −1
=
(1 − cos at)dt
p2 (p2 + a2 )
0 a
2
1 sin at t
= 2 t−
a a 0
h 1 i 1 1
L −1 2 2 = t − sin at .
p (p + a2 ) a2 a
h 1 i 1 1
L −1 = t − sin at .
p2 (p2 + a2 ) a2 a
h f (p) i Z t
S ince L −1
= F(u)du
p 0
Z t
h 1 1 1
i
L −1
=t − sin at dt
p3 (p2 + a2 )
0 a
2 a
1 t2 1 − cos at t
= 2 −
a 2 a a 0
2
1 t
h 1 1i
= 2 + 2 cos at − 2
a 2 a a
2
h 1 i 1 t
1
L −1 3 2 = + (cos at − 1) .
p (p + a2 ) a2 2 a2
h 1 i t2
L −1 = + (cos t − 1).
p2 (p2 + 1) 2
h p i t
L −1 = sin at = F(t), say
(p + a )
2 2 2 2a
h 1 i h p i
∴ L −1 = L −1
(p2 + a2 )2 p(p2 + a2 )2
Solution.
1 1
Let f (p) = = .
p2 + 2p + 2 (p + 1)2 + 1
Then
h 1 h 1 i
L −1 [ f (p)] = L −1 = e−t
L −1
= e−t sin t = F(t)
(p + 1)2 + 1 p2 + 1
h f (p) i Z t
L −1
= F(u)du
p 0
Z t
h 1 i
L −1
= e−u sin udu
p(p2 + 2p + 2) 0
h e−u it
= (− sin u − cos u)
1+1 0
1 h i
= − e−t (sin t + cos t) − (0 + 1)
2
h 1 i 1
L −1
= [1 − e−t (sin t + cos t)].
p(p + 2p + 2)
2 2
p+2
Example 8.10.8. Find the inverse Laplace transform of 1
p
log p+1
.
Solution.
p + 2
Let f (p) = log = log(p + 2) − log(p + 1)
p+1
1 1
f 0 (p) = −
p+2 p+1
h 1 1 i
L −1 [ f 0 (p)] = L −1 −
p+2 p+1
= e−2t − e−t
1
L −1 [ f (p)] = − L −1 [ f 0 (p)]
t
h p + 2 i 1 −2t
∴ L −1 log = − (e − e−t )
p+1 t
h f (p) i Z t
Since L −1 = F(u)du
p 0
h1 p + 2 i Z t −2u
e − e−u
L −1 log = − du.
p p+1 0 u
h1 p + 2 i Z t −u
e − e−2u
L −1 log = du.
p p+1 0 u
p2 +2
Example 8.10.9. Find the inverse Laplace transform of p(p2 +4)
.
Solution.
h p2 + 2 i
−1 (p + 4) − 2
h 2 i
L −1 = L
p(p2 + 4) p(p2 + 4)
h p2 + 4 2 i
= L −1
−
p(p2 + 4) p(p2 + 4)
h1i h 2 i
= L −1 − L −1
p p(p2 + 4)
h p2 + 2 i h 2 i
L −1 = 1 − L −1
(8.10.1)
p(p2 + 4) p(p2 + 4)
2
Let f (p) =
p2 + 4
L −1 [ f (p)] = sin 2t = F(t).
h p2 + 2 i 1
∴ (8.10.1) ⇒ L −1 = 1 + (cos 2t − 1)
p(p + 4)
2 2
h p2 + 2 i 1
L −1 = (1 + cos 2t).
p(p + 4)
2 2
Solution.
h p+1i h p 1 i
L −1 = L −1
+
p2 + 1 p2 + 1 p2 + 1
= cos t + sin t
h f (p) i Z t
L −1
F(u)du =
p 0
h p+1 i Z t
L −1
= (cos u + sin u)du
p(p2 + 1) 0
Solution.
h 1 i h 1 i
L −1 = L −1
p4 − 2p3 p3 (p − 2)
h 1 i
L −1 = e2t = F(t)
p−2
h f (p) i Z t
L −1
= F(u)du
p 0
Z t
h 1 i
L −1
= e2u du
p(p − 2) 0
e2u t
=
2 0
1 h i 1
L −1 = (e2t − 1)
p(p − 2) 2
Z t
h 1 i 1
Similarly L −1 2 = ( (e2u − 1))du
p (p − 2) 0 2
1 e2u t
= −u
2 2 0
2t
1 e
1
= −t−
2 2 2
Z t e2u
h 1 i 1 1
Finally L −1 3 = ( − t − du
p (p − 2) 0 2 2 2
2u 2
1 e
u u t
= − −
2 4 2 2 0
1 1 2t t2 t 1 i
h
= e − − −
2 4 2 2 4
1 h 1 2t 1 i
= (e − 1) − (t2 + t)
2 4 2
h 1 i 1
L −1 3 = (e2t − 2t2 − 2t − 1)
p (p − 2) 8
h i
Example 8.10.12. Find L −1 1
p(p2 −1)(p2 +1)
.
Solution.
h 1 i
−1 1
h 1 1 i
L −1 = L −
p(p2 − 1)(p2 + 1) 2 p2 − 1 p2 + 1
1 −1 h 1 i h 1 i
= L − L −1
2 p2 − 1 p2 + 1
1
= (sinh t − sin t)
2
= F(t)
Z t
h 1 i 1
∴L −1
= (sinh u − sin u)du
p(p − 1)(p + 1)
2 2
0 2
1
= (cosh u + cos u)t0
2
1
= (cosh t + cos t − (1 + 1))
2
h 1 i 1
L −1 = (cosh t + cos t − 2).
p(p − 1)(p + 1)
2 2 2
8.11 Exercises
p2 −w2 p2 p2
4. (p2 +w2 )2
; 5. (p−1)3
; 6. (p+4)3
;
p 2
1 1
7. (p−1)4
; 8. p(p2 +9)
; 9. p(p2 −16)
;
1 1 1
10. p(p+2)3
; 11. p(p−a)4
; 12. p(p2 +4p+5)
;
1 8 1
13. p2 (p−a)
; 14. p4 −4p2
; 15. p2 (p+1)2
;
p−2
16. 1
p2 p2 +4
; 17. 1
p3 (p−2)
.
Answers
Lesson 9
CONVOLUTION THEOREM
Learning Objectives
Upon completion of this lesson, students will be able to
9.1 Convolution
C
onvolution is useful for obtaining inverse Laplace transform of a product of two transforms
and solving ordinary differential equations.
Definition 9.1.1. Let F(t) and G(t) be two functions defined for t > 0. We define
Z t
F(t) ∗ G(t) = F(u)G(t − u)du
0
assuming that the integral on the right hand side exists. F(t)∗G(t) is called the convolution product
of F(t) and G(t).
Proof. Let Z t
φ(t) = F(t) ∗ G(t) = F(u)G(t − u)du.
0
Then
Z ∞ hZ t i
L [φ(t)] = e −pt
F(u)G(t − u)du dt
0 0
Z ∞Z t
= e−pt F(u)G(t − u)dudt
0 0
The double integral is considered within the region enclosed by the lines u = 0 and u = t.
On changing the order of integration, we get
Z ∞ Z ∞
L [φ(t)] = e−pt F(u)G(t − u)dtdu
Z0 ∞ u hZ ∞ i
= −pu
e F(u) e−p(t−u)G(t − u)dt du
Z0 ∞ u
hZ ∞ i
= e−pu F(u) e−pvG(v)dv du on putting t − u = v
Z0 ∞ 0
= e−pu F(u)[g(p)]du
0
Z ∞
= g(p) e−pu F(u)du
0
or φ(t) = L −1 [ f (p).g(p)]
h 1 i
F(t) = L −1 [ f (p)] = L −1 = e−at .
p+a
and
h 1 i
G(t) = L −1 [g(p)] = L −1 = e−bt .
p+b
∴ by convolution theorem,
h 1 i h 1 1 i
L −1 = L −1 .
(p + a)(p + b) (p + a) p + b
= L −1 [ f (p).g(p)]
= F(t) ∗ G(t)
Z t
= F(u)G(t − u)du
0
Z t
= e−au e−b(t−u) du
0
Z t
= e −bt
e−b(t−u) du
0
e−(a−b)u t
= e−bt
−(a − b) 0
1 −bt −(a−b)t
= − e e −1
a−b
1
= (e−at − e−bt ).
b−a
h i
Example 9.2.2. Find L −1 1
p(p2 +4)
by using convolution theorem.
h1i
F(t) = L −1 [ f (p)] = L −1 = 1.
p
and
h 1 i 1
G(t) = L −1 [g(p)] = L −1 = sin 2t.
p2 + 4 2
∴ by convolution theorem,
h 1 i
−1 1
h 1 i
L −1 = L .
p(p2 + 4) p p2 + 4
= L −1 [ f (p).g(p)]
= F(t) ∗ G(t)
Z t
= F(u)G(t − u)du
0
Z t
h 1 i 1
L −1
= 1. sin 2(t − u)du
p(p + 4)
2
0 2
1 t
Z
h 1 i
L −1
= sin 2(t − u)du
p(p2 + 4) 2 0
1 − cos 2(t − u)
=
2 −2
1
= (cos 0 − cos 2t)
4
h 1 i 1
L −1 = (1 − cos 2t).
p(p + 4)
2 4
h1i
F(t) = L −1 [ f (p)] = L −1 = 1.
p
and
h 1 i h 1
G(t) = L −1 [g(p)] = L −1 = L −1
= e−t sin t.
p2 + 2p + 2 (p + 1)2 + 1
∴ by convolution theorem,
h 1 i
−1 1
h 1 i
L −1 = L .
p(p2 + 2p + 2) p p2 + 2p + 2
= L −1 [ f (p).g(p)]
= F(t) ∗ G(t)
= G(t) ∗ F(t)
Z t
= G(u)F(t − u)du
0
Z t
= e−u sin u.1du
0
e−u t
= (− sin u − cos u)
1+1 0
1
= − [e−t (sin t + cos t) − 1.(0 + 1)]
2
h 1 i 1
L −1 = [1 − et (sin t + cos t)].
p(p2 + 2p + 2) 2
h i
Example 9.2.4. Find L −1 1
(p2 +a2 )2
by using convolution theorem.
h 1 i 1
F(t) = L −1 [ f (p)] = L −1 = sin at.
p2 + a2 a
and
h 1 i 1
G(t) = L −1 [g(p)] = L −1 = sin at.
p2 + b2 a
∴ by convolution theorem,
h 1 i h 1 1 i
L −1 = L −1
.
p(p2 + 2p + 2) p+ a2 p2 + b2
= L −1 [ f (p).g(p)]
= F(t) ∗ G(t)
Z t
= F(u)G(t − u)du
0
Z t
1 1
= sin au. sin a(t − u)du
0 a a
Z t
1
= 2 sin au sin(at − au)du
a 0
Z t
1 cos(2au − at) − cos at
= 2 du
a 0 2
1 h sin(2au − at) it
= − cos at.u
2a2 2a 0
1 1h 1 i
= sin at − t cos at + sin at
2a2 2a 2a
h 1 i 1
L −1 2 = [sin at − at cos at].
(p + a2 )2 2a3
h i
Example 9.2.5. Find L −1 1
(p−2)(p+2)2
by using convolution theorem.
∴ by convolution theorem,
h 1 i h 1 1 i
L −1 = L −1
.
(p − 2)(p + 2)2 (p + 2)2 p − 2
= L −1 [ f (p).g(p)]
= F(t) ∗ G(t)
Z t
h 1 i
L −1
= F(u)G(t − u)du
(p − 2)(p + 2)2 0
h p i
F(t) = L −1 [ f (p)] = L −1 = cos at.
p2 + a2
and
h 1 i 1
G(t) = L −1 [g(p)] = L −1 = sin at.
p2 + a2 a
∴ by convolution theorem,
h p i h p 1 i
L −1 = L −1 2 . 2
(p2 +a )
2 2 p + a p + a2
2
= L −1 [ f (p).g(p)]
= F(t) ∗ G(t)
Z t
= F(u)G(t − u)du
0
Z t
= cos au. sin a(t − u)du
0
1 t
Z
= cos au sin(at − au)du
a 0
1 t sin(au + at − au) − sin(au − at + au)
Z
= du
a 0 2
Z t
1
= [sin at − sin(2au − at)]du
2a 0
1h 1 it
=
sin at.u + cos(2au − at)
2a 2a 0
1 h 1 1 i
= t sin at + cos at − cos at
2a 2a 2a
h p i t
L −1
= sin at.
(p2 + a2 )2 2a
p2
h i
Example 9.2.7. Find L −1 (p2 +a2 )(p2 +b2 )
by using convolution theorem.
h p i
F(t) = L −1 [ f (p)] = L −1 = cos at.
p2 + a2
and
h p i
G(t) = L −1 [g(p)] = L −1 = cos bt.
p2 + b2
∴ by convolution theorem,
h p2 i h p p i
L −1 = L −1
.
(p2 + a2 )(p2 + b2 ) p2 + a2 p2 + b2
= L −1 [ f (p).g(p)]
= F(t) ∗ G(t)
Z t
= F(u)G(t − u)du
0
Z t
= cos au. cos b(t − u)du
0
Z t
= cos au cos(bt − bu)du
0
cos(au + bt − bu) + cos(au − bt + bu)
Z t
= du
0 2
1 th
Z i
= cos[(a − b)u + bt] + cos[(a + b)u − bt] du
2 0
1 h sin[(a − b)u + bt] sin[(a + b)u − bt] it
= +
2 a−b a+b 0
1h 1 1 i
= (sin at − sin bt) + (sin at + sin bt)
2 (a − b) a+b
p2
h i
Example 9.2.8. Find L −1 (p2 +4)2
by using convolution theorem.
h p i
F(t) = L −1 [ f (p)] = L −1 = cos 2t.
p2 + 4
and
h p i
G(t) = L −1 [g(p)] = L −1 = cos 2t.
p2 + 4
∴ by convolution theorem,
h p2 i h p p i
L −1 = L −1
.
(p2 + 4)2 p2 + 4 p2 + 4
= L −1 [ f (p).g(p)]
= F(t) ∗ G(t)
Z t
= F(u)G(t − u)du
0
Z t
= cos 2u. cos 2(t − u)du
0
Z t
= cos 2u cos(2t − 2u)du
0
cos(2u + 2t − 2u) + cos(2u − 2t + 2u)
Z t
= du
0 2
1 t
Z
= [cos 2t + cos(4u − 2t)]du
2 0
1h sin(4u − 2t) it
=
cos 2t.u +
2 4 0
1 h 1 i
= t cos 2t + (sin(4t − 2t) − sin(−2t))
2 4
h p2 i 1 sin 2t
L −1 2 = t cos 2t + .
(p + 4)2 2 2
h i
Example 9.2.9. Find L −1 p+1
(p2 +2p+2)2
by using convolution theorem.
Solution.
h p+1 i h p+1 i
L −1 = L −1
(p2 + 2p + 2)2 (p + 1)2 + 1)2
h p+1 i h p i
L −1 2 = e−t
L −1
(p + 2p + 2)2 (p2 + 1)2
h p i
F(t) = L −1 [ f (p)] = L −1 = cos t.
p2 + 1
and
h 1 i
G(t) = L −1 [g(p)] = L −1 = sin t.
p2 + 1
∴ by convolution theorem,
h p i h p 1 i
L −1 = L −1
.
(p2 + 1)2 p2 + 1 p2 + 1
= L −1 [ f (p).g(p)]
= F(t) ∗ G(t)
Z t
= F(u)G(t − u)du
0
Z t
= cos u. sin(t − u)du
0
h p+1 i t
∴ L −1 = e−t . sin t.
(p + 2p + 2)
2 2 2
h i
Example 9.2.10. Find L −1 1
p(p2 +4)2
by using convolution theorem.
h 1 i 1
F(t) = L −1 [ f (p)] = L −1 = sin 2t.
p2 + 4 2
and
h 1 i 1
G(t) = L −1 [g(p)] = L −1 = sin 2t.
p2 + 4 2
∴ by convolution theorem,
h 1 i h 1 1 i
L −1 = L −1
.
(p2 + 4)2 p2 + 4 p2 + 4
= L −1 [ f (p).g(p)]
h1 i
L −1 = F(t) ∗ G(t)
(p2 + 4)2
Z t
h 1 i
L −1
= F(u)G(t − u)du
(p2 + 4)2 0
1 t
Z
= sin 2u. sin 2(t − u)du
4 0
1 t
Z
= sin 2u sin(2t − 2u)du
4 0
Z t
h 1 i sin 2x x cos 2x
L −1
= − dx
p(p2 + 4)2 0 16 8
h cos 2x x 1 it
= − − sin 2x − cos 2x
32 16 32 0
cos 2t t 1 1 1
= − − sin 2t − cos 2t + +
32 16 32 32 32
1 t 1
= − cos 2t − sin 2t +
16 16 16
h 1 i 1
L −1 = (1 − cos 2t − t sin 2t).
p(p2 + 4)2 16
h i
Example 9.2.11. Prove that L −1 8
(p2 +1)3
= (3 − t2 )2 sin t − 3t cos t.
Solution. We have
h 8 i h 1 1 i
L −1 = 8L −1
.
(p2 + 1)3 (p2 + 1)2 p2 + 1
First we find
h 1 i
L −1
(p2 + 1)2
by using convolution theorem.
Let f (p) = 1
p2 +1
and g(p) = 1
p2 +1
. Then
h 1 i
F(t) = L −1 [ f (p)] = L −1 = sin t.
p2 + 1
and
h 1 i
G(t) = L −1 [g(p)] = L −1 = sin t.
p2 + 1
∴ by convolution theorem,
h 1 i h 1 1 i
L −1 = L −1
.
(p2 + 1)2 p2 + 1 p2 + 1
= L −1 [ f (p).g(p)]
= F(t) ∗ G(t)
Z t
= F(u)G(t − u)du
0
Z t
= sin u. sin(t − u)du
0
Z t
cos(2u − t) − cos t
= du
0 2
1 h sin(2u − t) it
= − cos t.u
2 2 0
1h1 i
= (sin(2t − t) − t cos t)
2 2
h 1 i 1
L −1 2 = (sin t − t cos t).
(p + 1)2 2
h 1 i h 1 1 i
L −1 = L −1
.
(p2 + 1)3 (p2 + 1)2 p2 + 1
Z t
1
= [sin u − u cos u] sin(t − u)du
0 2
1 t
Z
= [sin u sin(t − u) − u(cos u sin(t − u))]du
2 0
1 t h cos(2u − t) − cos t i
Z h sin t + sin(2u − t) i
= −u du
2 0 2 2
1 t
Z
h 1 i
L −1
= [cos(2u − t) − cos t − u sin t + u sin(2u − t)]du
(p2 + 1)3 4 0
1h t2 t cos t sin t i
= sin t − t cos t − sin t − +
4 2 2 2
1
= (3 sin t − 3t cos t − t2 sin t)
8
h 8 i
L −1 2 = (3 − t2 ) sin t − 3t cos t.
(p + 1)3
h i
Example 9.2.12. Find (i) L −1 √ 1
by the convolution integral and hence deduce the value
p(p−a)
i
of p √1p+a . (ii) Applying convolution theorem, find L −1 √1 .
p p+1
1
h 1 i t2 1
F(t) = L [ f (p)] = L
−1
√ = 1 = √ √.
−1
p Γ( 2 ) π t
and
h 1 i
G(t) = L −1 [g(p)] = L −1 = eat .
p−a
∴ by convolution theorem,
h 1 i h 1 1 i
L −1 √ = L −1 √ .
p(p − a) p p−a
= L −1 [ f (p).g(p)]
= F(t) ∗ G(t)
Z t
= F(u)G(t − u)du
0
Z t
1
= √ √ ea(t−u) du
0 π u
at Z t −au
e e
= √ √ du
π 0 u
Deduction:
h1 i h 1 i
L −1 √ = L −1 √
p p+a (p + a − a) p + a
h 1 i
= e−at L −1 √ by f irst shi f ting theorem
(p − a) P
eat √
= e−at √ er f ( at)
a
h 1 i 1 √
L −1 √ = √ er f ( at).
p p+a a
h 1 √
L −1 = er f ( t).
p p+1
p
h i
Example 9.2.13. Find L −1 1
(p−2)(p2 +1)
by using convolution theorem.
h 1 i
F(t) = L −1 [ f (p)] = L −1 = e2t .
p−2
and
h 1 i
G(t) = L −1 [g(p)] = L −1 = sin t.
p2 + 1
∴ by convolution theorem,
h 1 i h 1 1 i
L −1 = L −1
.
(p − 2)(p2 + 1) p − 2 p2 + 1
= L −1 [ f (p).g(p)]
= F(t) ∗ G(t)
Z t
= F(u)G(t − u)du
0
Z t
h 1 i
L −1 = e2u sin(t − u)du
(p − 2)(p2 + 1) 0
h 1 i e2u t
L −1 = [2 sin(t − u) − cos(t − u)]
(p − 2)(p2 + 1) 5 0
Z ax
h e i
S ince eax sin(bx + c) = 2 [a sin(bx + c) − b cos(bx + c)]
a + b2
h 1 i e2t e0
L −1 = [2 sin 0 − cos 0] − [2 sin t − cos t]
(p − 2)(p2 + 1) 5 5
h 1 i 1
L −1 = (cos t − 2 sin t − e2t )
(p − 2)(p + 1)
2 5
9.3 Exercises
Answers
1. 13 (et − e−2t ); 2. 1
a2
(cosh at − 1);→
−
3. 1
a3
(sinh at − at); 4. 1
18
(sin 3t − cos 3t + e−3t );
t t
5. 2
sin t; 6. 2a
sinh at;
7. 81 [(3 − t2 ) sin t − 3t cos t]; 8. 1
2a
(sin at + at cos at);
9. t
8a3
(sin at − at cos at); 10. 1
2a
(sin at + sinh at);
t2
11. 1
16
(1 − t sin 2t − cos 2t); 12. 1 − t + 2
− e−t ;
13. 12 (sin t − cos t + e−t ); 14. 1
16
[e2t −)(4t + 1)e−2t ];
15. 31 (cos t − cos 2t).
Lesson 10
FOURIER SERIES
Learning Objectives
Upon completion of this lesson, students will be able to
• find the method for obtaining Fourier series in the interval [c, c + 2π]
10.1 Introduction
F
ourier series is an infinite series representation of a function in terms of sines and cosines of
an angle and its multiples. We know that Taylor’s series expansion is valid only functions
which are continuous and differentiable. But Fourier series is possible for continuous functions,
periodic functions and functions discontinuous in their values and derivatives. Fourier series
is useful to solve ordinary and partial differential equations particularly with periodic functions
appearing as non - homogeneous terms.
Suppose that a given function f (x) defined in [−π, π] or [0, 2π] or in any other interval can be
expressed as a trigonometric series as
a0
f (x) = + a1 cos x + a2 cos 2x + .... + an cos nx + ... + b1 sin x + b2 sin 2x + .... + bn sin nx + ...
2
a0
f (x) = + (a1 cos x + b1 sin x) + (a2 cos 2x + b2 sin 2x) + (a3 cos 3x + b3 sin 3x)
2
+ ..... + (an cos nx + bn sin nx) + ...
∞
a0 X
f (x) = + (an cos nx + bn sin nx)
2 n=1
where the a0 s and b0 s are constants within a desired range of values of the variable. Such series
is known as the Fourier series for f (x) and the constants a0 , an and bn (n = 1, 2, 3, ..) are
called Fourier coefficients of f (x).
A function f (x) is said to be of period of T or to be periodic with period T > 0 if for all real x,
f (x + T ) = f (x)
f (x) = f (x + T ) = f (x + 2T ) = ....
Example.1 Since
sin x = sin(x + 2π) = sin(x + 4π) = ...
the function sin x is periodic with period 2π. There is no positive value T, 0 < T < 2π such
that sin(x + T ) = sin x for all x.
Note. 1 If T is the period of f (x) then nT is also period of f for any integer n.
2. If T is period of f (x) and g(x) then the function h(x) = a f (x) + bg(x) has period T , where
a and b are constants.
3. If f (x) is a periodic function of period T, then f (ax) with a , 0, is a periodic function of
period T
a
.
4. The period of sum of a number of periodic functions is the L.C.M of the periods.
5. A constant function is periodic for any positive T.
Theorem 10.3.1. If f (x) is a periodic function with period 2π and if f (x) can be represented
by trigonometric series in (−∞, ∞)
∞
a0 X
f (x) = + (an cos nx + bn sin nx)
2 n=1
then
1 c+2π
Z
a0 = f (x)dx
π c
1 c+2π
Z
an = f (x) cos nxdx (10.3.1)
π c
1 c+2π
Z
bn = f (x) sin nxdx
π c
Proof. Let f (x) be represented in the interval [c, c + 2π] by the Fourier series
∞ ∞
a0 X X
f (x) = + an cos nx + bn sin nx (10.3.2)
2 n=1 n=1
Let us assume that the series is uniformly convergent in the interval c ≤ x ≤ c + 2π. Then the
Z c+2π ∞
X h sin nx c+2π cos nx c+2π i
a0
f (x)dx = (c + 2π − c) + an + bn −
c 2 n=1
n c n c
X∞
= a0 π + [an .0 − bn .0]
n=1
Z c+2π
f (x)dx = a0 π
c
Z c+2π
1
Hence a0 = f (x)dx.
π 0
To evaluate an :
Multiplying both sides of (16.3.2) by cos mx and then integrating from x = c to x = c + 2π, we
get
c+2π ∞
a0 c+2π
Z Z X Z c+2π
f (x) cos mxdx = cos mxdx + an cos nx cos mxdx
c 2 c n=1 c
X∞ Z c+2π
+ bn sin nx cos mxdx
n=1 c
The first and third integral on the right - hand side are always zero, but second integral is equal to
To evaluate bn : Multiplying both sides of (16.3.2) by sin mx and then integrating from x = c to
x = c + 2π, we get
c+2π ∞
a0 c+2π
Z Z X Z c+2π
f (x) sin mxdx = sin mxdx + an cos nx sin mxdx
c 2 c n=1 c
X∞ Z c+2π
+ bn sin nx sin mxdx
n=1 c
The first two integral on the right - hand side are always zero, but third integral is equal to π when
m = n; otherwise it also vanishes when m , n. Hence
Z c+2π ∞ Z c+2π
1 Xh
f (x) sin nxdx = bn 2 sin2 nxdx
c 2 n=1 c
Z c+2π
bn
= (1 − cos 2nx)dx
2 c
an sin 2nx c+2π
= x−
2 2n c
bn
= (2π)
2
Z c+2π
1
∴ bn = f (x) sin nxdx.
π 0
Corollary 10.3.1. If f (x) is to be expanded as a Fourier series in the interval 0 ≤ x ≤ 2π, put
c = 0 then the Euler’s formulae reduces to
1 2π
Z
a0 = f (x)dx
π 0
1 2π
Z
an = f (x) cos nxdx (10.3.3)
π 0
1 2π
Z
bn = f (x) sin nxdx
π 0
Corollary 10.3.2. If f (x) is to be expanded as a Fourier series in the interval [−π, π], put c = −π
then the Euler’s formulae reduces to
1 π
Z
a0 = f (x)dx
π −π
1 π
Z
an = f (x) cos nxdx (10.3.4)
π −π
Z π
1
bn = f (x) sin nxdx
π −π
In many of the problems, we come across expansions of f (x) in [0, 2π] or [−π, π] and hence
formulae (16.3.3) and (10.3.4) to be remembered carefully.
Note. Because of the periodic nature, Fourier series constructed for one period is valid for all
values.
Dirichlet has formulated certain conditions known as Dirichlet conditions under which certain
functions possess valid Fourier expansions.
A function f (x) has a valid Fourier series expansion of the form
∞
a0 X
+ (an cos nx + bn sin nx)
2 n=1
1.
0, for m , n
Z π
cos mx cos nxdx =
π, for m = n > 0
−π
for m = n = 0
2π,
2.
for m , n and m = n = 0
π
0,
Z
sin mx sin nxdx =
−π
π, for m = n > 0
3. sin nπ = 0, sin 2nπ = 0, for nZ, where Z is the set of all integers.
4. cos nπ = (−1)n , cos 2nπ = 1, nZ.
5. sin n + 12 π = (−1)n , nZ.
6. cos n + 12 π = 0, nZ.
7.
n−1
(−1) 2 ,
if n is odd
nπ
=
sin
2
0,
if n is even
8.
0, if n is odd
nπ
=
cos
2 n
(−1) 2 , if n is even
∞ ∞
a0 X X
f (x) = + an cos nx + bn sin nx
2 n=1 n=1
and Z c+2π
1
an = f (x) cos nxdx; n = 1, 2, 3, ...
π c
Step.4 Substitute the values of a0 , an and bn in f (x) in Step.1 to get the required Fourier series.
Example 10.5.1. Express f (x) = x − π as Fourier series in the interval −π < x < π.
∞ ∞
a0 X X
x−π= + an cos nx + bn sin nx (10.5.1)
2 n=1 n=1
1 π
Z
a0 = f (x)dx
π −π
1 π
Z
= (x − π)dx
π −π
1h π
Z Z π i
= xdx − π dx
π −π −π
Z π
1h i
= 0 − π.2 dx since x is an odd function
π 0
1
= (−2π)(x)π0
π
a0 = −2π
1 π
Z
an = f (x) cos nxdx
π −π
1 π
Z
= (x − π) cos nxdx
π −π
1h π
Z Z π i
= x cos nxdx − π cos nxdx
π −π −π
Z π
1h i
= 0 − π.2 cos nxdx since x cos nx is an odd function
π 0
1 sin nx π
= (−2π)
π n 0
an = −2(sin nπ − sin 0)
an = 0.
1 π
Z
bn = f (x) sin nxdx
π −π
1 π
Z
= (x − π) sin nxdx
π −π
1h π
Z Z π i
= x sin nxdx − π sin nxdx
π −π −π
Z π
1h i
bn = 2 x sin nxdx − π(0) since sin nx is an odd function
π 0
2 h − cos nx − sin nx iπ
bn = x − 1.
π n n2 0
2 π cos nπ
h i −2
= + 0 − (0 + 0) = cos nπ
π n π
−2
bn = (−1)n
π
2
bn = (−1)n+1 , for n = 1, 2, 3, ..
π
Substituting theses values of a0 , an and bn in (19.4.1),we get the required Fourier series of f (x)
in (−π, π) as
∞
X 2
x − π = −π + (−1)n+1 sin nx
n=1
n
1 1 1 i
x − π = −π + 2 sin x − sin 2x + sin 3x − sin 4x + ..... .
2 3 4
Example 10.5.2. Express f (x) = π − x as Fourier series in the interval (0, 2π).
∞ ∞
a0 X X
x−π= + an cos nx + bn sin nx (10.5.2)
2 n=1 n=1
1 2π
Z
a0 = f (x)dx
π 0
1 2π
Z
= (π − x)dx
π 0
1h x2 i2π
= πx −
π 2 0
a0 = 0.
1 2π
Z
an = f (x) cos nxdx
π 0
1 2π
Z
= (π − x) cos nxdx
π 0
1h sin nx − cos nx i2π
= (π − x) − (−1)
π n n2 0
an = 0.
1 2π
Z
bn = f (x) sin nxdx
π 0
1 2π
Z
= (π − x) sin nxdx
π 0
1h − cos nx − sin nx i2π
= (π − x) − (−1)
π n n2 0
1 h cos 2nπ 1 i
= − (−π) + (π) + 0 + 0
π n n
1 h 2π i
=
π n
2
bn =
n
Substituting theses values of a0 , an and bn in (10.5.2),we get the required Fourier series of f (x)
in (−π, π) as
∞
X 2
π−x = sin nx
n=1
n
1 1 1
π − x = 2 sin x + sin 2x + sin 3x + sin 4x + ..... .
2 3 4
Example 10.5.3. Find a Fourier series to represent f (x) = x2 in the interval (0, 2π).
∞ ∞
a0 X X
x2 = + an cos nx + bn sin nx (10.5.3)
2 n=1 n=1
1 2π
Z
a0 = f (x)dx
π 0
1 2π 2
Z
= x dx
π 0
1 h x3 i2π
=
π 3 0
8 2
a0 = π.
3
1 2π 2
Z
an = x cos nxdx
π 0
1 h 2 sin nx − cos nx − sin nx i2π
= x − 2x +2
π n n2 n3 0
1h 1 i
= 0 + 2(2π) 2 + 0 − (0 + 0 + 0)
π n
1 4π
=
π n2
4
an = 2 .
n
1 2π
Z
bn = f (x) sin nxdx
π 0
1 2π 2
Z
= x sin nxdx
π 0
1 h 2 − cos nx − sin nx cos nx i2π
= x − 2x + 2
π n n2 n3 0
1 h 1 1 2 i
= − 4π2 . + 0 + 2 3 − 0 + 0 + 3
π n n n
1 −4π2
=
π n
−4π
bn = .
n
Substituting theses values of a0 , an and bn in (10.5.3),we get the required Fourier series of f (x)
in (−π, π) as
∞ ∞
4 2 X 4 X 4π
x2 = π + 2
cos nx − sin nx
3 n=1
n n=1
n
4 1 1 1 1
x2 = π2 + 4 cos x + 2 cos 2x + 2 cos 3x + ..... − 4π sin x + sin 2x + sin 3x + ...
3 2 3 2 3
Example 10.5.4. Determine the Fourier series expansion of the function f (x) = 1
12
(3x2 −6xπ+2π2 )
in the interval (0, 2π).
Solution.
∞ ∞
1 a0 X X
(3x2 − 6xπ + 2π2 ) = + an cos nx + bn sin nx (10.5.4)
12 2 n=1 n=1
Z 2π
1
a0 = f (x)dx
π 0
1 2π 1
Z
a0 = (3x2 − 6xπ + 2π2 )dx
π 0 12
1 h 3 i2π
= x − 3x2 π + 2π2 x
12π 0
1
a0 = (8π3 − 12π3 + 4π3 )
12π
a0 = 0.
1 2π
Z
an = f (x) cos nxdx
π 0
1 2π 1
Z
= (3x2 − 6xπ + 2π2 ) cos nxdx
π 0 12
1 h 2 sin nx − cos nx − sin nx i2π
= (3x − 6xπ + 2π2 ) − (6x − 6π) + 6
12π n n2 n3 0
1 h 1 1 i
= 0 + 6π 2 + 0 − (0 − 6π 2 )
12π n n
1 12π
=
12π n2
1
an = .
n2
1 2π
Z
bn = f (x) sin nxdx
π 0
1 2π 1
Z
= (3x2 − 6xπ + 2π2 ) sin nxdx
π 0 12
Substituting theses values of a0 , an and bn in (10.5.4), we get the required Fourier series of f (x)
in (−π, π) as
∞
1 X 1
(3x − 6xπ + 2π ) =
2 2
cos nx.
12 n=1
n2
Example 10.5.5. Determine the Fourier series expansion of the function f (x) = e x in the interval
(0, 2π).
Solution.
∞ ∞
a0 X X
e =
x
+ an cos nx + bn sin nx (10.5.5)
2 n=1 n=1
1 2π
Z
a0 = f (x)dx
π 0
1 2π x
Z
= e dx
π 0
1 x 2π
= (e )0
π
1 2π
a0 = (e − 1).
π
1 2π
Z
an = f (x) cos nxdx
π 0
1 2π x
Z
= e cos nxdx
π 0
1 h ex i2π
= (cos nx + n sin nx)
1π 1 + n2 0
1 h e2π 1 i
= (cos 2nπ + 0 − (1 + 0)
π 1 + n2 1 + n2
2π
e −1
an = .
π(1 + n2 )
1 2π
Z
bn = f (x) sin nxdx
π 0
1 2π x
Z
= e sin nxdx
π 0
1 h ex i2π
= (sin nx − n cos nx)
π 1 + n2 0
2π
1 e
h 1 i
= (0 − n) − (0 − n)
π 1 + n2 1 + n2
e2π − 1
bn = (−n) .
π(1 + n2 )
Substituting theses values of a0 , an and bn in (10.5.5), we get the required Fourier series of f (x)
in (−π, π) as
∞ ∞
1 2π X e2π − 1 X e2π − 1
ex = (e − 1) + cos nx + (−n) sin nx
2π n=1
π(1 + n 2)
n=1
π(1 + n2)
2π ∞ ∞
e −1 1h X cos nx X n sin nx i
ex = + − .
π 2 n=1 1 + n2 n=1 1 + n2
Example 10.5.6. Find the Fourier series representing f (x) = x, 0 < x < 2π. Sketch the graph of
f (x) from −4π to 4π.
Solution.
∞ ∞
a0 X X
x= + an cos nx + bn sin nx (10.5.6)
2 n=1 n=1
1 2π
Z
a0 = f (x)dx
π 0
1 2π
Z
= xdx
π 0
1 x2 2π
=
π 2 0
a0 = 2π.
1 2π
Z
an = f (x) cos nxdx
π 0
1 2π
Z
= x cos nxdx
π 0
1 h sin nx − cos nx i2π
= x − 1.
1π n n2 0
1 h 1 1 i
= 0+ 2 − 0+ 2
π n n
an = 0.
1 2π
Z
bn = f (x) sin nxdx
π 0
1 2π
Z
= x sin nxdx
π 0
1 h − cos nx − sin nx i2π
bn = x − 1.
1π n n2 0
1 h 2π i
bn = − + 0 − (0 + 0)
π n
2
bn = −
n
Substituting theses values of a0 , an and bn in (10.5.6), we get the required Fourier series of f (x)
in (−π, π) as
∞
X 1
x = π−2 sin nx
n=1
n
1 1
x = π − 2 sin x + sin 2x + sin 3x + ... .
2 3
Example 10.5.7. Find a Fourier series to represent f (x) = x − x2 in the interval (−π, π). Hence
show that
∞
X (−1)n−1 π2
=
n=1
n2 12
(or)
1 1 1 π2
+ + + ..... = .
12 22 32 12
Solution. Let f (x) = x − x2 be represented by the Fourier series
∞ ∞
a0 X X
x−x =
2
+ an cos nx + bn sin nx (10.5.7)
2 n=1 n=1
1 π
Z
a0 = f (x)dx
π π
1 π
Z
= (x − x2 )dx
π −π
1h π
Z Z 2π
= xdx − x2 dx
π −π
Z π −π
1h
= 0−2 x2 dx
π 0
1 x3 π
= (−2).
π 3 0
3
1 2π
a0 = −
π 3
2π2
a0 = − .
3
1 π
Z
an = f (x) cos nxdx
π −π
Z π
1
= (x − x2 ) cos nxdx
π −π
1h π
Z Z π i
= x cos nxdx − x2 cos nxdx
π −π π
Z π
1h i
= 0−2 x2 cos nx
π 0
2 h 2 sin nx − cos nx − sin nx iπ
= − x − 2x + 2
π n n2 n3 0
2 h cos nπ i
= − 0 + 2(π) 2 + 0 − (0 + 0 + 0)
π n
2 2π(−1)n
= − (n , 0)
π n2
4(−1)n+1
an = .
n2
1 2π
Z
bn = f (x) sin nxdx
π −π
1 2π
Z
= (x − x2 ) sin nxdx
π −π
1h π
Z Z π i
= x sin nxdx − x2 sin nxdx
π −π −π
Z π
1h i
= 2. x sin nxdx − 0
pi 0
2 h − cos nx −sinnx iπ
= x − 1.
pi n n2 0
n
2 h (−1) i
= −π + 0 − (0 + 0)
π n
(−1)n
= −
n
(−1)n+1
bn = .
n
Substituting theses values of a0 , an and bn in (10.5.7), we get the required Fourier series of f (x)
in (−π, π) as
∞ ∞
−π2 X 4(−1)n+1 X 2(−1)n+1
x−x = 2
+ cos nx + sin nx
3 n=1
n2 n=1
n
−π2 1 1 1 1
x − x2 = + 4 cos x − 2 cos 2x + 2 cos 3x − ..... + 2π sin x − sin 2x + sin 3x − ... .
3 2 3 2 3
−π2 1 1 1
0 =
+ 4 1 − 2 + 2 − 2 + ...
3 2 3 4
1 1 1 π2
i.e., 1 − 2 + 2 − 2 + ... =
2 3 4 12
∞
X (−1) n−1
π2
or 2
= .
n=1
n 12
1 1 1 1 π2
− + − + ... = .
12 22 32 42 12
(π−x)2
Solution. Let f (x) = 4
be represented by the Fourier series
∞ ∞
(π − x)2 a0 X X
= + an cos nx + bn sin nx (10.5.8)
4 2 n=1 n=1
1 2π
Z
a0 = f (x)dx
π 0
1 2π (π − x)2
Z
= dx
π 0 4
1 h (π − x)3 i2π
= −
4π 3 0
1
= − [(−π)3 − π3 ]
12π
π2
a0 = .
6
Z 2π
1
an = f (x) cos nxdx
π 0
1 2π (π − x)2
Z
an = cos nxdx
π 0 4
1h sin nx − cos nx − sin nx i2π
= (π − x)2 − 2(π − x)(−1) + 2
4π n n2 n3 0
1h 1 1 i
= 0 + 2(π) 2 + 0 − (0 − 2π. 2 + 0)
4π n n
1 4π
= (n , 0)
4π n2
1
an = , i f n , 0.
n2
1 2π
Z
bn = f (x) sin nxdx
π 0
1 2π (π − x)2
Z
= sin nxdx
π 0 4
1h − cos nx − sin nx cos nx i2π
= (π − x)2 − 2(π − x)(−1) + 2
4π n n2 n3 0
1 h 1 2 1 2 i
= − π2 + 3 − − π2 . + 0 + 3
4π n n n n
1
= (0) = 0.
4π
Substituting theses values of a0 , an and bn in (10.5.8), we get the required Fourier series of f (x)
in (0, 2π) as
∞
(π − x)2 π2 X 1
= + cos nx
4 12 n=1 n2
(π − x)2 π2 1 1
= + cos x + 2 cos 2x + 2 cos 3x + .....
4 12 2 3
Deduction.
Putting x = π in the above series, we have
π2 1 1 1
0 =
− 1 − 2 + 2 − 2 + ...
12 2 3 4
1 1 1 π2
i.e., 1 − 2 + 2 − 2 + ...... = .
2 3 4 12
Example 10.5.9. Obtain the Fourier series expansion of f (x) given that f (x) = (π − x)2 in the
π2
interval 0 < x < 2π. Hence show that 1
12
+ 1
22
+ 1
32
+ 1
42
+ ... = 6
.
h π2 1 1 i
(π − x)2 = 4 + cos x + cos 2x + cos 3x + .....
12 22 32
π4 π2 1 1 1
= + 2 + 2 + 2 + .....
4 12 1 2 3
1 1 1 π2 π2
2
+ 2+ 2 + ..... = −
1 2 3 4 12
1 1 1 π2
2
+ 2+ 2 + ..... = .
1 2 3 6
Example 10.5.10. Find the Fourier series to represent the function f (x) = e−ax in the interval
x = −π to x = π. Deduce from that
π h 1 1 1 i
=2 2 − 2 + 2 − ....
sinh π 2 +1 3 +1 4 +1
Solution.
∞ ∞
a0 X X
e−ax
= + an cos nx + bn sin nx (10.5.9)
2 n=1 n=1
1 π
Z
a0 = f (x)dx
π −π
1 π −ax
Z
= e dx
π −π
1 e−ax π
=
π −a −π
1
= − (e−aπ − a−aπ )
aπ
1
a0 = 2 sinh aπ.
aπ
1 π
Z
an = f (x) cos nxdx
π −π
1 π −ax
Z
= e cos nxdx
π −π
1 h e−ax iπ
an = (−a cos nx + n sin nx)
π a2 + n2 −π
1 h e−aπ eaπ i
an = (−a cos nπ + 0) − (−a cos nπ + 0)
π a2 + n2 a2 + n2
a
= (eaπ − e−aπ ) cos nπ
π(a + n2 )
2
2a(−1)n sinh aπ
an = .
π(a2 + n2 )
1 π
Z
bn = f (x) sin nxdx
π −π
Z π
1
= e−ax sin nxdx
π −π
1 h e−ax iπ
= (−a sin nx − n cos nx)
1π a2 + n2 −π
1 h e−aπ eaπ i
= (0 − n cos nπ) − (0 − n cos nπ)
π a2 + n2 a2 + n2
n
= (eaπ − e−aπ ) cos nπ
π(a2 + n2 )
2n(−1)n sinh aπ
an = .
π(a2 + n2 )
Substituting theses values of a0 , an and bn in (10.5.9),we get the required Fourier series of f (x)
in (−π, π) as
∞ ∞
sinh aπ X 2a(−1)n sinh aπ X 2n(−1)n sinh aπ
e−ax = + cos nx + (−n) sin nx
aπ n=1
π(a 2 + n2 )
n=1
π(a 2 + n2 )
2 sinh π h 1 1 1 1 1 i
1 = − + 2 − 2 + 2 − ........
π 2 2 2 +1 3 +1 4 +1
π 1 1 1
= 2 2 − 2 + 2 − .... .
sinh π 2 +1 3 +1 4 +1
Example 10.5.11. Find the Fourier series representing f (x) = x sin x, 0 < x < 2π.
Solution.
∞ ∞
a0 X X
x sin x = + an cos nx + bn sin nx (10.5.10)
2 n=1 n=1
1 2π
Z
a0 = f (x)dx
π 0
1 2π
Z
= x sin xdx
π 0
1 2π
= x(− cos x) − 1.(− sin x)
π 0
1
= [−2π + 0 − (0 + 0)]
π
a0 = −2.
1 2π
Z
an = f (x) cos nxdx
π 0
1 2π
Z
= x sin x cos nxdx
π 0
1 2π sin(n + 1)x − sin(n − 1)x
Z
= x
π 0 2
Z 2π
1
= x[sin(n + 1)x − sin(n − 1)x]
π 0
1 h cos(n + 1)x cos(n − 1)x sin(n + 1)x sin(n − 1)x i2π
= x − + − 1. − + (n , 1)
2π n+1 n−1 (n + 1)2 (n − 1)2 0
1 h cos 2(n + 1)π cos 2(n − 1)π i
= 2π − + , n,1
2π n+1 n−1
1 1
= − + n,1
n+1 n−1
2
an = 2
, n , 1.
n −1
If n = 1, we have
Z 2π
1
a1 = x sin 2x
2π 0
1 h − cos 2x − sin 2x i2π
= x − 1.
2π 2 4 0
1 1
= (−π) = − .
2π 2
1 2π
Z
bn = f (x) sin nxdx
π 0
1 2π
Z
= x sin x sin nxdx
π 0
1 2π cos(n − 1)x − cos(n + 1)x
Z
= x
π 0 2
Z 2π
1
= x[sin(n + 1)x − sin(n − 1)x]
π 0
1 h sin(n − 1)x sin(n + 1)x cos(n − 1)x cos(n + 1)x i2π
= x − − 1. − + (n , 1)
2π n−1 n+1 (n − 1)2 (n + 1)2 0
1h cos 2(n − 1)π cos 2(n + 1)π 1 1 i
= 0+ − − 0 + − , n,1
2π (n − 1)2 (n + 1)2 (n − 1)2 (n + 1)2
1h 1 1 1 1
= − − + n,1
2π (n − 1) 2 (n + 1)2 (n − 1)2 (n + 1)2
bn = 0, n , 1.
If n = 1, then
Z 2π
1
b1 = x 2sin2 xdx
2π 0
Z 2π
1
= x(1 − cos 2x)dx
2π 0
1h sin 2x x2 cos 2x i2π
= x x− − 1. +
2π 2 2 4 0
2
1 4π 1 1 i
= 2π.2π − − +
2π 2 4 4
b1 = π.
Substituting theses values of a0 , an and bn in (10.5.10),we get the required Fourier series of
f (x) in (−π, π) as
∞
1 X 2
x sin x = −1 − + cos nx + π sin x
2 n=2 n2 − 1
∞
1 X cos nx
x sin x = −1 + π sin x − cos x + 2 2−1
.
2 n=2
n
Example 10.5.12. Obtain the Fourier series expansion of f (x) given that f (x) = kx(π − x) in the
interval (0, 2π), where k is a constant.
∞ ∞
a0 X X
k(πx − x2 ) = + an cos nx + bn sin nx (10.5.11)
2 n=1 n=1
1 2π
Z
a0 = f (x)dx
π 0
1 π
Z
= k(πx − x2 )dx
π 0
k x2 x3 2π
= π −
π 2 3 0
k 8
a0 = 2π3 − π3
π 3
2kπ2
a0 = − .
3
1 2π
Z
an = f (x) cos nxdx
π 0
1 2π
Z
= k(πx − x2 ) cos nxdx
π 0
kh sin nx − cos nx − sin nx i2π
= (πx − x2 ) − (π − 2x) + (−2)
π n n 2 n3 0
kh 1 π i
= − 0 − 3(π) 2 + 0 − 0 + 2 + 0
π n n
k −4π
= − (n , 0)
π n2
4k
an = − 2 n , 0.
n
1 2π
Z
bn = f (x) sin nxdx
π 0
1 2π
Z
bn = k(πx − x2 ) sin nxdx
π 0
Substituting theses values of a0 , an and bn in (10.5.11),we get the required Fourier series of
f (x) in (−π, π) as
∞ ∞
−kπ2 X 1 X 1
k(πx − x ) = 2
− 4k 2
cos nx + 2kπ sin nx
3 n=1
n n=1
n
Example 10.5.13. Find the Fourier series to represent the function f (x) = eax in the interval
0 < x < 2π.
Solution.
∞ ∞
a0 X X
e =
ax
+ an cos nx + bn sin nx (10.5.12)
2 n=1 n=1
1 2π
Z
a0 = f (x)dx
π 0
1 2π ax
Z
= e dx
π 0
1 eax 2π
=
π a 0
1 2aπ
a0 = (e − 1)
aπ
1 2π
Z
an = f (x) cos nxdx
π 0
1 2π ax
Z
= e cos nxdx
π 0
1 h eax i2π
= (a cos nx + n sin nx)
π a2 + n2 0
2aπ
1 e
h e0 i
= (a cos 2nπ + 0) − (a + 0)
π a2 + n2 a2 + n2
a
an = (e2aπ − 1)
π(a2 + n2 )
1 2π
Z
bn = f (x) sin nxdx
π 0
1 2π ax
Z
= e sin nxdx
π 0
1 h eax i2π
= (a sin nx − n cos nx)
1π a2 + n2 0
2aπ 0
1h e e i
= (0 − n) − (0 − n)
π a2 + n2 a2 + n2
n
bn = (1 − e2aπ )
π(a2 + n2 )
Substituting theses values of a0 , an and bn in (10.5.12),we get the required Fourier series of
f (x) in (−π, π) as
∞ ∞
e2aπ − 1 a(e2aπ − 1) X 1 e2aπ−1 X n
e ax
= + cos nx − sin nx
2aπ π n=1
a2 + n2 π n=1 a2 + n2
Example 10.5.14. Find the Fourier series representing f (x) = x cos x, 0 < x < 2π.
Solution.
∞ ∞
a0 X X
x sin x = + an cos nx + bn sin nx (10.5.13)
2 n=1 n=1
1 2π
Z
a0 = f (x)dx
π 0
1 2π
Z
= x cos xdx
π 0
1 2π
= x(sin x) − 1.(− cos x)
π 0
1
= [0 + 1 − (0 + 1)]
π
a0 = 0.
1 2π
Z
an = f (x) cos nxdx
π 0
1 2π
Z
an = x cos x cos nxdx
π 0
If n = 1, we have
1 2π
Z
a1 = x cos2 xdx
π 0
Z 2π
1
= x(1 + cos 2x)dx
2π 0
1h sin 2x x2 cos 2x i2π
= x x+ − 1. −
2π 2 2 4 0
1 4π2 1 1 i
= 2π.2π − + −
2π 2 4 4
1
= (2π2 )
2π
a1 = π.
1 2π
Z
bn = f (x) sin nxdx
π 0
1 2π
Z
= x cos x sin nxdx
π 0
1 2π sin(1 + n)x − sin(1 − n)x
Z
= x
π 0 2
Z 2π
1
= x[sin(1 + n)x − sin(1 − n)x]
2π 0
1 h − cos(1 + n)x cos(1 − n)x sin(1 + n)x sin(1 − n)x i2π
= x + − 1. − + (n , 1)
2π 1+n 1−n (1 + n)2 (1 − n)2 0
1h cos 2(1 + n)π cos 2(1 − n)π i
= − 2π + − (0 + 0) , n , 1
2π 1+n 1−n
1 1
= + n,1
1+n 1−n
2n
bn = , n , 1.
1 − n2
If n = 1, then
Z 2π
1
b1 = x sin 2xdx
2π 0
1 h − cos 2x − sin 2x i2π
= x − 1.
2π 2 4 0
1 h −2π i
= + 0 − (0 + 0)
2π 2
1
b1 = − .
2
Substituting theses values of a0 , an and bn in (10.5.13),we get the required Fourier series of
f (x) in (−π, π) as
∞
1 X n
x cos x = π cos x − sin x + 2 sin nx
2 n=2,3,..
1 − n2
Example 10.5.15. Find a Fourier series to represent f (x) = x2 − x in the interval (−π, π). Hence
show that
1 1 1 π2
+ + + ..... = .
12 22 32 12
Solution. Let f (x) = x2 − x be represented by the Fourier series
∞ ∞
a0 X X
x −x=
2
+ an cos nx + bn sin nx (10.5.14)
2 n=1 n=1
1 π
Z
a0 = f (x)dx
π π
1 π 2
Z
= (x − x)dx
π −π
1h π 2
Z Z 2π
= x dx − xdx
π −π −π
Z π
1h i
= 2 x2 dx − 0
π 0
1 x3 π
= (2).
π 3 0
1 2π3
a0 =
π 3
2π2
a0 = .
3
1 π
Z
an = f (x) cos nxdx
π −π
Z π
1
= (x2 − x) cos nxdx
π −π
1h π 2
Z Z π i
= x cos nxdx − x cos nxdx
π −π π
1 2π
Z
bn = f (x) sin nxdx
π −π
1 2π 2
Z
= (x − x) sin nxdx
π −π
1h π 2
Z Z π i
= x sin nxdx − x sin nxdx
π −π −π
Z π
1h i
= 0 − 2. x sin nxdx
pi 0
2 h − cos nx −sinnx iπ
= − x − 1.
pi n n2 0
n
2 h (−1) i
= − −π + 0 − (0 + 0)
π n
(−1)n
= 2
n
2(−1)n
bn = .
n
Substituting theses values of a0 , an and bn in (10.5.14),we get the required Fourier series of
f (x) in (−π, π) as
∞ ∞
π2 X 4(−1)n X 2(−1)n
x −x =
2
+ cos nx + sin nx
3 n=1 n2 n=1
n
π2 1 1 1
0 =
+ 4 − 1 + 2 − 2 + 2 + ...
3 2 3 4
1 1 1 π2
i.e., − 1 + 2 − 2 + 2 − ... = −
2 3 4 12
1 1 1 π2
1 − 2 + 2 − 2 + .. = .
2 3 4 12
10.6 Exercises
1 1 1 π2
+ + + ... = .
12 22 32 6
Answers.
h i
1. x − π = −π + 2 sin x − 12 sin 2x + 31 sin 3x − 14 sin 4x + ...
π−x
2. 2
= sin x + 12 sin 2x + 13 sin 3x + ...
P (−1)n
3. π2 − x2 = 23 π2 − 4 ∞ n=1 n2 cos nx.
5. f (x) = 32 π2 − 4 ∞ 1
P
n=1 n2 cos nx.
2π−1
h P cos nx P∞ n sin nx i
6. (i) e x − 1 = e π 21 + ∞ n=1 n2 +1 − n=1 n2 +1 − 1;
2π
h i
(ii) e−x = 1−eπ 12 + ∞ cos nx
+ ∞ n sin nx
P P
n=1 n +1
2 n=1 n +1 ;
2
h n P (−1)n n sin nx i
(iii) eax = 2 sinh aπ 1
+ ∞ (−1) cos nx
+ ∞
P
π 2a n=1 a2 +n2 n=1 a2 +n2
;
2aπ 2aπ P cos nx 1−e2aπ P∞ n sin nx i
(iv) eax = e 2−1 + ae π −1 ∞ n=1 a2 +n2 + π n=1 a2 +n2 .
Lesson 11
11.1 Introduction
I
n deriving the Euler’s formulae a0 , an and bn it was assumed that f (x) is continuous.
If a function have finite number of discontinuities then such a function is expressible as a
Fourier series.
For instance, let the function f (x) be defined by
φ(x), if c < x < x0
f (x) =
ψ(x), ifx0 < x < c + 2π
where x0 is the point of discontinuity in (c, c + 2π). In such cases also we obtain the Fourier
series for f (x) in the usual way. The values of a0 , an and bn are given by
Z x0 Z c+2π
1h i
a0 = φ(x)dx + ψ(x)dx
π c x0
1 h x0
Z Z c+2π i
an = φ(x) cos nxdx + ψ(x) cos nxdx
π c x0
Z x0 Z c+2π
1 h i
bn = φ(x) sin nxdx + ψ(x) sin nxdx
π c x0
It may be seen from the graph, that at a point of finite discontinuity x = x0 , there is a finite jump
in the value of the function f (x) at x = x0 . Both the limits on the right and left exists and are
different. At such a point, the Fourier series converges to
1
[ f (x0 − 0) + f (x0 + 0)].
2
∞
a0 X
f (x) = + (an cos nx + bn sin nx)
2 n=1
in [c, c + 2π] then the right hand side series converges to f (x) if x is a point of continuity of
f (x) and converges to
1
[ f (x + 0) + f (x − 0)]
2
if x is a point of discontinuity of f (x).
This result is useful to determine the sum of certain series.
as a Fourier series.
Solution. The Fourier series of f (x) in the interval (0, 2π) is given by
∞ ∞
a0 X X
f (x) = + an cos nx + bn sin nx (11.2.1)
2 n=1 n=1
Where
1 2π
Z
a0 = f (x)dx
π 0
1h π
Z Z 2π
= f (x)dx + f (x)dx
π 0 π
1h π
Z Z 2π
= 1. dx + 0. dx
π 0 π
1 π
= (x)
π 0
1
= (π)
π
a0 = 1.
1 2π
Z
an = f (x) cos nxdx
π 0
1h π
Z Z 2π
= f (x) cos nxdx + f (x) cos nxdx
π 0 π
1h π
Z Z 2π
= 1. cos nxdx + 0. cos nxdx
π 0 π
1 sin nx π
=
π n 0
1
= (0 − 0)
nπ
an = 0.
1 2π
Z
bn = f (x) sin nxdx
π 0
1h π
Z Z 2π
= f (x) sin nxdx + f (x) sin nxdx
π 0 π
1h π
Z Z 2π
= 1. sin nxdx + 0. sin nxdx
π 0 π
1 − cos nx π
=
π n 0
1
= − (cos nπ − 1)
nπ
0, when n is even
bn =
2
, when n is odd
nπ
∞
1 2X1 1 2 1 1
f (x) = + sin nx = + sin x + sin 3x + sin 5x + .....
2 π n=1 n 2 π 3 5
Example 11.2.2. Find the Fourier series to represent the function f (x) given by Expand
if − π < x < 0
−k,
f (x) =
if 0 < x < π
k,
Solution. The Fourier series of f (x) in the interval (0, 2π) is given by
∞ ∞
a0 X X
f (x) = + an cos nx + bn sin nx
2 n=1 n=1
Where
1 π
Z
a0 = f (x)dx
π −π
Z π
1h 0
Z
= f (x)dx + f (x)dx
π −π 0
Z π
1h 0
Z
= (−k).dx + k.dx
π −π 0
k
= [−(x)0−π + (x)π0 ]
π
a0 = 0.
1 π
Z
an = f (x) cos nxdx
π −π
Z π
1h 0
Z
= f (x) cos nxdx + f (x) cos nxdx
π −π 0
Z π
1h 0
Z
= (−k). cos nxdx + k. cos nxdx
π −π 0
k h − sin nx 0 sin nx π i
= +
π n −π n 0
k
= (0 + 0)
π
an = 0.
1 π
Z
bn = f (x) sin nxdx
π −π
Z π
1h 0
Z
= f (x) sin nxdx + f (x) sin nxdx
π −π 0
Z π
1h 0
Z
= (−k). sin nxdx + k. sin nxdx
π −π 0
k h cos nx 0 − cos nx π i
= +
π n −π n 0
n n
k 1 − (−1)
h (−1) − 1 i
= −
π n n
2k
= (1 − (−1)n )
nπ
0, when n is even
bn =
4k
,
nπ when n is odd
∞
X 4k 4k 1 1
f (x) = sin nx = sin x + sin 3x + sin 5x + ..... .
n=1
nπ π 3 5
4k ππ 1 3π 1 5π
f sin =
+ sin + sin + .....
2 π 2 3 2 5 2
4k 1 1 1
k = 1 − + − + ....
π 3 5 7
1 1 1 π
i.e., 1 − + − + .... = .
3 5 7 4
Example 11.2.3. Find the Fourier series to represent the function f (x) given by Expand
0, if − π < x < 0
f (x) =
πx
, if0 < x < π
4
Solution. The Fourier series of f (x) in the interval (0, 2π) is given by
∞ ∞
a0 X X
f (x) = + an cos nx + bn sin nx
2 n=1 n=1
Where
1 π
Z
a0 = f (x)dx
π −π
Z π
1h 0
Z
= f (x)dx + f (x)dx
π −π 0
Z π
1h 0 πx
Z
= 0.dx + .dx
π −π 0 4
π x2 π
=
4π 2 0
1 π2
= .
4 2
π2
a0 = .
8
1 π
Z
an = f (x) cos nxdx
π −π
Z π
1h 0
Z
= f (x) cos nxdx + f (x) cos nxdx
π −π 0
Z π
1h 0 πx
Z
= 0. cos nxdx + cos nxdx
π −π 0 4
1 π h sin nx − cos nx iπ
= . x − 1.
π 4 n n2 0
1 h cos nπ 1 i
= 0+ − 0+ 2
4 n2 n
1
= [(−1)n − 1]
4n2
2n2 ,
−1
when n is odd
an =
0,
when n is even
1 π
Z
bn = f (x) sin nxdx
π −π
Z π
1h 0
Z
= f (x) sin nxdx + f (x) sin nxdx
π −π 0
Z π
1h 0 πx
Z
= 0. sin nxdx + sin nxdx
π −π 0 4
1 π h − cos nx − sin nx iπ
bn = . x − 1.
π 4 n n2 0
1 h −π cos nπ i
bn = + 0 − (0 + 0)
4 n
π(−1) n+1
bn =
4n
∞ ∞
π2 X (−1)n − 1 X (−1)n π
f (x) = + cos nx − sin nx
16 n=1 4n2 n=1
4n
π2 1 1 1 π 1 1
= − cos x + 2 cos 3x + 2 cos 5x + ... + sin x − sin 2x + sin 3x + ..... .
16 2 3 5 4 2 3
π2 1 1 1
f (0) = − 1+ + + ...
16 2 32 52
π2 1 1 1
0 = − 1+ + + ...
16 2 32 52
1 1 1 π2
i.e., 1 + 2 + 2 + 2 + .... = .
3 5 7 8
Example 11.2.4. Find the Fourier series of the periodic function defined as
−π, if − π < x < 0
f (x) =
if 0 < x < π
x,
Solution. The Fourier series of f (x) in the interval (0, 2π) is given by
∞ ∞
a0 X X
f (x) = + an cos nx + bn sin nx
2 n=1 n=1
Where
1 π
Z
a0 = f (x)dx
π −π
Z π
1h 0
Z
= f (x)dx + f (x)dx
π −π 0
Z π
1h 0
Z
a0 = (−π)dx + xdx
π −π 0
1h x2 π i
a0 = (−π)(x)0−π +
π 2 0
1h π i
2
= − π2 +
π 2
1 π 2
a0 = −
π 2
π
a0 = − .
2
1 π
Z
an = f (x) cos nxdx
π −π
Z π
1h 0
Z
= f (x) cos nxdx + f (x) cos nxdx
π −π 0
Z π
1h 0
Z
= (−π) cos nxdx + x cos nxdx
π −π 0
1h sin nx 0 sin nx cos nx π i
= (−π) + 1. x. +
π n −π n n2 0
1 1 1
= 0 + 2 cos nπ − 2
π n n
1
= 2 (cos nπ − 1)
nπ
1
= [(−1)n − 1].
πn 2
n2 π , when n is odd
−2
an =
0,
when n is even
1 π
Z
bn = f (x) sin nxdx
π −π
Z π
1h 0
Z
= f (x) sin nxdx + f (x) sin nxdx
π −π 0
Z π
1h 0
Z
= (−π) sin nxdx + x sin nxdx
π −π 0
1h − cos nx 0 cos nx sin nx π i
= −π + −x +
π n −π n n2 0
1 π
h π i
= (1 − cos nπ) − cos nπ
π n n
1
= (1 − 2 cos nπ)
n
∞ ∞
π 2 X 1 X 1
f (x) = − ((−1)n
− 1) cos nx + (1 − 2 cos nπ) sin nx
4 π n=1 n2 π n=1
n
π 2 1 1 1 1
= − − cos x + 2 cos 3x + 2 cos 5x + ... + 3 sin x − sin 2x + 3 sin 3x − ..... .
4 π 3 5 2 3
π 2 1 1
f (0) = − 1 + 2 + 2 + ... (11.2.2)
4 π 3 5
f (0 − 0) = −π f (0 + 0) = 0.
1 pi
∴ f (0) = [ f (0 − 0) + f (0 + 0)] = − .
2 2
Now (11.2.2) becomes
π π 2 1 1
= − − 1 + 2 + 2 + ...
−
2 4 π 3 5
1 1 1 π 2
i.e., 1 + 2 + 2 + 2 + .... = .
3 5 7 8
Example 11.2.5. Find the Fourier series to represent the function f (x) given by
if − π < x < 0
0,
f (x) =
x2 , if 0 < x < π
Solution. The Fourier series of f (x) in the interval (0, 2π) is given by
∞ ∞
a0 X X
f (x) = + an cos nx + bn sin nx
2 n=1 n=1
Where
1 π
Z
a0 = f (x)dx
π −π
Z π
1h 0
Z
= 0.dx + x2 dx
π −π 0
1 x3 π
a0 =
π 3 0
1 h π3 i
a0 =
π 3
π2
a0 = .
3
1 π
Z
an = f (x) cos nxdx
π −π
Z π
1h 0
Z
= f (x) cos nxdx + f (x) cos nxdx
π −π 0
Z π
1h 0
Z
= 0. cos nxdx + x2 cos nxdx
π −π 0
1 h 2 sin nx − cos nx − sin nx iπ
= x. − 2x + 2
π n n2 n3 0
1 2π
= cos nπ
π n2
2
an = (−1)n n , 0
n2
1 π
Z
bn = f (x) sin nxdx
π −π
Z π
1h 0
Z
= f (x) sin nxdx + f (x) sin nxdx
π −π 0
Z π
1h 0
Z
= 0. sin nxdx + x2 sin nxdx
π −π 0
1 h 2 − cos nx − sin nx cos nx iπ
= x − 2x + 2
π n n2 n3 0
2
1 −π 2
= cos nπ + 3 (cos nπ − 1)
π n n
−π 2
bn = (−1)n + 3 [(−1)n − 1] n , 0
n πn
∞ ∞ h
π2 X (−1)n X π 2
f (x) = +2 cos nx + (−1)n+1 + 3 [(−1)n − 1] sin nx
6 n=1
n 2
n=1
n πn
Example 11.2.6. Find the Fourier series to represent the function f (x) given by
if − π < x < 0
0,
f (x) =
if 0 < x < π
sin x,
Solution.
∞ ∞
a0 X X
f (x) = + an cos nx + bn sin nx (11.2.3)
2 n=1 n=1
1 π
Z
a0 = f (x)dx
π −π
Z π
1h 0
Z i
= 0.dx + sin xdx
π −π 0
1
= (− cos x)π0
π
1
= [1 + 1]
π
2
a0 = .
π
1 π
Z
an = f (x) cos nxdx
π −π
Z π
1 0
Z
= 0. cos nxdx + sin x cos nxdx
π −π 0
1 π sin(n + 1)x − sin(n − 1)x
Z
=
π 0 2
Z π
1
= [sin(n + 1)x + sin(1 − n)x]
2π 0
1 h cos(n + 1)x cos(n − 1)x iπ
= − + (n , 1)
2π n+1 (n − 1) 0
1 h cos(n + 1)π cos(n − 1)π 1 1 i
= − + + − , n,1
2π n+1 n−1 n+1 n−1
1 h −(−1)n+1 (−1)n−1 1 1 i
= + + − n,1
2π n + 1 n−1 n+1 n−1
1h 1 1
an = (1 − (−1)n+1 ) + ((−1)n−1 − 1), n , 1
2π n + 1 n−1
∴ an = 0, when n is odd
1 −4
and an =
2π n2 − 1
−2
an = when n is even and n , 1.
π(n2 − 1)
If n = 1, we have
1 π
Z
a1 = sin x cos xdx
π 0
Z π
1
= sin 2xdx
2π 0
1 cos 2x π
= −
2π 2 0
−1
= (cos 2π − cos 0)
4π
a1 = 0.
1 π
Z
bn = f (x) sin xdx
π 0
1 π
Z
= sin x sin nxdx
π 0
Z π
1 cos(n − 1)x − cos(n + 1)x
=
π 0 2
1 h sin(n − 1)x sin(n + 1)x iπ
= − (n , 1)
2π n−1 n+1 0
1
= (0), n , 1
2π
bn = 0, n , 1.
If n = 1, then
1 π 2
Z
b1 = sin xdx
π 0
Z π
1
= (1 − cos 2x)dx
2π 0
1 sin 2x π
= x−
2π 2 0
1 h i
= π − 0 − (0 − 0)
2π
1
b1 = .
2
Substituting theses values of a0 , an and bn in (11.2.3), we get the required Fourier series of f (x)
∞
1 2 X cos 2nx 1
f (x) = − + + sin x
π π n=1 (2n) − 1 2
2
π
Deduction. Putting x = 2
in the above series, we get
π ∞ π
1 2 X cos nπ 1
f = − + sin
2 π π n=1 (2n)2 − 1 2 2
π ∞
1 2X (−1)n 1
sin = − +
2 π π n=1 (2n + 1)(2n − 1 2
∞
1 1 2X (−1)n+1
1− − =
2 π π n=1 (2n + 1)(2n − 1)
∞
π−2 π X (−1)n+1
. =
2π 2 n=1
(2n + 1)(2n − 1)
1 1 1 π−2
− + − ... = .
1.3 3.5 5.7 4
π2
Show that f (x) = 8 cos x
π2 12
+ cos 3x
32
+ cos 5x
52
+ .... . Deduce that 1
12
+ 1
32
+ 1
52
+ ... = 8
.
Solution. The Fourier series of f (x) in the interval (0, 2π) is given by
∞ ∞
a0 X X
f (x) = + an cos nx + bn sin nx
2 n=1 n=1
Where
1 π
Z
a0 = f (x)dx
π −π
Z π
1h 0
Z
2x 2x
= 1+ dx + 1 − dx
π −π π 0 π
2 0 2 π i
1 h x x
= x+ + x+
π π −π π 0
1
= [0 + 0]
π
a0 = 0.
1 π
Z
an = f (x) cos nxdx
π −π
Z π
1h 0
Z
= f (x) cos nxdx + f (x) cos nxdx
π −π 0
Z π
1h 0
Z
2x 2x
= 1+ cos nxdx + 1− cos nxdx
π −π π 0 π
1 h sin nx 0 2 sin nx − cos nx 0 sin nx π 2 h sin nx − cos nx iπ i
= + x − 1. + − x − 1
π n −π π n n2 −π n 0 π n n2 0
1 h 2 1 cos nπ
2 cos nπ 1
i
= 0+ − + 0− − 2
π π n2 n2 π n2 n
2 h 1 cos nπ cos nπ 1 i
= − − + 2
π2 n2 n2 n2 n
2 h 2 2 cos nπ i
= −
π2 n2 n2
4
= (1 − (−1)n )
n2 π2
0, when n is even
∴ an =
n28π2 , when n is odd
1 π
Z
bn = f (x) sin nxdx
π −π
Z π
1h 0
Z
= f (x) sin nxdx + f (x) sin nxdx
π −π 0
Z π
1h 0
Z
2x 2x
= 1+ sin nxdx + 1− sin nxdx
π −π π 0 π
1 h − cos nx 0 2 − cos nx − sin nx 0
= + x − 1.
π n −π π n n2 −π
− cos nx π 2 h − cos nx − sin nx π i i
+ − x − 1.
n 0 π n n2 0
1 −1 cos nπ 2 −π cos nπ
h i h − cos nπ 1 2 −π cos nπ i
= + + + + −
π n n π n n n π n
= 0.
8h1 1 1 i
f (0) =+ + + ...
π2 12 32 52
8h1 1 1 i
1 = 2 2 + 2 + 2 + ...
π 1 3 5
1 1 1 π2
2
+ 2 + 2 + ... = .
1 3 5 8
Example 11.2.8. Find the Fourier series in [−π, π] for the function
2 (π + x), if − π ≤ x < 0
−1
f (x) =
12 (π − x), if 0 < x ≤ π.
Solution. The Fourier series of f (x) in the interval (0, 2π) is given by
∞ ∞
a0 X X
f (x) = + an cos nx + bn sin nx
2 n=1 n=1
Where
1 π
Z
a0 = f (x)dx
π −π
Z π
1 h 0 −1
Z
1
= (π + x)dx + (π − x)dx
π −π 2 0 2
1h x2 0 x2 π i
= − πx + + πx −
2π 2 −π 2 0
1 h π 2
π2i
= − π2 + + π2 −
2π 2 2
a0 = 0.
1 π
Z
an = f (x) cos nxdx
π −π
Z π
1h 0
Z i
= f (x) cos nxdx + f (x) cos nxdx
π −π 0
Z π
1h 0 1
Z
1 i
= − (π + x) cos nxdx + (π − x) cos nxdx
π −π 2 0 2
Z 0 Z 0 Z π Z π
−1 h i
= π cos nxdx + x cos nxdx − π cos nxdx + x cos nxdx
2π −π −π 0 0
−1 h sin nx 0 sin nx cos nx 0 sin nx π sin nx cos nx π i
an = π + x + −π + x +
2π n −π n n2 −π n 0 n n2 0
−1 h 1 cos nπ cos nπ 1 i
an = − + − 2
2π n2 n2 n2 n
= 0.
1 π
Z
bn = f (x) sin nxdx
π −π
Z π
1h 0
Z i
= f (x) sin nxdx + f (x) sin nxdx
π −π 0
Z π
1h 0 1
Z
1 i
= − (π + x) sin nxdx + (π − x) sin nxdx
π −π 2 0 2
Z 0 Z 0 Z π Z π
−1 h i
= π sin nxdx + x sin nxdx − π sin nxdx + x sin nxdx
2π −π −π 0 0
−1 h − cos nx 0 − cos nx sin nx 0 cos nx π − cos nx sin nx π i
= π + x + +π + x +
2π n −π n n2 −π n 0 n n2 0
−1 h −π π cos nπ π −π cos nπ i
= (1 − cos nπ) + + (cos nπ − 1) +
2π n n n n
1
= − (cos nπ − 1)
π
1
= − ((−1)n − 1).
n
0, when n is even
bn =
n2 ,
if n is odd.
∞
X 2
f (x) = sin nx
n=1,3,5...
n
h sin 3x sin 5x i
f (x) = 2 sin x + + + ... .
3 5
Example 11.2.9. Find the Fourier series for the following function
− cos x, if − π ≤ x < 0
f (x) =
if 0 < x ≤ π.
cos x,
Solution. The Fourier series of f (x) in the interval (0, 2π) is given by
∞ ∞
a0 X X
f (x) = + an cos nx + bn sin nx
2 n=1 n=1
Where
1 π
Z
a0 = f (x)dx
π −π
Z π
1h 0
Z
= − cos xdx + cos xdx
π −π 0
1 h 0 π i
= − sin x + sin x
π −π 0
1h i
= 0+0
π
a0 = 0.
1 π
Z
an = f (x) cos nxdx
π −π
Z π
1h 0
Z i
= f (x) cos nxdx + f (x) cos nxdx
π −π 0
Z 0 Z π
1 h i
= − cos x cos nxdx + cos x cos nxdx
π −π 0
Z 0 Z π
1 h i
= − cos x cos nxdx + cos x cos nxdx
π −π 0
Z 0 Z π
1h i
= − cos(−u) cos(−nu)(−du) + cos x cos nxdx ,
π π 0
an = 0.
1 π
Z
bn = f (x) sin nxdx
π −π
Z π
1h 0
Z i
= − cos x sin nxdx + cos x sin nxdx
π −π 0
Z 0 Z π
1h i
= − cos(−u) sin(−nu)(−du) + cos x sin nxdx
π π 0
When n = 1, we have
1 π
Z
b1 = f (x) sin xdx
π −π
Z π
1h 0
Z i
= − cos x sin xdx + cos x sin xdx
π −π 0
Z 0 Z π
1 h i
= − 2 sin x cos xdx + 2 sin x cos xdx
2π −π 0
Z 0 Z π
1h i
= − sin 2xdx + sin 2xdx
2π −π 0
1 h − cos 2x 0 − cos 2x π i
= − +
2π 2 −π 2 0
1 1h 1 i
b1 = (1 − 1) + (1 − 1) = 0.
2π 2 2
4h 2 4 6 i
f (x) = sin 2x + sin 4x + sin 6x + ... .
π 1.3 3.5 5.7
if − π ≤ x < 0
x,
f (x) =
if0 < x < π2
0,
x − π , if π < x ≤ π.
2 2
Solution. The Fourier series of f (x) in the interval (0, 2π) is given by
∞ ∞
a0 X X
f (x) = + an cos nx + bn sin nx
2 n=1 n=1
Where
1 π
Z
a0 = f (x)dx
π −π
Z π2 Z π
1h 0 π i
Z
= xdx + 0. dx + x − dx
π −π 0 π
2
2
2
x − π2 π i
1 h x2 0
= +
π 2 −π 2 π
2
1 h π2 1 π2 i π2 i
= 0− + − 0 + π2 −
π 2 2 4 2
1 h π 2
π 2i
a0 = − +
π 2 8
3π
a0 = − .
8
1 π
Z
an = f (x) cos nxdx
π −π
Z π2 Z π
1h 0 π
Z i
= x cos nxdx + 0. cos nxdx + x − cos nx
π −π 0 π
2
2
1 sin nx
h − cos nx 0
π sin nx
− cos nx π i
= x − 1. + x − − 1
π n n2 −π 2 n n2 π
2
nπ
1 h 1 cos nπ cos nπ cos 2 i
an = − + −
π n2 n2 n2 n2
1 h nπ i
an = 2 1 − cos .
nπ 2
1 π
Z
bn = f (x) cos nxdx
π −π
Z π2 Z π
1h 0 π
Z i
= x sin nxdx + 0. sin nxdx + x − sin nx
π −π 0 π
2
2
1 h − cos nx − sin nx 0 π − cos nx − sin nx π i
= x − 1. + x − − 1
π n n2 −π 2 n n2 π
2
∞ ∞
3π X 1h nπ i 1 X 1 h 3π 1 nπ i
f (x) = − + 1 − cos cos nx − (−1) n
+ sin sin nx
16 π n=1 n2 2 π n=1 π 2n n2 2
11.3 Exercises
1.If
if − π < x ≤ 0
0,
f (x) =
if 0 ≤ x ≤ π.
x,
prove that
2. If
π
2 + x, if − π < x < 0
f (x) =
π2 − x,
if 0 < x < π.
show that
4 h cos x cos 3x cos 5x i
f (x) = + + + ... .
π 12 32 52
3. Find the Fourier series of the following function:
−1 + x, if − π < x < 0
f (x) =
1 + x, if 0 < x < π.
Hint :
x − x = 0, when − π ≤ x ≤ 0
f (x) =
x + x = 2x, when 0 ≤ x ≤ π.
7. If
when 0 < x < π
x(π − x),
f (x) =
when 0 < x < 2π.
−π(π − x),
Prove that
8 1 1
f (x) = sin x + 2 sin 3x + 2 sin 5x + ... .
π 3 5
Answers.
3. f (x) = π2 (π + 2) sin x − 22 sin 2x + 2(π+2)
3π
sin 3x − 42 sin 4x + 2(π+2)
5π
sin 5x + ..
4. f (x) = π2 − π4 cos x + cos323x + cos525x + ...
Lesson 12
FOURIER SERIES OF
EVEN AND ODD FUNCTIONS
Learning Objectives
Upon completion of this lesson, students will be able to
12.1 Introduction
I
n this Lesson, we study the Fourier Series of even and odd functions.
Definition 12.2.1. A function f (x) is said to be even (or symmetric) function if f (−x) = f (x)
and odd (or skew - symmetric) function if f (−x) = − f (x).
For example, x2 , x4 + x+ 1, e x + e−x , cos x, sec x are all even functions of x, while x, x3 ,
x5 + 2x3 + 3, sin x, csc x, tan x are all odd functions.
Graphically an even function is symmetrical about y− axis (which is a mirror for the reflection of
1 1
f (x) = [ f (x) + f (−x)] + [ f (x) − f (−x)].
2 2
Note. It is desirable to consider the even or odd nature of a function f (x) when we are dealing
with domain of definition in the form (−l, l), (−π, π) etc.
We know that a function f (x) defined in (−π, π) can be represented by the Fourier series
∞ ∞
a0 X X
f (x) = + an cos nx + bn sin nx,
2 n=1 n=1
where
1 π
Z
a0 = f (x)dx
π −π
1 π
Z
an = f (x) cos nxdx
π −π
1 π
Z
bn = f (x) sin nxdx
π −π
Z π Z π
1 2
a0 = f (x)dx = f (x)dx.
π −π π 0
Since cos nx is an even function, f (x) cos nx is also an even function. Hence
Z π Z π
1 2
an = f (x) cos nxdx = f (x) cos nxdx.
π −π π 0
Hence if f (x) is defined in (−π, π) and f (x) is an even function, f (x) can be expanded as Fourier
series contains only cosine terms of the form
∞
a0 X
f (x) = + an cos nx,
2 n=1
where
2 π
Z
a0 = f (x)dx
π 0
Z π
2
an = f (x) cos nxdx
π 0
Hence if f (x) is defined in (−π, π) and f (x) is an odd function, f (x) can be expanded as Fourier
series contains only sine terms of the form
∞
X
f (x) = an sin nx,
n=1
where
Z π
2
an = f (x) sin nxdx.
π 0
Note. The function which is neither even nor odd functions contains both sine and cosine terms.
π
Example 12.4.1. Find Fourier series of f (x) = x in −π < x < π. Deduce that 4
= 1− 13 + 51 − 17 +...
Solution. Since f (−x) = −x = − f (x). Therefore f (x) is an odd function in (−π, π).
∞
X
∴x= bn sin nx,
n=1
where
2 π
Z
bn = x sin nxdx
π 0
2 h − cos nx − sin nx iπ
= x −
π n n2 0
2
= − π cos nπ
nπ
2
bn = (−1)n+1 .
n
∞
X (−1)n+1 sin 2x sin 3x
∴x=2 sin nx = 2 sin x − + − ... (12.4.1)
n=1
n 2 3
Deduction. Since the function f (x) = x is continuous, bounded and monotonic on the interval
−π < x < π, Dirichlet’s conditions apply. Therefore the series will be converges to f (x) in the
interval −π < x < π.
π 1 1
Hence (12.4.1) ⇒ = 2 1 − + − ..
2 3 5
π 1 1
= 1 − + − ....
4 3 5
Example 12.4.2. Find Fourier series of f (x) = x2 in −π < x < π. Hence deduce that
π2
(i) 1 − 1
22
+ 1
32
− 1
42
+ .... = 12
.
π2
(ii) 1 + 1
22
+ 1
32
+ 1
42
+ .... = 6
.
π2
(iii) 1 + 1
32
+ 1
32
+ 1
52
+ .... = 8
.
Solution. Since f (−x) = (−x)2 = x2 = f (x). Therefore f (x) is an even function in (−π, π).
∞
a0 X
∴ x2 = + an cos nx (12.4.2)
2 n=1
where
2 π 2
Z
a0 = x dx
π 0
2
a0 = π2 .
3
2 π 2
Z
an = x cos nxdx
π 0
2 h 2 sin nx − cos nx − sin nx iπ
= x − 2x + 2
π n n2 n3 0
2h cos nx i
= 0 + 2π 2 + 2.0
π n
4 cos nπ
=
n2
4
an = 2
(−1)n .
n
∞
π2 X 4
x =
2
+ (−1)n cos nx
3 n=1 n2
∞
π2 X (−1)n+1
= −4 2
cos nx
3 n=1
n
π2 cos 2x cos 3x
∴ x2 = − 4 cos x − + − ... (12.4.3)
3 22 32
Deduction.
(i) Putting x = 0 in (12.4.3), we get
π2 1 1 1
0 = − 4 1 − 2 + 2 − 2 + ...
3 2 3 4
1 1 1 π2
1 − 2 + 2 − 2 + ... = . (12.4.4)
2 3 4 12
(ii) Putting x = π in (12.4.3), we get
π2 cos 2π cos 3π
π2 = − 4 cos π − 2
+ 2
− ...
3 2 3
π 2 1 1 1
π2 = − 4 − 1 − 2 − 2 − 2 − ...
3 2 3 4
π2 1 1 1
π2 = + 4 1 + 2 + 2 + 2 + ...
3 2 3 4
1 1 1 π2
1+ + + + ... = . (12.4.5)
22 32 42 6
(iii) Adding (12.4.4) and (12.4.5) and dividing it by 2, we get the required result as
1 1 1 π2
1+ + + + ... = .
32 52 72 8
Example 12.4.3. Expand the function f (x) = x3 as a Fourier series of in −π < x < π.
Solution. Since f (−x) = (−x)3 = −x3 = − f (x). Therefore f (x) is an odd function in (−π, π).
∞
X
∴x =
3
bn sin nx,
n=1
where
2 π 3
Z
bn = x sin nxdx
π 0
2 h 3 − cos nx − sin nx cos nx sin nx iπ
= x − 3x2 + 6x − 6
π n n2 n3 n4 0
2h π 3
6π i
= − cos nπ + 3 cos nπ
π n n
h π2 6 i
bn = 2(−1)n+1 − .
n n3
∞ h π2
X 6i
∴x =2
3
(−1)n+1 − sin nx.
n=1
n n3
Example 12.4.4. Find Fourier series of f (x) = |x| in −π < x < π. Hence deduce that
π2
1+ 1
32
+ 1
52
+ 1
72
+ .... = 8
.
Solution. Since f (−x) = | − x| = |x| = f (x). Therefore f (x) is an even function in (−π, π).
∞
a0 X
∴ |x| = + an cos nx
2 n=1
where
2 π
Z
a0 = |x|dx
π 0
2 π
Z
a0 = xdx
π 0
2 x2 π
a0 =
π 2 0
a0 = π.
2 π
Z
an = |x| cos nxdx
π 0
2 π
Z
= x cos nxdx
π 0
2 h sin nx − cos nx iπ
= x − 1.
π n n2 0
2 h cos nπ 1 i
= 0+ − 0+ 2
π n2 n
2
an = [(−1)n − 1].
n2 π
0, when n is even
an =
− n42 π , when n is odd.
∞
π 4 X 1
|x| = − cos nx.
2 π n=1,3,5,.. n2
π 4 1 1 1
0= − + + + ...
2 π 12 32 52
1 1 1 π2
+ + + ... = .
12 32 52 8
Solution. As sin ax is an odd function, its Fourier series expansion will consist of sine terms
only.
∞
X
∴ sin ax = bn sin nx,
n=1
where
2 π
Z
bn = sin ax sin nxdx
π 0
1 π
Z
bn = [cos(a − n)x − cos(a + n)x]dx
π 0
∞
2 sin aπ X (−1)n n
∴ sin ax = sin nx
π n=1
a 2 − n2
∞
2 sin aπ X (−1)n+1 n
= sin nx
π n=1
a2 − n2
2 sin aπ h sin ax 2 sin 2x 3 sin 3x i
sin ax = − + + .....
π 12 − a2 22 − a2 32 − a2
Example 12.4.6. Expand the function f (x) = x sin x as a Fourier series in the interval −π < x <
π. Hence deduce that 1
1.3
− 1
1.5
+ 1
5.7
− .... = 41 (π − 2).
∞
a0 X
x sin x = + an cos nx
2 n=1
2 π
Z
a0 = f (x)dx
π 0
Z π
2
= x sin xdx
π 0
2 π
= x(− cos x) − 1.(− sin x)
π 0
2
= [−π(−1) + 0 − (0 + 0)]
π
a0 = 2.
2 π
Z
an = f (x) cos nxdx
π 0
2 π
Z
an = x sin x cos nxdx
π 0
If n = 1, we have
1 π
Z
a1 = x sin 2x
π 0
1 h − cos 2x − sin 2x iπ
= x − 1.
π 2 4 0
1
= (−π)
2π
1
a1 = − .
2
X 2(−1)n+1 ∞
1
∴ x sin x = 1 − cos x + 2−1
cos nx
2 n=2
n
∞
1 X 2(−1)n+1
x sin x = 1 − cos x + cos nx
2 n=2
(n − 1)(n + 1)
1 −2 2 2
x sin x = 1 − cos x + cos 2x + cos 3x − cos 4x + ... .
2 1.3 2.4 3.5
π
Deduction. Putting x = 2
in the above series, we get
π π 2 2 −2
sin = 1−0− (−1) + (0) + (1) + ....
2 2 1.3 2.4 3.5
π 2 2 2 2
= 1+ − + − + ....
2 1.3 3.5 5.7 7.9
π 2 2 2 2
−1 = − + − + ....
2 1.3 3.5 5.7 7.9
1 1 1 1 1
− + − + .... = (π − 2).
1.3 3.5 5.7 7.9 4
√
f (−x) = 1 − cos(−x) = 1 − cos x = f (x).
p
∞
√ a0 X
1 − cos x = + cos nx
2 n=1
2 π
Z
a0 = f (x)dx
π 0
2 π√
Z
= 1 − cos xdx
π 0
√ Z
2 2 π x θ 1 − cos θ
= sin dx, sin2 =
π 0 2 2 2
√
x
2 2 cos 2 π
= −
π 1
2
0
√
4 2 π
= − cos( ) − cos 0
√π 2
4 2
a0 = .
π
2 π
Z
an = f (x) cos nxdx
π 0
2 π√
Z
= 1 − cos x cos nxdx
π 0
√ Z
2 2 π x
= sin cos nxdx
π 0 2
√ Z π
2 h 1 1 i
an = sin + n x + sin − n x dx
π 0 2 2
an = −
π 1
2
+ n 1
2
−n 0
√
2 h− cos 2 + n π cos 2 − n π
1 1
1 1 i
= − + +
π 1
2
+n 1
2
−n 1
2
+ n 12 − n
√
2h 1 1 i π π
= + since cos nπ + = cos nπ − =0
π 12 + n 12 − n 2 2
√
2h 1 i
=
π 14 − n2
√
2h 4 i
an = .
π 1 − 4n2
√ ∞ √
√ 2 2 X 2 4
∴ 1 − cos x = + cos nx
π n=1
π 1 − 4n 2
√ √ ∞
√ 2 2 4 2X 1
1 − cos x = − cos nx.
π π n=1 4n2 − 1
Example 12.4.8. Find the Fourier series to represent the function f (x) = sin x, −π < x < π.
∞
X
sin x = bn sin nx,
n=1
where
2 π
Z
bn = sin x sin nxdx
π 0
Z π
1
= 2 sin x sin nxdx
π 0
1 π
Z
= [cos(1 − n)x − cos(1 + n)x]dx
π 0
1 h sin(1 − n)x sin(1 + n)x iπ
= −
π 1−n 1+n 0
If n = 1, then
2 π 2
Z
b1 = sin xdx
π 0
1 π
Z
= (1 − cos 2x)dx
π 0
1 sin 2x π
= x−
π 2 0
b1 = 1.
Example 12.4.9. Expand the function f (x) = | sin x| as a Fourier series in the interval −π < x <
π.
∞
a0 X
| sin x| = + an cos nx
2 n=1
2 π
Z
a0 = f (x)dx
π 0
2 π
Z
= | sin x|dx
π 0
2
= (− cos x)π0
π
2
= − [−1 − 1]
π
4
a0 = .
π
2 π
Z
an = f (x) cos nxdx
π 0
Z π
2
= | sin x| cos nxdx
π 0
2 π sin(n + 1)x − sin(n − 1)x
Z
= dx
π 0 2
1 π
Z
an = [sin(n + 1)x + sin(1 − n)x]dx
π 0
−2
an = [1 + (−1)n ], n , 1
π(n − 1)
2
0, when n is odd
an =
4
, when n is odd.
− (n2 −1)π
If n = 1, we have
2 π
Z
a1 = sin 2x
π 0
1 − cos 2x π
=
π 2 0
−1
= (cos 2π − 1)
2π
a1 = 0.
∞
2 X −4
∴ | sin x| = + cos nx
π n=2,4,6,.. π(n2 − 1)
∞
2 4 X cos nx
= − cos nx
π π n=2,4,6,.. n2 − 1
∞
2 4 X cos 2nx
= − Replacing n by 2n
π π n=1 n2 − 1
2 4 cos 2x cos 4x
Hence | sin x| = − + + ..... .
π π 3 15
π2 x2
Example 12.4.10. Express f (x) = 12
− 4
as a Fourier series in the interval −π < x < π.
Solution. Since
π2 (−x)2 π2 x2
f (−x) = − = − = f (x)
12 4 12 4
Therefore f (x) is an even function and hence
∞
a0 X
f (x) = + an cos nx
2 n=1
where
2 π
Z
a0 = f (x)dx
π 0
2 π h π2 x2 i
Z
= − dx
π 0 12 4
2 h π2 x x 3 iπ
= −
π 12 12 0
a0 = 0.
and
2 π
Z
an = f (x) cos nxdx
π 0
2 π h π2 x 2 i
Z
= − cos nxdx
π 0 12 4
Hence
∞
X (−1)n+1 cos 2x cos 3x cos 4x
f (x) = cos nx = cos x − + − + ....
n=1
n2 22 32 42
∞
2 X n(−1)n
sinh ax = sinh aπ sin nx.
π n=1
n 2 + a2
eax −e−ax
Solution. Let f (x) = sinh ax = 2
Now
e−ax − eax eax − e−ax
f (−x) = =− = − sinh ax = − f (x).
2 2
Therefore f (x) is an odd function.
∞
X
f (x) = bn sin nx
n=1
where
2 π
Z
bn = f (x) sin nxdx
π 0
2 π
Z
= sinh ax sin nxdx
π 0
Z π ax
2 e − e−ax
= sin nxdx
π 0 2
1 h π ax
Z Z π i
= e sin nxdx − e−ax sin nxdx
π 0 0
1 h eax π e−ax π i
= (a sin nx − n cos nx) − (−a sin nx − n cos nx)
π a2 + n2 0 a2 + n2 0
aπ −aπ
1 h e 1 e 1 i
= (−n cos nπ) − (−n) − (−n cos nπ) − (−n)
π a2 + n2 a2 + n2 a2 + n2 a2 + n2
n
= [−eaπ (−1)n + 1 + (−1)n e−aπ − 1]
π(a + n )
2 2
n
= (−1)n [e−aπ − eaπ ]
π(n + a2 )
2
n
= (−1)n+1 [eaπ − e−aπ ]
π(n + a2 )
2
n(−1)n+1
bn = 2 sinh aπ
π(n2 + a2 )
∞
2 X n(−1)n+1
∴ sinh x = sinh aπ sinh aπ sin nx.
π n=1
n 2 + a2
Example 12.4.12. If f (x) = cosh ax, expand f (x) as a Fourier series in (−π, π).
eax +e−ax
Solution. Let f (x) = cosh ax = 2
Now
e−ax + eax
f (−x) = = f (x).
2
Therefore f (x) is an even function.
∞
a0 X
f (x) = + an cos nx
2 n=1
where
2 π
Z
a0 = f (x)dx
π 0
2 π
Z
= cosh axdx
π 0
2 sinh ax π
=
π a 0
2 π
Z
an = f (x) cos nxdx
π 0
2 π
Z
= cosh ax sin nxdx
π 0
2 π eax + e−ax
Z
= sin nxdx
π 0 2
1 h π ax
Z Z π i
= e sin nxdx + e−ax sin nxdx
π 0 0
1 h eax π e−ax π i
= (a cos nx + n sin nx) + (−a cos nx + n sin nx)
π a2 + n2 0 a2 + n2 0
aπ −aπ
1 h e 1 e 1 i
= (a cos nπ + 0) − (a + 0) − (−a cos nπ + 0) − (−a + 0)
π a2 + n2 a2 + n2 a2 + n2 a2 + n2
a h i
= eaπ (−1)n − 1 + [−e−aπ (−1)n + 1]
π(a2 + n2 )
a
= (−1)n [eaπ − e−aπ ]
π(n + a2 )
2
2a(−1)n
= sinh aπ.
π(n2 + a2 )
∞
2a X 2a(−1)n
∴ cosh ax = sinh aπ sinh aπ cos nx.
π n=1
n2 + a2
Example 12.4.13. Obtain Fourier series for the function f (x) given by
− 2 (π + x), for − π < x ≤ 0
1
f (x) =
12 (π − x), for 0 ≤< π.
Solution. Since
−1
f (−x) = (π − x) in (−π, 0) = − f (x) in (0, π)
2
and
1
f (−x) = (π + x) in (0, π) = − f (x) in (−π, 0)
2
∞
X
∴ f (x) = bn sin nx
n=1
where
2 π
Z
bn = f (x) sin nxdx
π 0
2 π1
Z
= (π − x) sin nxdx
π 0 2
1h − cos nx − sin nx iπ
= (π − x) − (−1)
π n n2 0
1 h π i
= − 0− +0
π n
1
= .
n
∞
X 1 1 1
∴ f (x) = sin nx = sin x + sin 2x + sin 3x + ....
n=1
n 2 3
2x
f (−x) = 1 − in (−π, 0) = − f (x) in (0, π)
π
and
2x
f (−x) = 1 + in (0, π) = f (x) in (−π, 0)
π
Therefore f (x) is an even function in (−π, π).
(b)
∞
a0 X
∴ f (x) = + an cos nx
2 n=1
where
2 π
Z
a0 = f (x)dx
π 0
2 π
Z
2x
= 1− dx
π 0 π
a0 = 0.
2 π
Z
an = f (x) cos nxdx
π 0
2 π
Z
2x
= 1− cos nxdx
π 0 π
2h 2x sin nx 2 − cos nx iπ
= Big(1 − − (− )
π π n π n2 0
2 h cos nπ 2 i
= − 0 − 2 2 − (0 − 2
π nπ nπ
4
= [1 − (−1)n ].
n2 π2
n2 π2 ,
8
if n is odd
an =
0,
if n is even.
∞
8 X 1 8h cos 3x cos 5x i
∴ f (x) = 2 cos nx = cos x + + + .... .
π n=1,3,5,.. n2 π2 32 52
81 1 1
f (0) =
+ + + ...
π2 12 32 52
81 1 1
1 = 2 2 + 2 + 2 + ...
π 1 3 5
1 1 1 π2
+ + + ...... = .
12 32 52 8
12.5 Exercises
x2
1. If (i) f (x) = 2x ; (ii) f (x) = 4
expand f (x) as a Fourier series in the interval (−π, π).
2. Find the Fourier series expansion of the function f (x) = x cos x, −π < x < π.
3. Find the Fourier series to represent the function f (x) = | cos x|, −π < x < π.
4. If x lies between −π and π and a is neither zero nor an integer, prove that
8. Given
π + x for − π < x ≤ 0
f (x) =
π − x, for 0 < x < π.
Is the function f (x) even? If so, find the Fourier series for f (x).
Answers.
1.(i) x
2
= sin x. 12 sin 2x + 31 sin 3x. 14 sin 4x + ....
x2 π2
(ii). 2
= 12
− 1
12
cos x + 1
22
1
cos 2x −
32
cos 3x + ...
P∞ (−1)n n sin nx
2. x cos x = sin x +
−1
2
2 n=2 n2 −1 .
3. | cos x| = π2 + π4 31 cos 2x − 151 cos 4x + ...
π cosh ax P a(−1)n
5. 2 sinh aπ
= 2a1 + ∞ n=1 a2 +n2 cos nx.
Lesson 13
13.1 Introduction
I
t is often required to obtain Fourier series of a function f (x) in the interval (0, π).
The Sine Series. If it be required to express f (x) as a sine series in (0, π), we define an odd
function f1 (x) in (−π, π), identical with f (x) in (0, π). That is, we extend the function reflecting
it with respect to the origin, so that f (−x) = − f (x).
Hence the half range sine series in (0, π) is given by
∞
X
f (x) = bn sin nx
n=1
Z π
2
where bn = f (x) sin nxdx.
π 0
The Cosine Series. If it be required to express f (x) as a cosine series in (0, π), we define an odd
function f2 (x) in (−π, π), identical with f (x) in (0, π). That is, we extend the function reflecting
it with respect to the y - axis, so that f (−x) = − f (x).
Hence the half range cosine series in (0, π) is given by
∞
a0 X
f (x) = + an cos nx
2 n=1
2 π
Z
where a0 = f (x)dx
π 0
2 π
Z
and an = f (x) cos nxdx.
π 0
Note.1 Suppose f (x) = x in [0, π]. It can have Fourier cosine series expansion as well as Fourier
sine series expansion in [0, π].
2. Similarly f (x) = x2 in [0, π] can have Fourier cosine series expansion as well as Fourier sine
series expansion in [0, π].
We note that, if f (x) satisfies Dirichlet’s conditions in (0, π) we can have valid expansion of f (x)
in terms of sines only or cosines only in that interval.
Example 13.2.1. Find the half range cosine and sine series of f (x) = x, 0 < x < π. Hence
π2
deduce that 1
12
+ 1
32
+ 1
52
+ ... = 8
.
Solution. The half range cosine series expansion of f (x) in [0, π] is given by
∞
a0 X
f (x) = + an cos nx
2 n=1
where
2 π
Z
a0 = f (x)dx
π 0
2 π
Z
= xdx
π 0
2 x2 π
=
π 2 0
a0 = π.
2 π
Z
an = f (x) cos nxdx
π 0
2 π
Z
= x cos nxdx
π 0
2 h sin nx cos nx iπ
= x − 1. −
π n n2 0
n
2 h (−1) 1 i
= 0 + 2 − (0 + 2 )
π n n
2
an = [(−1)n − 1]
πn2
0, for n even
an =
2 π , for n odd
n−4
∞
π 4 X 1
∴x = − cos nx
2 π n=1,3,5,.. n2
π 4 cos x cos 3x
i.e., x = − + + ..... .
2 π 12 32
Deduction. When x = 0, f (x) = 0, i.e., f (0) = 0. Putting x = 0 in the above series, we have
π 4 1 1 1
0 =
− + + .....
2 π 12 32 52
1 1 1 π2
+ + ..... = .
12 32 52 8
∞
X
f (x) = bn sin nx
n=1
where
2 π
Z
bn = f (x) sin nxdx
π 0
2 π
Z
= x sin nxdx
π 0
2 h − cos nx sin nx iπ
= x − 1. −
π n n2 0
n
2 h (−1) i 2(−1)n+1
bn = −π + 0 − (0 + 0) = .
π n n
∞
X (−1)n+1 2
∴x = sin nx
n=1
n
sin 2x sin 3x
i.e., x = 2 sin x − + − ..... .
2 3
Example 13.2.2. Expand πx − x2 in half range sine series in the interval [0, π] up to three terms.
π3
Deduce that 1
13
− 1
33
+ 1
53
− 1
73
+ ..... = 32
.
Solution. The half range sine series expansion of f (x) in [0, π] is given by
∞
X
f (x) = bn sin nx
n=1
where
2 π
Z
bn = f (x) sin nxdx
π 0
2 π
Z
= (πx − x2 ) sin nxdx
π 0
2h − cos nx sin nx cos nx iπ
= (πx − x2 ) − (π − 2x). − + (−2)
π n n2 n3 0
n
2 h (−1) 2 i
= 0 + 0 − 2 3 − (0 + 0 − 3 )
π n n
2h 2
= (1 − (−1)n )
π n3
4
bn = [1 − (−1)n ].
n3 π
0, for n even
bn =
n83 π , for n odd
∞
X 8
∴ πx − x = 2
sin nx
n=1,3,5,..
πn3
8 sin 3x sin 5x
i.e., πx − x2 = sin x + + + ..... .
π 33 53
π
Deduction. Putting x = 2
in the above series, we have
π π 8 π 1 3π 1 5π
x− = sin + 3 sin + 3 sin + .....
2 2 π 2 3 2 5 2
π2 8h 1 1 1 i
= 1 − 3 + 3 − 3 + ....
4 π 3 5 7
1 1 1 π2
1− + − + .... = .
33 53 73 32
Example 13.2.3. Find cosine and sine series for f (x) = π − x in [0, π].
Solution. The half range cosine series expansion of f (x) in [0, π] is given by
∞
a0 X
f (x) = + an cos nx
2 n=1
where
2 π
Z
a0 = f (x)dx
π 0
2 π
Z
= (π − x)dx
π 0
2 x2 π
= πx −
π 2 0
2 2 π2
= π − − (0 − 0)
π 2
a0 = π.
2 π
Z
an = f (x) cos nxdx
π 0
2 π
Z
= (π − x) cos nxdx
π 0
2h sin nx cos nx iπ
= (π − x) − (−1). −
π n n2 0
n
2 h (−1) 1 i
= 0 − 2 − (0 − 2 )
π n n
2h 1
= (1 − (−1)n )
π n2
2
bn = [1 − (−1)n ].
n2 π
0, for n even
an =
n42 π , for n odd
∞
π 4 X 1
∴π−x = + cos nx
2 π n=1,3,5,.. n2
∞
X
π−x= bn sin nx
n=1
2 π
Z
bn = f (x) sin nxdx
π 0
Z π
2
= (π − x) sin nxdx
π 0
2h − cos nx sin nx iπ
= (π − x) − (−1). −
π n n2 0
2h 1 i
= − 0 − 0 − (−π )
π n
2
bn =
n
∞
X 1 1 1
∴π−x=2 sin nx = 2 sin x + sin 2x + sin 3x + ... .
n=1
n 2 3
πx
Example 13.2.4. Obtain the half range cosine and sine series for the function f (x) = 8
(π − x) in
[0, π].
Solution. The half range cosine series expansion of f (x) in [0, π] is given by
∞
a0 X
f (x) = + an cos nx
2 n=1
where
2 π
Z
a0 = f (x)dx
π 0
2 π πx
Z
= (π − x)dx
π 0 8
1 π
Z
= (π − x)dx
4 0
1 x2 x3 π
= π −
4 2 3 0
1 π
3
π 3
π3
a0 = − = .
4 2 3 24
2 π
Z
an = f (x) cos nxdx
π 0
2 π πx
Z
= (π − x) cos nxdx
π 0 8
1 π
Z
= (πx − x2 ) cos nxdx
4 0
1h sin nx cos nx − sin nx iπ
= (πx − x2 ) − (π − 2x). − + (−2)
4 n n2 n3 0
n
1 h (−1) 1 i
= 0 − π 2 + 0 − (0 + π 2 − 0)
4 n n
1 h −1
= (1 + (−1) ) n
4 n2
π
an = − 2 [1 + (−1)n ].
4n
0, when n is odd
an =
2 , when n is even
−π
2n
∞
πx π3 π X 1
∴ (π − x) = − cos nx
8 48 2 n=2,4,6,.. n2
πx π3 π 1 cos 4x cos 6x
i.e., (π − x) = − cos 2x + + + ..... .
8 48 2 22 42 62
∞
X
π−x= bn sin nx
n=1
2 π
Z
bn = f (x) sin nxdx
π 0
2 π πx
Z
= (π − x) sin nxdx
π 0 8
1 π
Z
= (πx − x2 ) sin nxdx
4 0
1h − cos nx sin nx cos nx iπ
= (πx − x2 ) − (π − 2x). − + (−2)
4 n n2 n3 0
n
1 h (−1) 1 i
= 0 − 0 − 2 3 − (0 − 0 − 2 3 )
4 n n
1
bn = [1 − (−1) ] n
2n3
0, when n is even
bn =
n13 , when n is odd
∞
πx X 1 1 1 1
∴ (π − x) = 3
sin nx = 3 sin x + 3 sin 3x + 3 sin 5x + ....
8 n=1
n 1 3 5
Example 13.2.5. Express the function f (x) = sin x as Fourier cosine series in the interval (0, π).
Hence show that ∞n=1 4n2 −1 = 2 .
P 1 1
∞
a0 X
sin x = + an cos nx
2 n=1
2 π
Z
a0 = f (x)dx
π 0
Z π
2
= sin xdx
π 0
2
= (− cos x)π0
π
2
= − [−1 − 1]
π
4
a0 = .
π
2 π
Z
an = f (x) cos nxdx
π 0
2 π
Z
= sin x cos nxdx
π 0
Z π
2 sin(1 + n)x + sin(1 − n)x
= dx
π 0 2
1 π
Z
= [sin(1 + n)x + sin(1 − n)x]dx
π 0
1 h cos(n + 1)x cos(1 − n)x iπ
= − − (n , 1)
π n+1 1−n 0
−1 h cos(n + 1)π cos(1 − n)π 1 1 i
= + − − , n,1
π n+1 1−n n+1 n−1
1 h (−1)n+1 − 1 (−1)n+1 − 1 i
an = − +
π n+1 1−n
−1 h 1 1 1 1 i
an = (−1)n+1 + − +
π 1+n 1−n 1+n 1−n
−1 h 2 2 i
= (−1)n+1 . 2 −
π n − 1 1 − n2
2
= [(−1)n+1 − 1]
π(n − 1)
2
−2
an = [1 + (−1)n ], n , 1
π(n2 − 1)
0, when n is odd
an =
4
, when n is odd.
− (n2 −1)π
If n = 1, we have
2 π
Z
a1 = sin 2x
π 0
1 − cos 2x π
=
π 2 0
−1
= (cos 2π − 1)
2π
a1 = 0.
∞
2 X −4
∴ sin x = + cos nx
π n=2,4,6,.. π(n2 − 1)
∞
2 4 X cos nx
= − cos nx
π π n=2,4,6,.. n2 − 1
∞
2 4 X cos 2nx
sin x = − Replacing n by 2n
π π n=1 4n2 − 1
∞ ∞
2 4X 1 X 1 1
0= − ⇒ = .
π π n=1 4n − 1
2
n=1
2
4n − 1 2
Example 13.2.6. Find the half - range sine and cosine series of f (x) = e x , 0 < x < π.
Solution. The half range sine series expansion of f (x) = e x in (0, π) is given by
∞
X
e =x
bn sin nx
n=1
where
2 π
Z
bn = f (x) sin nxdx
π 0
2 π x
Z
= e sin nxdx
π 0
2 ex π
= (sin nx − n cos nx)
π 1 + n2 0
π
2 e
1
= (0 − n cos nπ) − (0 − n)
π 1 + n2 1 + n2
n+1 π
2 h (−1) ne n i
= +
π 1 + n2 1 + n2
2n
bn = [1 + (−1)n+1 eπ ]
π(1 + n ) 2
∞
2 X n[1 + (−1)n+1 eπ
∴e =
x
sin nx.
π n=1 n2 + 1
∞
a0 X
f (x) = + an cos nx
2 n=1
where
2 π
Z
a0 = f (x)dx
π 0
Z π
2
= e x dx
π 0
2 x π
= e
π 0
2 π
= (e − 1)
π
2 π
Z
an = f (x) cos nxdx
π 0
Z π
2
= e x cos nxdx
π 0
2 ex π
= (cos nx + n sin nx)
π 1 + n2 0
2 eπ 1
an = (cos nπ + 0) − (1 + 0)
π 1 + n2 1 + n2
2 h eπ cos nπ − 1 i
an =
π 1 + n2
π
2 e (−1)n − 1
an =
π 1 + n2
∞
eπ − 1 2 X eπ (−1)n − 1
∴ ex = + cos nx.
π π n=1 n2 + 1
Example 13.2.7. Obtain Fourier sine series for f (x) = x sin x, 0 < x < π and show that
π−2
1
1.3
− 1
1.5
+ 1
5.7
− .... = 4
.
Solution.
∞
a0 X
x sin x = + an cos nx
2 n=1
2 π
Z
a0 = f (x)dx
π 0
Z π
2
= x sin xdx
π 0
2 π
= x(− cos x) − 1.(− sin x)
π 0
2
= [−π(−1) + 0 − (0 + 0)]
π
a0 = 2.
2 π
Z
an = f (x) cos nxdx
π 0
Z π
2
= x sin x cos nxdx
π 0
1 π sin(n + 1)x − sin(n − 1)x
Z
= x dx
π 0 2
1 π
Z
= x[sin(n + 1)x − sin(n − 1)x]dx
π 0
1 h cos(n + 1)x cos(n − 1)x sin(n + 1)x sin(n − 1)x iπ
= x − + − 1. − + (n , 1)
π n+1 n−1 (n + 1)2 (n − 1)2 0
1 h cos(n + 1)π cos(n − 1)π i
= 2π − + , n,1
π n+1 n−1
h (−1)n+1 (−1)n−1 (−1)2 i
an = − − n,1
n+1 n−1
h (−1)n+1 (−1)n+1 i
an = − − , n,1
n+1 n−1
hn − 1 − n − 1i
= (−1)(−1)n+1 n,1
n2 − 1
2(−1)n+1
an = , n , 1.
n2 − 1
If n = 1, we have
Z π
a1 = x sin 2x
π 0
1 h − cos 2x − sin 2x iπ
= x − 1.
π 2 4 0
1
= (−π)
2π
1
a1 = − .
2
π
Deduction. Putting x = 2
in the above series, we get
π π 2 2 −2
sin = 1−0− (−1) + (0) + (1) + ....
2 2 1.3 2.4 3.5
π 2 2 2 2
= 1+ − + − + ....
2 1.3 3.5 5.7 7.9
π 2 2 2 2
−1 = − + − + ....
2 1.3 3.5 5.7 7.9
1 1 1 1 1
− + − + .... = (π − 2).
1.3 3.5 5.7 7.9 4
eax −e−ax
Example 13.2.8. Find the half - range sine series for the function f (x) = eaπ −e−aπ
in (0, π).
Solution.
∞
X
f (x) = bn sin nx
n=1
where
2 π
Z
bn = f (x) sin nxdx
π 0
2 π eax − e−ax
Z
= sin nxdx
π 0 eaπ − e−aπ
2 hZ π Z π i
= e ax
sin nxdx − e−ax
sin nxdx
π(eaπ − e−aπ ) 0 0
2 h e ax π e−ax π i
= (a sin nx − n cos nx) − (−a sin nx − n cos nx)
π(eaπ − e−aπ ) a2 + n2 0 a2 + n2 0
aπ −aπ
2 h e 1 e 1 i
= (−n cos nπ) − (−n) − (−n cos nπ) − (−n)
π(eaπ − e−aπ ) a2 + n2 a2 + n2 a2 + n2 a2 + n2
2 1
= [−eaπ (−1)n + 1 + (−1)n e−aπ − 1]
π(e − e ) a + n
aπ −aπ 2 2
2
= (−1)n [e−aπ − eaπ ]
π(e − e )(n2 + a2 )
aπ −aπ
2
= [eaπ − e−aπ ]
π(e − e )(n + a )
aπ −aπ 2 2
2n(−1)n+1
= .
π(n2 + a2 )
∞
eax − e−ax 2 X n(−1)n+1
∴ aπ = sin nx.
e − e−aπ π n=1 n2 + a2
Example 13.2.9. If
π
in 0 < x <
1,
2
f (x) =
π
< x < π.
−1,
in 2
Solution.
∞
a0 X
f (x) = + an cos nx
2 n=1
where
2 π
Z
a0 = f (x)dx
π 0
Z π Z π
2h 2
= 1.dx + (−1)dx
π 0 π
2
2 h π2
(x)0 − (x)ππ
i
a0 =
π 2
2 h π π i
a0 = −0 − π−
π 2 2
a0 = 0.
and
2 π
Z
an = f (x) cos nxdx
π 0
Z π Z π
2h 2
= 1. cos nxdx + (−1) cos nxdx
π 0 π
2
2 h sin nx π2 sin nx π i
= −
π n 0 n π2
2h1 nπ 1 nπ i
= sin − 0 − 0 − sin
π n 2 n 2
2 2 nπ
= . sin
π n 2
4 nπ
an = sin
nπ 2
∴ an = 0, when n is even.
Hence
∞
X 4 nπ
f (x) = sin cos nx
1,3,5,...
nπ 2
4 π 1 3π 1 5π
=Big(sin cos x + sin cos 3x + sin cos 5x + ....
π 2 3 2 5 2
4 1 1
f (x) = cos x − cos 3x + cos 5x − ...
π 3 5
Solution.
∞
X
f (x) = bn sin nx
n=1
where
Z π
2
bn = f (x) sin nxdx
π 0
Z π Z π
2h 2 π
bn = . sin nxdx + (π − x) sin nxdx
π 0 2 π
2
2 h −π nπ π π nπ 1 nπ i
= cos + + cos + 2 sin
π 2n 2 2n 2n 2 n 2
2h π 1 nπ i
bn = + sin
π 2n n2 2
Hence
∞
X 2h π 1 nπ i
f (x) = + 2 sin sin nx
1
π 2n n 2
∞
2 Xh π 1 nπ i
f (x) = + 2 sin
π n=1 2n n 2
in π2 < x < π.
0,
Solution.
∞
a0 X
f (x) = + an cos nx
2 n=1
where
2 π
Z
a0 = f (x)dx
π 0
Z π Z π
2h 2
= cos x.dx + 0.dx
π 0 π
2
2h πi
= (sin x)02
π
2
= (1 − 0)
π
2
a0 = .
π
and
Z π
2
an = f (x) cos nxdx
π 0
Z π Z π
2h 2
an = cos x cos nxdx + 0. cos nxdx
π 0 π
2
Z π
1 2
= [cos(n + 1)x + cos(n − 1)x]dx
π 0
1 h sin(n + 1)x sin(n − 1)x i π2
= + n,1
π n+1 n−1 0
π π
1 h sin(n + 1) 2 sin(n − 1) 2 i
= + , n,1
π n+1 n−1
1h 1 nπ π 1 nπ πi
= cos sin − cos sin , n,1
π n+1 2 2 n−1 2 2
1 nπ h 1 1 i
= cos − , n,1
π 2 n+1 n−1
2 nπ
an = − 2 cos , n , 1.
π(n − 1) 2
If n = 1, then
Z π
1 2
a1 = 2 cos2 xdx
π 0
Z π
1 2
= (1 + cos 2x)dx
π 0
1 sin 2x π2
= x+
π 2 0
1 π
h i
= + 0 − (0 + 0)
π 2
1
a1 = .
2
Hence
∞
1 1 2X 1 nπ
f (x) = + cos x − cos cos nx
π 2 π n=1 n2 − 1 2
1 1 2h cos π 1 3π i
= + cos x − cos 2x + cos cos 3x + ....
π 2 π (2 − 1)(2 + 1) (3 − 1)(3 + 1) 2
1 1 2 h cos 2x cos 4x cos 6x i
= + cos x − − + − + ...
π 2 π 1.3 3.5 5.7
1 1 2 h cos 2x cos 4x cos 6x i
f (x) = + cos x + + + + ....
π 2 π 1.3 3.5 5.7
4 1 1
f (x) = cos x − cos 3x + cos 5x − ...
π 3 5
Solution.
∞
X
f (x) = bn sin nx
n=1
where
2 π
Z
bn = f (x) sin nxdx
π 0
Z π Z π
2h 2
= x sin nxdx + (π − x) sin nxdx
π 0 π
2
2 h −π nπ 1 nπ π nπ 1 nπ i
= cos + 2 sin −0 + 0+ cos + 2 sin
π 2n 2 n 2 2n 2 n 2
4 nπ
bn = sin
πn2 2
bn = 0, when n is even
and
4 nπ
bn = sin , when n is odd.
nπ
2 2
Hence
∞
X 4 nπ
f (x) = 2π
sin sin nx
1
n 2
4h 1 nπ 1 3π 1 5π i
= sin sin x + sin sin 3x + sin sin 5x + ...
π 12 2 32 2 52 2
4 h 1 1 i
= sin x − 2 sin 3x + 2 sin 5x − .... .
π 3 5
π − x, in π2 < x < π.
Solution.
∞
a0 X
f (x) = + an cos nx
2 n=1
where
2 π
Z
a0 = f (x)dx
π 0
Z π Z π
2h 2 i
= xdx + (π − x)dx
π 0 π
2
2 π
2 h x 2 x2 π i
= + πx −
π 2 0 2 π2
2 h π2 2 π2 π2 π2 i
= + π − − −
π 8 2 2 8
2
2 2π
=
π 8
π
a0 = .
2
and
2 π
Z
an = f (x) cos nxdx
π 0
Z π Z π
2h 2
= x cos nxdx + (π − x) cos nxdx
π 0 π
2
2 h π nπ 1 nπ 1 cos nπ π nπ 1 nπ i
= sin + 2 cos − 2 + − − sin + cos
π 2n 2 n 2 n n2 2n 2 n2 2
2 h nπ i
an = 2 cos − 1 − (−1) n
πn2 2
an = 0, when n is odd
and
2 h nπ i
an = 2 cos − 2 , when n is even.
n2 π 2
Hence
∞
π X 2 h nπ i
f (x) = + 2 cos − 2 cos nx
4 n=2,4,6,.. n2 π 2
∞
π X 2
= + [2 cos nπ − 2] cos 2nx, Replacing n with 2n
4 n=1 π(4n)2
∞
π 1 X 1
f (x) = + 2
[2(−1)n − 2] cos 2nx
4 2π n=1 n
∞
π 1 X −2
f (x) = + cos 2nx
4 π n=1,3,5,.. n2
π 2h 1 1 1 i
f (x) = − 1 2 cos 2x + 2 cos 6x + 2 cos 10x + ...... .
4 π 1 3 5
Example 13.2.14. Find the Fourier sine series of f (x) = cos x in 0 < x < π.
Solution.
∞
X
f (x) = bn sin nx
n=1
where
2 π
Z
bn = f (x) sin nxdx
π 0
2 π
Z
= cos x cos nxdx
π 0
Z π
1 2
= [sin(n + 1)x + sin(n − 1)x]dx
π 0
1 h − cos(n + 1)x cos(n − 1)x iπ
= − n,1
π n+1 n−1 0
1 h (−1)n+1 (−1)n 1 1 i
= − + + + , n,1
π n+1 n−1 n+1 n−1
1h 1 1 1 1 i
= (−1)n + + + , n,1
π n+1 n−1 n+1 n−1
1h 1 1 i
= [(−1)n + 1] + + , n,1
π n+1 n−1
2n h 1 + (−1)n i
bn = , n , 1.
π n2 − 1
0, when n is odd
bn =
2 −1) ,
π(n4n
when n is even.
If n = 1, then
2 π
Z
b1 = cos x sin xdx
π 0
Z π
1
= sin 2xdx
π 0
1 − cos 2x π
=
π 2 0
1h i
b1 = − cos 2π − cos 0 = 0.
2π
Hence
∞
X 4n
f (x) = sin nx
n=2,4,6,...
π(n2 − 1)
∞
4 X 2n
= sin 2nx, Replacing n by 2n
π n=1 4n2 − 1
81 2 3
= sin 2x + sin 4x + sin 6x + ....
π 3 15 35
4 2 4 6
f (x) = sin 2x + sin 4x + sin 6x + ...... .
π 1.3 3.5 5.7
13.3 Exercises
then show that (i) f (x) = sin x − 312 sin 3x + 512 sin 5x − 1
72
sin 7x + ....
π2
(ii) f (x) = 16 − 12 112 cos x + 312 cos 6x + ....
12. Find the Fourier sine and cosine series in 0 < x < π for the function
π
, when 0 < x < π3
3
f (x) =
when π3 < x < 2π
0, 3
− π , when 2π < x < π.
3 3
Answers.
h i
1. 1 = π4 sin x + sin 3x
3
+ sin 5x
5
+ ..... .
π
h i
2. π − x = 2
+ 4 cos x
π 12
+ cos 3x
32
+ cos 5x
52
+ .... .
h i
π − x = 2 sin x + sin22x + sin33x + .... .
3. x2 = π3 − 4 cos
2
h i
12
x
− cos 2x
22
+ cos 3x
32
− cos 4x
42
+ ....
h i
x = π (π − 4) sin x − 2 π sin 2x + 13 π2 − 342 sin 3x − 14 π2 sin 4x + ......
2 2 2 1 2
π2
4. f (x) = 2 ∞ n 6
P
n=1 (−1) n3 − n
sin nx.
h aπ aπ aπ ) i
5. eax = π2 1+e
a2 +1
sin x + 2(1−ea2 +22
)
sin 2x + 3(1+e
a2 +32
sin 3x + ...
6. f (x) = sin x
13
+ sin 3x
33
+ sin 5x
53
+ ...
7. x sin x = 1 − cos 2x
2
− 2 cos 2x
1.3
+ 2 cos 3x
2.4
− 2 cos 4x
3.5
+ ....
h i
8. x cos x = − 12 sin x + 2 a sin 2x
1.3
− 3 sin 3x
2.4
+ 4 sin 4x
3.5
− ....
9. f (x) = 5
2
+ 5 cos x + 23 cos 2x + cos 3x + ....
10. 2 sin 2x cos x = sin x + sin 3x.
12. f (x) = sin 2x + 21 sin 4x + 41 sin 8x + 15 sin 10x + 71 sin 4x + .....
h i
f (x) = √23 cos x − 15 cos 5x + 17 cos 7x − 11 1
cos 11x + ...... .
Lesson 14
• find the Fourier series of functions in [c, c + 2l] and [−l, l].
14.1 Introduction
S
o far, we have expanded a given function in a Fourier series over the intervals (−π, π)
and (0, 2π) of length 2π. In most engineering problems, the period of the function to
be expanded is not 2π but some other quantity say 2l. In order to apply earlier discussions to
functions of period 2l, this interval must be converted to the length 2π. This involves only a
proportional change in the scale.
Let f (x) be a periodic function with period 2l defined in the interval c < x < c + 2l. We must
introduce a new variable z such that the period becomes 2π.
2l 2π l πx
Put = i.e., x = z or z = .
x z π l
πc
When x = c, z = = d (say)
l
π πc
when x = c + 2l, z = (c + 2l) = + 2π = d + 2π.
l l
Hence the function f (x) of period 2l in (c, c + 2l) is transferred to the new function
lz
f = F(z) (say)
π
∞ ∞
a0 X X
F(z) = + an cos nx + bn sin nx (14.1.1)
2 n=1 n=1
where
1 d+2π
Z
a0 = F(z)dz,
π d
1 d+2π
Z
an = F(z) cos nzdz,
π d
1 d+2π
Z
and bn = F(z) sin nzdz
π d
πx
To change the variable z to x, put z = l
so that dz = πl dx.
When z = d, x = c and when z = d + 2π, x = c + 2l. Also
lz
F(z) = f = f (x)
π
Then
1 c+2l π 1 c+2l
Z Z
a0 = f (x). dx = f (x)dx,
π c l l c
1 c+2l nπx π 1 c+2l
Z Z
nπx
an = f (x) cos . dx = f (x) cos dx
π c l l l c l
1 c+2l nπx π 1 c+2l
Z Z
nπx
bn = f (x) sin . dx = f (x) sin dx
π c l l l c l
Using these a0 , an , and bn in (14.1.1), the Fourier expansion for f (x) in the interval c < x <
c + 2l is given by
∞ ∞
a0 X nπx X nπx
f (x) = + an cos + bn sin
2 n=1 l n=1
l
where
1 c+2l
Z
a0 = f (x)dx,
l c
1 c+2l
Z
nπx
an = f (x) cos dx (14.1.2)
l c l
1 c+2l
Z
nπx
bn = f (x) sin dx.
l c l
1 2l
Z
a0 = f (x)dx,
l 0
1 2l
Z
nπx
an = f (x) cos dx
l 0 l
1 2l
Z
nπx
bn = f (x) sin dx.
l 0 l
1 l
Z
a0 = f (x)dx,
l −l
1 l
Z
nπx
an = f (x) cos dx
l −l l
1 l
Z
nπx
bn = f (x) sin dx.
l −l l
0, if m , n,
Z c+2l nπx nπx
dx =
sin sin l, if m = n , 0,
c l l
0, if m = n = 0
0, if m , n,
Z c+2l nπx nπx
dx =
cos cos if m = n , 0,
l,
c l l
if m = n = 0
2l,
Z l Z l
1 nπx 2 nπx
∴ an = f (x) cos dx = f (x) cos dx.
l −l l l 0 l
Z l
1 nπx
∴ bn = f (x) sin dx = 0.
l −l l
Hence if f (x) is even in the interval (−l, l) then the Fourier cosine series is
∞
a0 X nπx
f (x) = + an cos
2 n=1 l
Z l
2
where a0 = f (x)dx,
l 0
2 l
Z
nπx
and an = f (x) cos dx
l 0 l
Z l
1 nπx
∴ an = f (x) cos dx = 0.
l −l l
Z l Z l
1 nπx 2 nπx
∴ bn = f (x) sin dx = f (x) sin dx.
l −l l l 0 l
Hence if f (x) is an odd in the interval (−l, l) then the Fourier sine series is
∞
X nπx
f (x) = bn sin
n=1
l
Z l
2 nπx
where bn = f (x) sin dx
l 0 l
Remark. The even function or odd function is to be considered only if the function defined in the
interval (−l, l).
Note.
1. In the above discussion, if we put 2l = 2π, i.e., l = π, we get the discussions regarding the
Solution. Since f (x) = x2 is an even function, the corresponding Fourier series is given by
∞
a0 X nπx
f (x) = + an cos . (14.3.1)
2 n=1 l
where
2 l
Z
a0 = f (x)dx
l 0
2 l 2
Z
= x dx
l 0
2 x3 l
=
l 3 0
22
a0 = l .
3
and
2 l
Z
nπx
an = f (x) cos dx
l 0 l
2 l 2
Z
nπx
= x cos dx
l 0 l
nπx − cos nπx sin nπx
2 h 2 sin l il
= x nπ − 2x n π
2 2
l
+ 2 − n π
3 3
l
l l 2 3
0
l l
∞
l2 X 4l2 nπx
x =
2
+ (−1)n cos
3 n=1 n π 2 2 l
∞
l2 4l2 X (−1)n nπx
= + 2
cos
3 3 n=1 n l
l2 4l2 h 1 πx 1 2πx 1 3πx i
= + − 2 cos + 2 cos − 2 cos + ...
3 3 1 l 2 l 3 l
l 2
4l 2h
1 πx 1 2πx 1 3πx i
x2 = − cos − cos + cos − ...
3 3 12 l 22 l 32 l
1 4h 1 1 1 i
x2 = − cos πx − cos 2πx + cos 3πx − ... .
3 3 12 22 32
Example 14.3.2. Obtain Fourier series expansion for sin ax in the interval −l < x < l.
Solution. Since sin ax is an odd function in (−l, l) , therefore, the required series is of the form
∞
X nπx
sin ax = bn sin (14.3.2)
n=1
l
where
2 l
Z
nπx
bn = f (x) sin dx
l 0 l
2 l
Z
nπx
= sin ax cos dx
l 0 l
∞
X (−1)n+1 2nπ sin al nπx
∴ (14.3.2) ⇒ sin ax = 2π sin al sin
n=1
n2 π2 − a2 l 2 l
h 1 πx 2 2πx 3 3πx i
sin ax = 2π sin al sin − 2 sin + 2 sin − ... .
π2 2
−a l 2 l 2
2π − a l 2 l 2
3π − a l 2 l
Example 14.3.3. Find the Fourier series to represent f (x) = 1 − x2 in the interval −l ≤ x ≤ l.
Solution. Since f (x) = 1 − x2 is an even function, the corresponding Fourier series is given by
∞
a0 X nπx
f (x) = + an cos . (14.3.3)
2 n=1 l
Here l = 1
∞
a0 X
∴ 1 − x2 = + an cos nπx
2 n=1
where
2 l
Z
a0 = f (x)dx
1 0
2 l
Z
= (1 − x2 )dx
1 0
x3 1 1 4
= 2 x− =2 1− = .
3 0 3 3
and
Z 1
2
an = f (x) cos nπxdx
1 0
Z 1
= 2 (1 − x2 ) cos nπxdx
0
h sin nπx − cos nπx sin nπx i1
= 2 (1 − x2 ) − (−2x) + (−2) − 3 3
nπ n2 π2 nπ 0
4
= − 2 2 cos nπ
nπ
4
an = 2 2 (−1)n+1 .
nπ
∞
2 4 X (−1)n+1
1−x = + 2
cos nπx
3 π2 n=1 n2
2 4h1 1 1 i
= + 2 2 cos πx − 2 cos 2πx + 2 cos 3πx − ...
3 π 1 2 3
π2 ,
1
< x < 2.
2
Solution. Let
∞ ∞
a0 X nπx X nπx
f (x) = + an cos + bn sin
2 n=1 l n=1
l
Here 2l = 1 ⇒ l = 12 . Thus
∞ ∞
a0 X X
f (x) = + an cos 2nπx + bn sin 2nπx (14.3.4)
2 n=1 n=1
where
Z 2l
1
a0 = f (x)dx
l 0
Z 1
= 2 f (x)dx
0
1
π π i
hZ 2
Z 1
= 2 − dx + dx
0 2 1
2
2
h π 1 π i
= 2 − (x)0 + (x)11
2
2 2 2
h π πi
= 2 − +
4 4
a0 = 0.
Z 2l
1 nπx
an = f (x) cos dx
l 0 l
Z 1
= 2 f (x) cos 2nπxdx
0
1
π π
hZ 2
Z 1 i
= 2 − cos 2nπxdx + cos 2nπxdx
0 2 1
2
2
h π sin 2nπx 21 π sin 2nπx 1 i
= 2 − +
2 2nπ 0 2 2nπ 21
h π π i
= 2 − (0 − 0) + (0 − 0)
2 2
an = 0.
Z 2l
1 nπx
bn = f (x) sin dx
l 0 l
Z 1
bn = 2 f (x) sin 2nπxdx
0
1 1
= (cos 2nπ − 1) − (cos 2nπ − cos nπ)
2n 2n
1 h i
bn = (−1)n − 1 − 1 − (−1)n
2n
0, when n is even
bn =
− 2n , when n is odd.
∞
X −2 h 1 1 i
∴ (14.3.4) ⇒ f (x) = sin 2nπx = (−2) sin 2πx + sin 6πx + sin 10πx + .... .
n=1,3,5,...
n 3 5
0, −2 < x < −1
f (x) =
k, −1 < x < 1
0, 1 < x < 1
∞ ∞
a0 X nπx X nπx
f (x) = + an cos + bn sin
2 n=1 l n=1
l
Here l = 2. Thus
∞ ∞
a0 X nπx X nπx
f (x) = + an cos + bn sin (14.3.5)
2 n=1 2 n=1
2
where
1 2
Z
a0 = f (x)dx
2 −2
1 h −1
Z Z 1 Z 2 i
= 0.dx + k.dx + 0.dx
2 −2 −1 1
k 1
= (x)−1 = k.
2
1 l
Z
nπx
an = f (x) cos dx
l −l l
1 2
Z
2nπx
= f (x) cos dx
2 −2 2
1 1
Z
nπx
= k. cos dx
2 −1 2
Z 1
nπx
= k cos dx
0 2
sin nπx 1 2k nπ
= k nπ 2 = sin .
2
0 nπ 2
0, when n is even
an =
2k
sin nπ ,
nπ 2
when n is odd.
1 l
Z
nπx
bn = f (x) sin dx
l −l l
1 2
Z
2nπx
= f (x) sin dx
2 −2 2
1 1
Z
nπx
= k. sin dx
2 −1 2
nπx
= 0, since sin is odd.
2
k 2k h πx 1 3πx 1 5πx i
∴ (14.3.5) ⇒ f (x) = + cos − cos + cos − .... .
2 π 2 3 2 5 2
Solution. Let
∞ ∞
a0 X nπx X nπx
f (x) = + an cos + bn sin
2 n=1 l n=1
l
Here 2l = 2 ⇒ l = 1. Thus
∞ ∞
a0 X X
f (x) = + an cos nπx + bn sin nπx (14.3.6)
2 n=1 n=1
where
Z 2l
1
a0 = f (x)dx
l 0
2
π−x
Z
= dx
0 2
1 x2 2
= πx −
2 2 0
a0 = π − 1
1 2l
Z
nπx
an = f (x) cos dx
l 0 l
Z 2
= f (x) cos nπxdx
0
π−x
Z 2
= cos nπxdx
0 2
1h sin nπx cos nπx i2
= (π − x) − (−1) − 2 2
2 nπ nπ 0
1 h cos 2nπ 1 i
= − 2 2 + 2 2
2 nπ nπ
1
= (1 − cos 2nπ)
2n2 π2
1
an = (1 − 1)
2n2 π
an = 0.
1
bn =
nπ
∞
π−1 X 1
∴ (14.3.6) ⇒ f (x) = + sin nπx.
2 n=1
nπ
Example 14.3.7. Find the Fourier series with period 3 to represent f (x) = x + x2 in (0, 3).
Solution. Let
∞ ∞
a0 X nπx X nπx
f (x) = + an cos + bn sin
2 n=1 l n=1
l
Here 2l = 3 ⇒ l = 32 . Thus
∞ ∞
a0 X 2nπx X 2nπx
f (x) = + an cos + bn sin (14.3.7)
2 n=1 3 n=1
3
where
1 2l
Z
a0 = f (x)dx
l 0
2 3
Z
= (x + x2 )dx
3 0
2 h x 2 x 3 i3
= + = 9.
3 2 3 0
1 2l
Z
nπx
an = f (x) cos dx
l 0 l
2 3
Z
2nπx
= (x + x2 ) cos dx
3 0 3
2h sin 2nπx − cos 2nπx − sin 2nπx i3
= (x + x ) 2nπ
2 3
− (1 + 2x) 4n2 π2
3
+ (2) 8n3 π3
3
3 3
0
9 27
2h 3 9 i
= −
3 4n2 π2 4n2 π2
9
an = .
n π2
2
1 2l
Z
nπx
bn = f (x) sin dx
l 0 l
2 3
Z
2nπx
= (x + x2 ) sin dx
3 0 3
2h − cos 2nπx − sin 2nπx cos 2nπx i3
= (x + x )
2
2nπ
3
− (1 + 2x) 4n π
2 2
3
+ (2) 8n π
3 3
3
3 3
0
9 27
12
bn = − .
nπ
∞ ∞
9 9 X 1 2nπx 12 X 1 2nπx
∴ (14.3.13) ⇒ f (x) = + 2 cos − sin .
2 π n=1 n2 3 π n=1 n 3
Example 14.3.8. Expand f (x) = e−x as a Fourier series in the interval (−1, 1).
Solution. Let
∞ ∞
a0 X nπx X nπx
f (x) = + an cos + bn sin
2 n=1 l n=1
l
Here l = 1. Thus
∞ ∞
a0 X X
f (x) = + an cos nπx + bn sin nπx (14.3.8)
2 n=1 n=1
where
1 l
Z
a0 = f (x)dx
l −l
Z 1
= e−x dx
−1
= −(e−x )1−1
e1 − e−1
= 2
2
a0 = 2 sinh 1.
1 l
Z
nπx
an = f (x) cos dx
l −l l
Z 1
= f (x) cos nπxdx
−1
Z 1
= e−x cos nπxdx
−1
h e−x i1
= (− cos nπx + nπ sin nπx)
1 + n2 π2 −1
1 h i
an = 2 e−1
(− cos nπ + nπ sin nπ) − e(− cos nπ + nπ sin nπ)
n + π2
1 h i
an = − e−1
(−1) n
+ e(−1)n
n2 + π2
(−1)n
= (e − e−1 )
1 + n2 π2
(−1)n 2 sinh 1
an = .
1 + n2 π2
1 l
Z
nπx
bn = f (x) cos dx
l −l l
Z 1
= e−x sin nπxdx
−1
Z 1
= e−x cos nπxdx
−1
h e−x i1
= (− sin nπx − nπ cos nπx)
1 + n2 π2 −1
1 h −1 i
= e (− sin nπ − nπ cos nπ) − e(sin nπ − nπ cos nπ)
n2 + π2
1 h i
= − nπe−1
(−1)n
+ e(−1) n
nπ
n2 + π2
nπ(−1)n
= (e − e−1 )
1+n π 2 2
2nπ(−1)n sinh 1
bn = .
1 + n2 π2
∞ ∞
X (−1)n 2 sinh 1 X 2nπ(−1)n sinh 1
∴ (14.3.8) ⇒ f (x) = sinh 1 + cos nπx + sin nπx.
n=1
1 + n2 π2 n=1
1 + n2 π2
Solution. Since f (x) = x2 − 2 is an even function, the corresponding Fourier series is given by
∞
a0 X nπx
f (x) = + an cos . (14.3.9)
2 n=1 l
Here l = 2.
∞
a0 X nπx
∴ f (x) = + an cos .
2 n=1 2
where
Z l
2
a0 = f (x)dx
l 0
2 2 2
Z
a0 = (x − 2)dx
2 0
x3 l
= − 2x
3 0
4
a0 = − .
3
and
2 l
Z
nπx
an = f (x) cos dx
l 0 l
2 2 2
Z
nπx
= x cos dx
2 0 2
h sin nπx − cos nπx sin nπx i2
= (x2 − 2) nπ 2 − 2x n π
2 2
2
+ 2 − n π
3 3
2
0
2 4 8
h 16 i
= 0 + 2 2 cos nπ + 0 − (0 + 0 − 0)
nπ
16
an = 2 2 (−1)n .
nπ
∞
2 16 X (−1)n nπx
x2 − 2 = − + 2 cos
3 π n=1 n2 2
2 16 h 1 πx 1 2πx 1 3πx i
= − + 2 − 2 cos + 2 cos − 2 cos + ...
3 π 1 2 2 2 3 2
2 16 h1 πx 1 2πx 1 3πx i
x2 = − − 2 2 cos − 2 cos + 2 cos − ...
3 π 1 2 2 2 3 2
Example 14.3.10. Express f (x) = 3x2 − 2 as Fourier series in the interval (−3, 3).
Solution. Since f (x) = 3x2 − 2 is an even function, the corresponding Fourier series is given by
∞
a0 X nπx
f (x) = + an cos . (14.3.10)
2 n=1 l
Here l = 3.
∞
a0 X nπx
∴ f (x) = + an cos .
2 n=1 3
where
2 l
Z
a0 = f (x)dx
l 0
2 3 2
Z
= (3x − 2)dx
3 0
2 3x3 3
= − 2x
3 3 0
a0 = 14.
and
2 l
Z
nπx
an = f (x) cos dx
l 0 l
2 3 2
Z
nπx
= x cos dx
3 0 3
2h 2 sin nπx − cos nπx sin nπx i2
= (3x − 2) nπ 3
− 6x n π
2 2
3
+ 6 − n π
3 3
3
3 3
0
9 27
108
an = 2 2 (−1)n .
nπ
∞
108 X (−1)n nπx
3x2 − 2 = 7 + cos
π2 n=1 n2 3
108 h 1 πx 1 2πx 1 3πx i
= 7 + 2 − 2 cos + 2 cos − 2 cos + ...
π 1 3 2 3 3 3
108 h1 πx 1 2πx 1 3πx i
x2 = 7 − 2 2 cos − 2 cos + 2 cos − ...
π 1 3 2 3 3 3
Example 14.3.11. If f (x) = |x|, expand f (x) as a Fourier series in the interval (−2, 2).
Solution. Since f (x) = |x| is an even function, the corresponding Fourier series is given by
∞
a0 X nπx
f (x) = + an cos . (14.3.11)
2 n=1 l
Here l = 2.
∞
a0 X nπx
∴ f (x) = + an cos .
2 n=1 2
where
2 l
Z
a0 = f (x)dx
l 0
2 2
Z
= |x|dx
2 0
Z 2
= xdx
0
x2 2
=
2 0
a0 = 2.
and
2 l
Z
nπx
an = f (x) cos dx
l 0 l
2 2
Z
nπx
= |x| cos dx
2 0 2
Z 2
nπx
= x cos dx
0 2
h sin nπx − cos nπx i2
= x nπ 2
−1 n2 π2
2
0
2 4
cos nπ 1
= 0 + n2 π2 − 0 + n2 π2
4 4
4
= 2 2 [(−1)n − 1].
nπ
∞
8 1 X nπx
|x| = 1 − cos
π2
n=1,3,5,...
n2 2
8h1 πx 1 3πx 1 5πx i
|x| = 1 − 2 2 cos + 2 cos + 2 cos + ..... .
π 1 2 3 2 5 2
Example 14.3.12. Express f (x) = 1 + sin x as a Fourier series in the interval (−1, 1).
∞ ∞
a0 X X
f (x) = + an cos nπx + bn sin nπx (14.3.12)
2 n=1 n=1
where
Z l
a0 = f (x)dx
−l
Z 1
= (1 + sin x)dx
−1
Z 1 Z 1
= dx + sin xdx
−1 −1
= (x)1−1 +0
a0 = 2.
1 l
Z
nπx
an = f (x) cos dx
l −l l
Z 1
= (1 + sin x) cos nπxdx
−1
1 l
Z
nπx
bn = f (x) sin dx
l −l l
Z 1
= (1 + sin x) sin nπxdx
−1
Z 1 Z 1
= sin nπxdx + sin x sin nπxdx
−1 −1
Z 1
bn = 0 + 2 sin x sin nπxdx
0
Z 1
bn = [cos(nπ − 1)x − cos(nπ + 1)x]dx
0
h sin(nπ − 1)x sin(nπ + 1)x i1
= −
nπ − 1 nπ + 1 0
sin(nπ − 1) sin(nπ + 1)
= −
nπ − 1 nπ + 1
1 1
= [sin nπ. cos 1 − cos nπ. sin 1] − [sin nπ. cos 1 + cos nπ. sin 1]
nπ − 1 nπ + 1
1 1
= (0 − (−1)n sin 1) − ((−1)n sin 1)
nπ − 1 nπ + 1
1 1
= −(−1)n sin 1 +
nπ − 1 nπ + 1
2nπ sin 1
bn = (−1)n+1 .
n2 π2 − 1
∞
X (−1)n+1
∴ (14.3.12) ⇒ 1 + sin x = 1 + 2π sin 1 sin nπx.
n=1
n2 π2 − 1
Example 14.3.13. Find the Fourier series with period 3 to represent f (x) = 2x− x2 in 0 < x < 3.
Solution. Let
∞ ∞
a0 X nπx X nπx
f (x) = + an cos + bn sin
2 n=1 l n=1
l
Here 2l = 3 ⇒ l = 32 . Thus
∞ ∞
a0 X 2nπx X 2nπx
f (x) = + an cos + bn sin (14.3.13)
2 n=1 3 n=1
3
where
1 2l
Z
a0 = f (x)dx
l 0
2 3
Z
= (2x − x2 )dx
3 0
2 h 2x2 x3 i3
= −
3 2 3 0
2
a0 = (9 − 9)
3
a0 = 0.
1 2l
Z
nπx
an = f (x) cos dx
l 0 l
2 3
Z
2nπx
= (2x − x2 ) cos dx
3 0 3
2 h −36 18 i
= −
3 4n2 π2 4n2 π2
9
an = − 2 2.
nπ
1 2l
Z
nπx
bn = f (x) sin dx
l 0 l
2 3
Z
2nπx
= (2x − x2 ) sin dx
3 0 3
2h − cos 2nπx − sin 2nπx cos 2nπx i3
= 2
(2x − x ) 2nπ
3
− (2 − 2x) 4n2 π2
3
+ (−2) 3
8n3 π3
3 3
0
9 27
3
= − .
nπ
∞ ∞
9 X 1 2nπx 3 X 1 2nπx
∴ (14.3.13) ⇒ f (x) = − cos + sin .
π n=1 n
2 2 3 π n=1 n 3
Example 14.3.14. Find the Fourier series for f (x) = 2lx − x2 in 0 < x < 2l and hence deduce
π2
that 1 − 1
22
+ 1
32
− 1
42
+ .... = 2
.
Solution. The Fourier series for f (x) = 2lx − x2 in the interval (0, 2l) is given by
∞ ∞
a0 X nπx X nπx
f (x) = + an cos + bn sin
2 n=1 l n=1
l
where
1 2l
Z
a0 = f (x)dx
l 0
1 2l
Z
= (2lx − x2 )dx
l 0
1 h 2lx2 x3 i2l
= −
l 2 3 0
1 3 8 3
a0 = 4l − l
l 3
a0 = 4l2 .
1 2l
Z
nπx
an = f (x) cos dx
l 0 l
1 2l
Z
nπx
= (2lx − x2 ) cos dx
l 0 l
1h sin nπx − cos nπx − sin nπx i2l
= (2lx − x2 ) nπ l − (2l − 2x) n π
2 2
l
+ (−2) n π
3 3
l
l l
0
l2 l3
1 h −2l3 2l3 i
= −
l n2 π2 n2 π2
4l2
an = − 2 2.
nπ
1 2l
Z
nπx
bn = f (x) sin dx
l 0 l
1 2l
Z
nπx
= (2lx − x2 ) sin dx
l 0 2
1h − cos nπx − sin nπx cos nπx i2l
= (2lx − x2 ) nπ
l
− (2l − 2x) n π
2 2
l
+ (−2) n π
3 3
l
l l
0
l2 l3
1 h 2l3 2l3 i
= 0+0− 3 3 − 0+0− 3 3
l nπ nπ
= 0.
∞
4l2 X 1 nπx
∴ f (x) = 2l − 2
2
cos .
π n=1 n 2 l
∞
4l2 X 1
0 = 2l2 −
π2 n=1 n2
∞
X 1 π2
= .
n=1
n2 2
∞
X nπx
f (x) = bn sin ,
n=1
l
where
2 l
Z
nπx
bn = f (x) sin dx
l 0 l
2 l
Z
nπx
= a sin dx
l 0 l
nπx
2a cos l l
= − nπ
l l
0
2a
= − (cos nπ − cos 0)
nπ
2a
= [1 − (−1)n ].
nπ
0, when n is even
bn =
4a
, when n is odd
nπ
.
∞
4a X 1 nπx 4a πx 1 3πx 1 5πx
∴ f (x) = sin = sin + sin + sin + ..... .
π n=1,3,5,.. n l π l 3 l 5 l
and f (x) = 3
2
when x = 0, 1, 3 and f (x + 3) = f (x) for all x.
Solution. Let
∞ ∞
a0 X nπx X nπx
f (x) = + an cos + bn sin
2 n=1 l n=1
l
Here 2l = 3 ⇒ l = 32 . Thus
∞ ∞
a0 X 2nπx X 2nπx
f (x) = + an cos + bn sin (14.3.14)
2 n=1 3 n=1
3
where
1 2l
Z
a0 = f (x)dx
l 0
2 3
Z
= f (x)dx
3 0
2h 1
Z Z 3 i
= 1.dx + 2.dx
3 0 1
2h 1 i
= (x)0 + 2(x)31
3
2
= (1 + 4)
3
10
a0 = .
3
1 2l
Z
nπx
an = f (x) cos dx
l 0 l
2h 1
Z Z 3
2nπx 2nπx
= 1. cos dx + 2. cos dx
3 0 3 1 3
2nπx sin 2nπx
2 h sin 3 1 3 i
= 2nπ
+ 2 2nπ
3
3 3
0
3
1
1 2nπ 2 2nπ
= sin + 0 − sin
nπ 3 nπ 3
1 2nπ
an = − sin .
nπ 3
1 2l
Z
nπx
bn = f (x) sin dx
l 0 l
2h 1
Z Z 3
2nπx 2nπx
= 1. sin dx + 2. sin dx
3 0 3 1 3
2nπx − cos 2nπx
2 h − cos 3 1 3 i
= 2nπ
+ 2 2nπ
3
3 3
0
3
1
2h 3 2nπ 3 2nπ i
= − cos −1 − cos 2nπ − cos
nπ 2nπ 3 nπ 3
1 2nπ 1 2 2 2nπ
= − cos + − + cos
nπ 3 nπ nπ nπ 3
1 2nπ 2 2nπ
bn = sin + 0 − sin
nπ 3 nπ 3
1 2nπ
bn = cos −1 .
nπ 3
∞ ∞
5 X −1 2nπ 2nπx X 1 2nπ 2nπx
∴ (14.3.14) ⇒ f (x) = + sin cos + cos − 1 sin
3 n=1 nπ 3 3 n=1
nπ 3 3
∞
5 1 X 1 h 2nπ 2nπx 2nπ 2nπx 2nπx i
= − sin cos + cos sin + sin
3 π n=1 n 3 3 3 3 3
∞
5 1 X 1h 2nπx 2nπ 2nπx i
= − sin − + sin
3 π n=1 n 3 3 3
∞ 2nπ(1 − x)
5 1 X 1h 2nπx i
= − sin + sin .
3 π n=1 n 3 3
Solution. Here l = 2.
∞ ∞
a0 X nπx X nπx
∴ f (x) = + an cos + bn sin .
2 n=1 2 n=1
2
where
1 2
Z
a0 = f (x)dx
2 −2
1h 0
Z Z 2 i
= 2dx + xdx
2 −2 0
1h x2 2 i
= 2(x)0−2 +
2 2 0
1
= (4 + 2)
2
a0 = 3.
and
Z 2
1 nπx
an = f (x) cos dx
2 −2 2
1h 0
Z Z 2
nπx nπx i
an = 2. cos dx + x. cos
2 −2 2 0 2
nπx nπx cos nπx i2 i
1 h sin 2 0 h sin
= 2 nπ + x nπ 2 − 1. n2 π22
2 2
−2
2
0
4
1h 4 4 i
= cos nπ −
2 n2 π2 n62π2
2
an = 2 2 [(−1)n − 1].
nπ
0, when n is even
an =
− n24π2
when n is odd
1 2
Z
nπx
bn = f (x) sin dx
2 −2 2
1h 0
Z Z 2
nπx nπx i
= 2. sin dx + x. sin
2 −2 2 0 2
nπx nπx − sin nπx
1 h − cos 2 0 h cos 2 i2 i
= 2 nπ + x − nπ − 1. n π
2 2
2
2 2
−2
2
0
4
1 h −4 4 i 1h 4 4 i
= + cos nπ + − cos nπ + 2 2 sin nπ
2 nπ nπ 2 nπ nπ
1 4
= −
2 nπ
2
bn = − .
nπ
∞ ∞
3 4 X 1 nπx 2 X 1 nπx
f (x) = − cos − sin .
2 π2 n=1,3,5,...
n 2 2 π n=1
n 2
∞ ∞
a0 X nπx a0 X
f (x) = + an cos = + an cos nπx
2 n=1 l 2 n=1
where
Z l
2
a0 = f (x)dx
l 0
Z 1
= 2 f (x)dx
0
Z l
2 nπx
an = f (x) cos dx
l 0 l
Z 1
= 2 f (x) cos nπxdx
0
Z 1
1
= 2 − x cos nπxdx
0 2
h 1 sin nπx cos nπx i1
= 2 −x − (−1) − 2 2
2 nπ nπ 0
h cos nπ 1 i
= 2 0− 2 2 − 0− 2 2
nπ nπ
2
= (1 − cos nπ)
n2 π2
2
an = [1 − (−1)n ]
n π2
2
0, if n is even
an =
n24π2 ,
if n is odd
∞ ∞
4 X 1 4 X 1
∴ f (x) = cos nπx = cos(2n − 1)πx.
π2 n=1,3,5,.. n2 π2 n=1 (2n − 1)2
if − l < x < 0
−1,
f (x) =
if x = 0
0
if 0 < x < 1
1,
π
Hence deduce that 4
= 1 − 13 + 51 − 17 + 19 − ....
∞ ∞
a0 X nπx X nπx
f (x) = + an cos + bn sin
2 n=1 l n=1
l
where
1 l
Z
a0 = f (x)dx
l −l
1h 0
Z Z 1 i
= (−1) f (x)dx + 1.dx
l −l 0
1h i
= (−x)0−l + (x)l0
l
1
= [0 + l + l − 0]
l
a0 = 0.
1 l
Z
nπx
an = f (x) cos dx
l −l l
1h 0
Z Z l
nπx nπx i
= (−1) cos dx + (1) cos dx
l −l l 0 l
nπx nπx
1 h sin l 0 sin l l i
= − nπ + nπ
l l
−l
l
0
1 1
= − (0 − 0) + (0 − 0)
nπ nπ
an = 0.
1 l
Z
nπx
bn = f (x) sin dx
l −l l
1h 0
Z Z l
nπx nπx i
= (−1) sin dx + (1) sin dx
l −l l 0 l
1 1
= − (1 − cos nπ) + (cos nπ − 1)
nπ nπ
2
bn = [1 − (−1)n ].
nπ
0, if n is even
bn =
4
,
nπ if n is odd
∞
4 X 1 nπx
∴ f (x) = sin .
π n=1,3,5,.. n l
Deduction. Putting x = l
2
in the above series, we get
∞
4 X 1 l nπ
f =
sin
2 π n=1,3,5,... n 2
4 π 1 3π 1 5π
1 = sin + sin + sin + ...
π 2 3 2 5 2
4 1 1 1
1 = 1 − + − + ....
π 3 5 7
1 1 1 π
∴ 1 − + − + .... = .
3 5 7 4
14.4 Exercises
1. Find the Fourier series expansion for the function f (x) = x − x2 in (−1, 1).
2. Find the Fourier series of the following function:
−2 < x < 0
0,
f (x) =
0<x<2
1,
(i)
0<x<l
0,
f (x) =
l < x < 2l
a,
(ii)
−5 < x < 0
0,
f (x) =
0<x<5
3,
(i)
0, −2 < x < −1
f (x) =
k, −1 < x < 1
0, 1 < x < 2
(ii)
−2 ≤ x <≤ −1
0,
1 + x, −1 ≤ x ≤ 0
f (x) =
1 − x, 0 ≤ x ≤ 1
0, 1≤x≤2
Answers.
1. x − x2 = − 31 + π42 cos πx
12 − cos 2πx
22 + cos 3πx
32 − .... + 2
π
sin πx − 1
2
sin 2πx + 1
3
sin 3πx − .....
πx
2. f (x) = 2 + π sin 2 + 3 sin 2 + 5 sin 2 + ..... .
1 2 1 3πx 1 5πx
πx
3. (i) f (x) = a2 − 2a
P∞ 1
π n=1 2n−1 sin(2n − 1) l
Lesson 15
• find the half range Fourier cosine series of functions in [0, l].
15.1 Introduction
S
o far we have considered the Fourier series expansion of a function defined in an interval
(c, c + 2l) of length 2l. In various engineering problems it is required to obtain a Fourier
series expansion of a function defined in half of the above interval say (0, l) of length l. Such
expansions are known as half - range expansion or half - range Fourier Series. As it is
immaterial whatever the function may be outside the range 0 < x < l, we extend the function to
cover the range −l < x < l so that the new function may be even or odd. The Fourier expansion of
such a function consists of cosine or sine terms only.The expansion containing only cosine terms
is known as half - range Fourier cosine series and the expansion containing only sine terms is
known as half - range Fourier sine series.
Cosine Series. If it is required to expand f (x) in the interval (0, l) then we extend the function
reflecting in the y - axis, so that f (−x) = f (x). We can define a new function g(x) such that
Clearly g(x) is an even function in (−l, l) and it is identical with f (x) in (0, l). We can now
construct the Fourier series for this even function g(x) in (−l, l). We know that the expansion will
be of the form
∞
a0 X nπx
g(x) = + an cos
2 n=1 l
where
2 l
Z
a0 = g(x)dx
l 0
2 l
Z
nπx
an = g(x) cos dx.
l 0 l
But g(x) is identical with f (x) in (0, l). ∴ in the interval (0, l), the Fourier cosine series is
∞
a0 X nπx
f (x) = + an cos
2 n=1 l
where
2 l
Z
a0 = f (x)dx
l 0
2 l
Z
nπx
an = f (x) cos dx.
l 0 l
Sine Series. If it is required to expand f (x) in the interval (0, l) then we extend the function
reflecting it in the origin, so that f (−x) = f (x). We can define a new function g(x) such that
Clearly g(x) is an odd function in (−l, l) and it is identical with f (x) in (0, l). We can now
construct the Fourier series for this odd function g(x) in (−l, l). We know that the expansion will
be of the form
∞
X nπx
g(x) = an sin
n=1
l
where
Z l
2 nπx
bn = g(x) sin dx.
l 0 l
∞
X nπx
f (x) = an sin
n=1
l
where
Z l
2 nπx
bn = f (x) sin dx.
l 0 l
Example 15.2.1. Find the half - range sine series of f (x) = 1 in [0, l].
∞
X nπx
f (x) = bn sin
n=1
l
where
2 l
Z
nπx
bn = f (x) sin dx
l 0 l
2 l
Z
nπx
= 1. sin dx
l 0 l
nπx
2 cos l l
= − nπ
l l
0
2 l
= . − cos nπ + 1
l nπ
2
= [(−1)n+1 + 1]
nπ
0, when n is even
∴ bn =
4
, when n is odd
nπ
∞
X 4 nπx
f (x) = sin
n=1,3,5,..
nπ l
4 πx 1 3πx 1 5πx
1 = sin + sin + sin + ... .
π l 3 l 5 l
Example 15.2.2. Obtain the half - range cosine and sine series for f (x) = x in [0, l].
∞
a0 X nπx
f (x) = + an cos
2 n=1 l
where
l l
2 x 2 l
Z Z
2 2
a0 = f (x)dx = xdx = = l.
l 0 l 0 l 2 0
and
2 l
Z
nπx
an = f (x) cos dx
l 0 l
2 l
Z
nπx
= x cos dx
l 0 l
nπx cos nπx
2 h sin l il
= x nπ − 1 n2 π2 l
l l
0
l2
2 h l2 l2 i
= 0 + 2 2 (−1)n − 0 + 2 2
l nπ nπ
2l
an = [(−1)n − 1]
n2 π2
0, when n is even
∴ an =
− n24lπ2 ,
when n is odd
∞
l 4l X 1 nπx
f (x) = − 2 cos
2 π n=1,3,5,.. n 2 l
l 4l πx 1 3πx 1 5πx
x = − 2 cos + 2 cos + 2 cos + ... .
2 π l 3 l 5 l
Note. Expand f (x) = x as a half - range cosine series in 0 < x < 2. Putting l = 2 in the above
series, we get
8 πx 1 3πx 1 5πx
x=1− cos + cos + cos + ... .
π2 2 32 2 52 2
Sine Series.
∞
X nπx
f (x) = bn sin
n=1
l
where
2 l
Z
nπx
bn = f (x) sin dx
l 0 l
2 l
Z
nπx
= x sin dx
l 0 l
nπx − sin nπx
2 h − cos l il
= x nπ − 1 n2 π2
l
l l
0
l2
2 h l2 i
= − (−1)n + 0 − (0 + 0)
l nπ
2l
bn = −
nπ(−1)n
2l
bn = (−1)n+1 .
nπ
∞
2l X (−1)n+1 nπx
f (x) = sin
nπ n=1 n l
2l πx 1 2πx 1 3πx
x = sin − sin + cos − ... .
π l 2 l 3 l
4 πx 1 2πx 1 3πx
x= sin − sin + cos − ... .
π 2 2 2 3 2
Example 15.2.3. Find the half range sine series expansion of f (x) = x2 in [0, 4].
∞
X nπx
f (x) = bn sin
n=1
l
where Here l = 4.
∞
X nπx
∴ f (x) = bn sin
n=1
4
where
2 l
Z
nπx
bn = f (x) sin dx
l 0 l
2 4 2
Z
nπx
= x sin dx
4 0 4
nπx − sin nπx cos nπx
1 h 2 − cos 4 i4
= x nπ − 2x n2 π2
4
+ 2 n3 π3
4
2 4
0
16 64
1h 4 32 2.64 128 i
= − (16) cos nπ + 2 2 (4) sin nπ + cos nπ −
2 nπ nπ n63π3 n3 π3
n+1 n
1 (−1) 64 (−1) 128 128
h i
= + − 3 3
2 nπ n3 π3 nπ
h 2 n+1 i
(−1)
bn = 32 3 3 [(−1)n − 1] +
nπ nπ
− nπ ,
32
when n is even
∴ bn =
4
+ 1
,
32 −
n3 π2 nπ
when n is odd
∞ ∞
32 X 1 nπx X 4 1 nπx
x2 = − sin + 32 − 3 3+ sin .
π n=2,4,6,.. n 4 n=1.3.5,..
n π nπ 4
Example 15.2.4. Find the half - range cosine series expansion of f (x) = x − x2 in 0 < x < l.
∞
a0 X nπx
f (x) = + an cos
2 n=1 l
Here l = 1.
∞
a0 X
∴ f (x) = + an cos nπx.
2 n=1
where
2 1
Z
a0 = f (x)dx
1 0
2 1
Z
= (x − x2 )dx
1 0
2 x2 x3 1
= −
1 2 3 0
1 1
= 2 −
2 3
1
a0 = .
3
and
2 l
Z
nπx
an = f (x) cos dx
l 0 l
2 1
Z
= (x − x2 ) cos nπxdx
1 0
h sin nπx − cos nπx − sin nπx i1
= 2 (x − x2 ) − (1 − 2x) − 2
nπ n2 π2 n3 π3 0
h 1 1 i
= 2 − 2 2 cos nπ − 2 2
nπ nπ
2
= − 2 2 (1 + cos nπ)
nπ
2
an = − 2 2 [1 + (−1)n ]
nπ
0, when n is odd
∴ an =
− n24π2 ,
when n is even
∞
1 4 X 1
x − x2 = − 2 cos nπx,
6 π n=2,4,6,.. n2
Example 15.2.5. Find half - range Fourier sine series for f (x) = ax + b, in 0 < x < l.
∞
X nπx
f (x) = bn sin .
n=1
l
Here l = 1
∞
X
∴ f (x) = bn sin πx
n=1
where
2 l
Z
nπx
bn = f (x) sin dx
l 0 l
2 1
Z
= (ax + b) sin nπxdx
1 0
h − cos nπx − sin nπx i1
= 2 (ax + b) −a
nπ n2 π2 0
h 1 a bi
= 2 − (a + b) cos nπ + 2 2 sin nπ +
nπ nπ nπ
2
= [(−1)n+1 (a + b) + b]
nπ
− nπ ,
2a
when n is even
∴ bn =
2
(a + 2b),
nπ when n is odd
2 2a 2 2a
ax + b = (a + 2b) sin πx − sin 2πx + (a + 2b) sin 3πx − sin 4πx + .....
π 2π 3π 4π
Example 15.2.6. Find the half - range cosine series for f (x) = x(2 − x) in 0 ≤ x ≤ 2 and hence
find sum of the series 1
12
− 1
22
+ 1
32
− 1
42
+ ...
∞
a0 X nπx
f (x) = + an cos
2 n=1 l
Here l = 2.
∞
a0 X nπx
∴ f (x) = + an cos .
2 n=1 2
where
2 1
Z
a0 = f (x)dx
1 0
2 2
Z
a0 = (2x − x2 )dx
2 0
2x2 x3 2
a0 = −
2 3 0
4
a0 = .
3
and
2 l
Z
nπx
an = f (x) cos dx
l 0 l
2 2
Z
nπx
= (2x − x2 ) cos dx
2 0 2
h sin nπx − cos nπx − sin nπx i2
= (2x − x ) nπ
2 2
− (2 − 2x) n2 π2
2
− 2 n3 π3
2
0
2 4 8
h 1 1 i
= 2 − 2 2 cos nπ − 2 2
nπ nπ
8
= − 2 2 (1 + cos nπ)
nπ
8
an = − 2 2 [1 + (−1)n ]
nπ
0, when n is odd
∴ an =
2 π2 ,
− n16
when n is even
2 4 1 1
2−1 = − 2 cos π + 2 cos 2π + 2 cos 3π + ......
3 π 2 3
2 4 1 1
1− = − 2 − 1 + 2 − 2 + ......
3 π 2 3
1 4 1 1
= 2 1 − 2 + 2 − ......
3 π 2 3
1 1 1 π2
1− + − + ...... = .
22 32 52 12
πx
Example 15.2.7. Find the half - range cosine series expansion of f (x) = sin l
in the interval
0 < x < l.
∞
a0 X nπx
f (x) = + an cos
2 n=1 l
where
2 1
Z
a0 = f (x)dx
1 0
2 l
Z πx
= sin dx
l 0 l
2
a0 = − (cos π − 1)
π
4
a0 = .
π
and
2 l
Z
nπx
an = f (x) cos dx
l 0 l
2 2 πx
Z
nπx
= sin cos dx
l 0 l 2
1 lh πx πx
Z
= sin(n + 1) − sin(n − 1) Big]dx
l 0 l l
πx πx i
1 h cos(n + 1) cos(n − 1) l l
= − π
l
+ π
l (n + 1) l (n − 1) l 0
1 h cos(n + 1)π cos(n − 1)π i 1 1 i
= − + − − +
π n+1 n−1 n+1 n−1
1 h (−1)n+1 (−1)n−1 1 1 i
an = − + + − , n,1
π n+1 n−1 n+1 n−1
0, when n is odd, n , 1
∴ an =
4
,
− π(n+1)(n−1)
when n is even, n , 1
If n = 1,
1 l πx πx
Z
a1 = 2 sin cos dx
l 0 l l
Z l
1 2πx
= sin dx
l 0 l
1 l h 2πx il
= . − cos
l 2π l 0
1
= − (cos 2π − cos 0)
2π
a1 = 0.
Example 15.2.8. Find the half range sine series of e x in the interval 0 < x < 1.
∞
X
ex = bn sin nπx.
n=1
where
Z 1
bn = 2 e x sin nπxdx
0
h ex i1
= 2 2 2 (sin nπx − nπ cos nπx)
n π +1 0
h e 1 i
= 2 2 2 (0 − nπ cos nπ) − 2 2 (0 − nπ)
n π +1 n π +1
h 1 i
= 2 2 2 (nπ − nπ cos nπ)
n π +1
2nπ
= 2 2 [1 − e(−1)n ].
n π +1
∞
X
cos πx = bn sin nπx.
n=1
where
Z 1
bn = 2 cos πx sin nπxdx
0
Z 1
= [sin(n + 1)πx + sin(n − 1)πx]dx
0
h cos(n + 1)πx cos(n − 1)πx i1
= − − , n,1
(n + 1)π (n − 1)π 0
1 h cos(n + 1)π cos(n − 1)π 1 1 i
= − + − + , n,1
π n+1 n−1 n+1 n−1
1h 1 1 1 1 i
= − (−1)n+1 + − + , n,1
π n+1 n−1 n+1 n−1
1 1 1
= − + [(−1)n+1 − 1], n , 1
π n+1 n−1
1 2n
= − . 2 [(−1)n+1 − 1], n , 1.
π n −1
0, when n is odd
∴ bn =
2 −1) ,
π(n4n
when n is even
If n = 1, then
Z 1
b1 = 2 sin πx cos πxdx
0
Z 1
= sin 2πxdx
0
1
= − (cos 2π − cos 0)
2π
b1 = 0.
∞ ∞ ∞
X 4n 4 X 2n 8X n
∴ cos πx = sin nπx = sin 2nπx = sin 2πx.
2,4,6,..
π(n2 − 1) π n=1 4n2 − 1 π n=1 4n2 − 1
Example 15.2.10. Find the half - range cosine series for the function f (x) = (x − 1)2 in the
π2
interval 0 < x < 1. Hence show that 1
12
+ 1
22
+ 1
32
+ ... = 6
.
Solution. Let
∞
a0 X
f (x) = + an cos nπx,
2 n=1
where
Z 1
a0 = 2 f (x)dx
0
Z 1
= 2 (x − 1)2 dx
0
h (x − 1)3 i1
= 2
3 0
2
a0 = .
3
and
Z 1
an = 2 f (x) cos nπxdx
0
Z 1
= 2 (x − 1)2 cos nπxdx
0
h sin nπx cos nπx − sin nπx i1
= 2 (x − 1)2 − 2(x − 1) − 2 2 + 2
nπ nπ n3 π3 0
h 2 i
= 2 (0 + 0 − 0) − 0 − 2 2 − 0
nπ
4
= 2 2.
nπ
∞
1 4 X 1
∴ (x − 1)2 = + 2 cos nπx.
3 π n=1 n2
π2
Deduce that 1
12
+ 1
32
+ 1
52
+ ..... = 8
.
Solution. Let
∞
a0 X nπx
f (x) = + an cos ,
2 n=1 L
where
Z L
2
a0 = f (x)dx
L 0
L
Z Z L
2h 2
= kxdx + k(L − x)dx
L 0 L
2
2k h x2 L2 x2 L i
= + Lx −
L 2 0 2 L2
2k h L2 2 L2 L2 L2 i
= + L − − −
L 8 2 2 8
2
2k 2L
= + L2 − L2
L 8
kL
=
2
and
Z L Z L
2h 2 nπx nπx i
an = kx cos dx + k(L − x) cos dx
L 0 L L
2
L
2kL h nπ i
= 2 2 2 cos − 1 − (−1)n
nπ 2
0, when n is odd
∴ an =
2 π2 cos 2 − 1 ,
n4kL
nπ
when n is even
∞
kL 4kL X 1 nπ nπx
f (x) = + 2 cos − 1 cos
4 π 2,4,6,.. n 2 2 L
kL 8kL h 1 2πx 1 6πx 1 10πx i
f (x) = − 2 2 cos + 2 cos + 2 cos + .... .
4 π 2 L 6 L 10 L
Deduction.
kL 8kL 1 1 1
f (0) =
− 2 2 + 2 + 2 + ....
4 π 2 6 10
kL 8kL 1 1 1 1
0 = − 2 . 2 2 + 2 + 2 + ....
4 π 2 1 3 5
kL 2kL 1 1 1
= + 2 + 2 + ....
4 π 1
2 2 3 5
1 1 1 π2
+ + + .... = .
12 32 52 8
Example 15.2.12. Find the Fourier sine series for f (x) = 2x − x2 , in 0 < x < 3 and
f (x + 3) = f (x).
∞
X nπx
f (x) = bn sin ,
n=1
3
where
2 3
Z
nπx
bn = f (x) sin dx
3 0 3
2 3
Z
nπx
= (2x − x2 ) sin dx
3 0 3
2h cos nπx sin nπx cos nπx i3
= (2x − x2 ) − nπ 3 − (2 − 2x) − n2 π23 + (−2) n3 π23
3 3
0
9 27
2h 3 54 54 i
= − 3. cos nπ + 0 − 3 3 cos nπ − 0 − 0 − 3 3
3 nπ nπ nπ
2h 9 54 i
= (−1)n + 3 3 [1 − (−1)n ]
3 nπ nπ
h (−1)n 6 i
= 6 + 3 3 [1 − (−1)n ]
nπ nπ
nπ ,
6
if n is even
∴ bn =
1
+ 12
,
6 −
nπ n3 π3
if n is odd
15.3 Exercises
1. Find the half - range cosine series for the function f (x) = x in 0, l.
2. Obtain the half - range cosine and sine series for x in (0, 2).
3. Find the half range cosine series for the function f (x) = (x − 1)2 in the interval 0 < x < 1.
Hence deduce that
(i)1 + 212 + 312 + .... = π6 ;
2
π2
(ii) ∞n=1 (2n−1)2 = 12 + 32 + + ... = .
P 1 1 1 1
52 8
Answers.
h i
1. x = l
2
− 4l
π2
cos 4πl + 1
32
cos 3πx
cos 5πxl
l
+
+ · ·1
·
52
πx
h i
2. x = l − π2 cos 2 + 32 cos 2 + 52 cos 2 + · · ·
8 1 3πx 1 5πx
and x = π4 sin πx
h i
2
− 12 sin 2πx
2
+ 13 cos 3πx
2
+ ···
h i
3. (x − 1)2 = 13 + π42 cos πx + 212 cos 2πx + 312 cos 3πx + · · ·
h i
4. x − x2 = π83 113 sin πx + 313 sin 3πx + 513 sin 5πx + · · ·
(−1)n
2 P
h i
8. x2 = 2Lπ ∞ 2 n
n=1 n3 π2 [(−1) − 1] − n sin nπx
L
.
9. 2x − 1 = − π8 cos12πx + cos33πx
h i
2 + cos55πx
2 + ··· .
10. f (x) = π1 − π42 sin πx + 3π 1
+ 324π2 sin 3πx + 5π 1
− 524π2 sin 5πx + · · ·
Lesson 16
FOURIER TRANSFORMS
Learning Objectives
Upon completion of this lesson, students will be able to
• find the Fourier transform of given functions by using the definition of Fourier transform
and inverse Fourier transform.
16.1 Introduction
A
transformation is a mathematical device which converts or changes one function into
another function. For example, differentiation and integration are transformations.
Fourier transforms are used in solving partial differential equations with boundary conditions.
In this chapter, some of the well known integral transforms will be introduced and their
properties will be studied which will be used in the solution of partial differential equations. Also
we discuss the application of finite and infinite Fourier integral transforms which are mathematical
devices from which we obtain the solutions of boundary value problems related to engineering.
For example, conduction of heat, free and forced vibrations of membrane, transverse vibrations of
where k(p, t) is called the kernel of the integral transform and is known function of p and t. The
function f (t) is called the inverse transform of f (p).
Z ∞
∴ f (t) = L [ f (t)] = e−pt f (t)dt.
0
(ii) If k(p, t) = eipt , a → ∞ and b → ∞ then we have the Infinite Fourier transform of f (t)
Z ∞
∴ f (p) = F [ f (t)] = eipt f (t)dt
−∞
Other special transforms arise when the kernel is a sine or cosine function or a Bessel’s function.
These lead to Fourier sine or cosine transforms and the Hankel transform respectively.
First we shall prove the Fourier integral theorem. Before proving this theorem we must know the
well known Dirichlet’s conditions which play an important role in the study of Fourier transforms
and covers many engineering applications.
A function f (x) is said to satisfy Dirichlet’s conditions in the interval (a, b), if
(i) f (x) is defined and is single valued function except possibly at a finite number of points in the
interval (a, b) and
(ii) f (x) and f 0 (x) are piecewise continuous in the interval (a, b).
Proof. Let f (x) be a function satisfying the Dirichlet’s conditions in every interval (−l, l) and
defined as
1h i
f (x) = f (x + 0) + f (x − 0)
2
at every point of discontinuity.
We know that the Fourier series of f (x) in (−l, l) is given by
∞
a0 X h nπx nπx i
f (x) = + an cos + bn sin (16.2.1)
2 n=1 l l
where
1 l
Z
a0 = f (t)dt
l −l
1 l
Z
nπt
an = f (t) cos dt
l −l l
1 l
Z
nπt
bn = f (t) sin dt
l −l l
Z l ∞ Z l ∞ Z l
1 1X nπt nπx 1X nπt nπx
f (x) = f (t)dt + f (t) cos cos dt + f (t) sin sin dt
2l −l l n=1 −l l l l n=1 −l l l
Z l ∞ Z l
1 1X nπ(t − x)
f (x) = f (t)dt + f (t) cos dt. (16.2.2)
2l −l l n=1 −l l
R∞
Let −∞
| f (x)|dx converges, i,e., it has a finite value. Now assuming that the limit as t → ∞ in
(16.2.2), we have the first term
h1 Z l i
lim f (t)dt = 0,
t→∞ 2l −l
since
1 Z l 1 Z l
f (t)dt ≤ | f (t)|dt.
2l −l 2l −l
π
Now we put l
= δp in the second term of (16.2.2), then it takes the form
∞ Z l
1 Xh i
f (t) cos[nδp(t − x)]dt δp (16.2.3)
π n=1 −l
since
∞
X Z ∞
lim φ(nδp)δp = φ(p)d p.
δp→0 0
n=1
Remembering cos(A − B) expansion, expanding the cos p(t − x), the Fourier integral of f (x)
given by (16.2.4) may be written as
Z ∞ Z ∞
1
f (x) = f (t)[cos(pt) cos(px) + sin(pt) sin(px)]dtd p.
π 0 −∞
Z ∞ Z ∞ Z ∞ Z ∞
1 1
f (x) = cos(px) f (t) cos(pt)dtd p + sin(px) f (t) sin(pt)dtd p (16.3.1)
π 0 −∞ π 0 −∞
When f (t) is an odd function, f (t). cos pt is an odd function and f (t) sin pt is an even function.
So, the first integral in the right side of (16.3.1) becomes zero. Therefore, we get
Z ∞ Z ∞
2
f (x) = sin px f (t) sin ptdtd p (16.3.2)
π 0 0
which is known as Fourier sine integral. In the same way, when f (t) is an even function, the
second integral in the right side of (16.3.1) becomes zero. Therefore, we get
Z ∞ Z ∞
2
f (x) = cos px f (t) cos ptdtd p (16.3.3)
π 0 0
Now write Z ∞
θ1 (p) = cos pt f (t)dt
−∞
and Z ∞
θ2 (p) = sin pt f (t)dt
−∞
Therefore
1 ∞ 1 ∞
Z Z
f (x) = θ1 (p) cos pxd p + θ2 (p) sin pxd p
π 0 π 0
1 ∞
Z i
= [θ1 (p) cos px + θ2 (p) sin px d p
π 0
where Z ∞
θ2 (p) = 2 sin pt f (t)dt.
0
where Z ∞
θ1 (p) = 2 cos pt f (t)dt.
0
The complex form of Fourier integral of any function f (x) is in the form
Z ∞ Z ∞ Z ∞ Z ∞
1 1
f (x) = eip(t−x)
f (t)dtd p = e −ipx
dp eipt f (t)dt
2π −∞ −∞ 2π −∞ −∞
Now, we write Z ∞
F(p) = eipt f (t)dt
−∞
Therefore Z ∞
1
f (x) = F(p)e−ipx d p.
2π −∞
Here F(p) is called the Fourier transform of f (x) and the function f (x) is called the inverse
Fourier transform of F(p). Thus we obtain the definition of Fourier transform as follows:
1. The Infinite Fourier Transform of f (x) .
Definition 16.6.1. Let f (x) be a function defined on (−∞, ∞) and be piecewise continuous in
each finite partial interval and absolutely integrable in (−∞, ∞). Then the Fourier transform of
f (x) denoted by F [ f (x)] = f (p) is defined as
Z ∞
1
F [ f (x)] = f (p) = √ eipx f (x)dx.
2π −∞
(ii) Z ∞
1
f (x) = √ f (p)eipx d p.
2π −∞
Definition 16.6.3. Some authors also define the Fourier transform as follows
(i) Z ∞
F [ f (x)] = f (p) = e−ipx f (x)dx.
−∞
(ii) Z ∞
1
f (x) = f (p)eipx d p.
2π −∞
(i) Let f (x) be a function defined in (0, ∞). Thus we obtain the definition of Fourier sine
transform as r Z ∞
2
f s (p) = F s [ f (x)] = f (x) sin pxdx
π 0
The function f (x) is called the inverse Fourier sine transform of f s (p)
r Z ∞
2
i.e., f (x) = F [ f s (p)] =
−1
f (p) sin pxd p
π 0 s
(ii) The Finite Fourier Sine Transform of f (x) when 0 < x < l, is defined as
Z l
pπx
f s (p) = F s [ f (x)] = f (x) sin dx,
0 l
∞
2X pπx
f (x) = f s (p) sin
l p=1 l
(i) Let f (x) be a function defined in (0, ∞). Thus we obtain the definition of Fourier cosine
transform as r Z ∞
2
f s (p) = Fc [ f (x)] = f (x) cos pxdx
π 0
The function f (x) is called the inverse Fourier cosine transform of f c (p)
r Z ∞
2
i.e., f (x) = F [ f c (p)] =
−1
f (p) cos pxd p
π 0 c
(ii) The Finite Fourier Cosine Transform of f (x) when 0 < x < l, is defined as
Z l
pπx
f c (p) = Fc [ f (x)] = f (x) cos dx,
0 l
∞
1 2X pπx
f (x) = f c (0) + f c (p) cos
l l p=1 l
Z ∞
∴ F [ f (t)] = f (t)eipt dt
−∞
Z 0 Z ∞
= f (t)e dt + ipt
f (t)eipt dt
−∞ 0
Z 0 Z ∞
F [ f (t)] = 0.e dt +
ipt
e−xt φ(t)eipt dt
−∞ 0
∴ F [ f (t)] = L [φ(t)]
1. Linear Property. If f (p) and g(p) are Fourier transforms of f (x) and g(x) respectively, then
Z ∞
1
F [a f (x) + bg(x)] = √ eipx [a f (x) + bg(x)]dx
2π −∞
Z ∞ Z ∞
1 h i
= √ a e f (x)dx + b
ipx
eipx g(x)dx
2π −∞ −∞
= a f (p) + bg(p).
Note.
Similarly, (i)
F s [a f (x) + bg(x)] = a f s (p) + bg s (p)
(ii)
Fc [a f (x) + bg(x)] = a f c (p) + bgc (p)
2. Change of Scale Property. If f (p) is the complex Fourier transforms of f (x), then
1 p
F [ f (ax)] = f
a a
where a > 0.
Proof. We have Z ∞
1
f (p) = √ eipx f (x)dx
2π −∞
Z ∞
1
∴ F [ f (ax)] = √ eipx f (ax)dx.
2π −∞
Put ax = t so that dx = dt
a
.
If x → −∞ then t → −∞ and if x → ∞ then t → ∞.
Z ∞ t
1 ip dt
F [ f (ax)] = √ e a f (t)
2π −∞ a
Z ∞ p
1 1 i t
= √ e a . f (t)dt
2π a −∞
1 p
F [ f (ax)] = f
a a
Note. If f s (p) and f c (p) are the Fourier sine and cosine transform of f (x) respectively, then
1 p h x i
F s [ f (ax)] = fs (or) F s f = a f s (ap).
a a a
and
1 p h x i
Fc [ f (ax)] = fc (or) Fc f = a f c (ap).
a a a
3. Shifting Property. If f (p) is the complex Fourier transforms of f (x), then the complex
Fourier transform f (x − a) is eipa f (p). i.e., F [ f (x − a)] = eipa f (p).
Proof. We have Z ∞
1
f (p) = √ eipx f (x)dx
2π −∞
Z ∞
1
∴ F [ f (x − a)] = √ eipx f (x − a)dx.
2π −∞
Z ∞
1
F [ f (x − a)] = √ eip(a+t) f (t)dt
2π −∞Z
1 ipa ∞ ipt
= √ e e . f (t)dt
2π −∞
Proof. We have Z ∞
1
f (p) = √ eipx f (x)dx
2π −∞
5. If f s (p) and f c (p) are the Fourier sine and cosine transforms of f (x) respectively, then
r Z ∞
2 h sin(p + a)x + sin(p − a)x i
F s [ f (x) cos ax] = eipx f (x) dx
π −∞ 2
r Z
1 h 2 ∞ ipx
Z ∞ i
= e f (x) sin(p + a)xdx + eipx f (x) sin(p − a)xdx
2 π −∞ −∞
1
F s [ f (x) cos ax] = [ f s (p + a) + f s (p − a)].
2
(ii)
r Z ∞
2
Fc [ f (x) sin ax] = eipx f (x) sin ax cos pxdx
π −∞
r Z ∞
2 h sin(p + a)x − sin(p − a)x i
= eipx f (x) dx
π −∞ 2
r Z
1 h 2 ∞ ipx
Z ∞ i
= e f (x) sin(p + a)xdx − eipx f (x) sin(p − a)xdx
2 π −∞ −∞
1
Fc [ f (x) sin ax] = [ f (p + a) − f s (p − a)].
2 s
1
F s [ f (x) sin ax] = [ f c (p − a) − f c (p − a)].
2
n
6. F [xn f (x)] = (−i)n ddpn [ f (p)].
Proof.
f (p) = F [ f (x)]
Z ∞
1
= √ f (x)eipx dx
2π −∞Z
∞
d d h 1 i
[ f (p)] = √ f (x)eipx dx
dp d p 2π −∞
∂
Z ∞
1
= √ f (x). (eipx )dx
2π Z−∞ ∂p
∞
1
= √ f (x)(ix)eipx dx
2π −∞
1 d
i.e., F [x f (x)] = [ f (p)]
i dp
d
F [x f (x)] = (−i) [ f (p)]
dp
Z ∞
d2 d h 1 i
[ f (p)] = √ f (x)(ix)eipx dx
d p2 d p 2π −∞
∂
Z ∞
1
= √ f (x) [eipx ](ix)dx
2π Z−∞ ∂p
∞
1
= √ f (x)(ix)2 eipx dx
2π −∞Z
∞
1
= (i)2 √ f (x)x2 eipx dx
2π −∞
1 d2
i.e., F [x2 f (x)] = [ f (p)]
(i)2 d p2
(−i)2 d2
= [ f (p)]
(i)2 (−i)2 d p2
d2
F [x2 f (x)] = (−i)2 2 [ f (p)]
dp
dn
In general F [xn f (x)] = (−i)n n [ f (p)].
dp
h
dn
7. F dxn
[ f (x)] = (−ip)n f (p) if f (x) → 0 as x → ±∞
Proof.
hd i
F [ f 0 (x)] = F f (x)
dxZ
∞
1
= √ f 0 (x)eipx dx
2π Z−∞
∞
1
= √ eipx d[ f (x)]
2π −∞ Z ∞
1 h ipx ∞ i
= √ e f (x) − f (x)(ip)eipx dx by integration by parts
−∞
2π −∞
= −ipF [ f (x)]
h d2 i
Now F [ f 00 (x)] = F f (x)
dxZ2
∞
1
= √ f 00 (x)eipx dx
2π Z−∞
∞
1
= √ eipx d[ f 0 (x)]
2π −∞ Z ∞
1 h ipx 0 ∞ i
= √ e f (x) − f 0 (x)(ip)eipx dx
−∞
2π −∞
Z ∞
1 h i
= √ − ip f 0 (x)eipx dx
2π −∞
1 h i
= √ − ip(−ipF [ f (x)]) since F [ f 0 (x)] = −ipF [ f (x)]
2π
F [ f 00 (x)] = (−ip)2 F [ f (x)]
Proof.
r Z ∞
2
F s [ f (x)] =
0
f 0 (x) sin pxdx
π
r Z0 ∞
2
= sin pxd[ f (x)]
π 0
r Z ∞
2h i
= ∞
(sin px. f (x))0 − f (x).p. cos pxdx
π 0
F s [ f 0 (x)] = −p f c (p).
R∞ sin p
R∞ sin ap. cos px
and hence evaluate 0 p
dp and −∞ p
d p.
Part (ii):
By the definition of inverse Fourier transform, we have
Z ∞
1
f (x) = √ f (p)e−ipx d p
2π Z−∞
∞
1 1 2 sin pa −ipx
f (x) = √ √ . e dp
2π −∞ 2π p
1 h ∞ sin pa
Z Z ∞
sin pa
f (x) = cos pxd p − i sin pxd p
π −∞ p −∞ p
1 ∞ sin pa
Z
f (x) = cos pxd p
π −∞ p
π, |x| < a
∞
Z
sin pa
cos pxd p =
p
−∞
|x| > a
0,
1 ∞ sin pa
Z
f (0) = dp
π −∞ p
2 ∞ sin pa
Z
1 = dp
π 0 p
∞
π
Z
sin x
dx = .
0 x 2
Example 16.9.2. Find the Fourier transform of f (x) defined by
1 − x ,
2
|x| ≤ 1
f (x) =
|x| > 1
0,
R∞ x cos x−sin x
R∞ x cos x−sin x
and hence evaluate (i) 0 x3
cos 2x dx and (ii) −∞ x3
dx.
Z 1
2
F [ f (x)] = √ (1 − x2 ) cos pxdx
2π 0
2 h sin px − cos px − sin px i1
= √ (1 − x2 ) − (−2x) + (−2)
2π p p2 p3 0
2 h cos p sin p i
= √ 0 − 2 2 + 2 3 − (0 − 0 + 0)
2π p p
Part (ii)
By the definition of inverse Fourier transform, we have
Z ∞
1
f (x) = √ f (p)e−ipx d p
2π Z−∞
∞
1 4 h sin p − p cos p i −ipx
= √ √ e dp
2π −∞ 2π p3
2 ∞ h sin p − p cos p i −ipx
Z
f (x) = e dp
π −∞ p3
2 h ∞ sin p − p cos p
Z Z ∞
sin p − p cos p i
= cos pxd p − i sin pxdx
π −∞ p3 −∞ p3
Z ∞
2 sin p − p cos p
= cos pxd p
π −∞ p3
Z ∞ sin p − p cos p
4
f (x) = cos pxd p. (16.9.1)
π 0 p3
Put x = 1
2
in (16.9.1), we get
4 ∞ sin p − p cos p
1 Z
p
f = cos dp
2 π 0 p3 2
4 ∞ sin p − p cos p
Z
1 p
1− = cos dp
4 π 0 p3 2
4 ∞ sin p − p cos p
Z
3 p
= cos dp
4 π 0 p3 2
Z ∞
sin p − p cos p p 3π
3
cos d p =
p 2 16
Z0 ∞
p cos p − sin p p 3π
∴ cos dp = −
0 p3 2 16
4 ∞ sin p − p cos p
Z
f (0) = dp
π 0 p3
4 ∞ sin p − p cos p
Z
1 = dp
π 0 p3
π
Z ∞
p cos p − sin p
∴ 3
dp = − .
0 p 4
Note. If the Fourier transform of f (x) is equal to f (p) then f (x) is called self - reciprocal.
x2
Example 16.9.4. Find the Fourier transform of f (x) defined by f (x) = e− 2 , −∞ < x < ∞.
x2
(or) Show that the Fourier transform of e− 2 is self - reciprocal.
Z ∞
1 1 2 2
F [ f (x)] = √ e 2 (−p −(x−ip) ) dx
2π −∞ Z
1 − p22 ∞ − 12 ((x−ip)2 )
= √ .e e dx
2π −∞
√
Put x−ip
√
2
= t. Then dx = 2dt.
1 − p22 ∞ −t2 √
Z
∴ F [ f (x)] = √ .e e 2dt
2π −∞
√
2 − p22 ∞ −t2
Z
= √ .e e dt
2π −∞
Z ∞
1 − p22 √ 2 √
= √ e . π, since e−x dx = π
π −∞
p2
F [ f (x)] = e− 2 .
x2 p2 x2
Since Fourier transform of e− 2 is e− 2 . Therefore e− 2 is self reciprocal.
0, −∞ < x < α
f (x) =
x, α ≤ x ≤ β
0, x > β
Example 16.9.6. Find the inverse Fourier transform f (x) of F (p) = e−|p|y .
Solution. We have
ifp < 0
−p,
|p| =
ifp > 0
p,
ifp = 0
0,
1 h e(y−ix)p i0 1 h e−(y+ix)p i∞
= √ + √
2π y − ix −∞ 2π −(y + ix) 0
1 h 1 i 1 h 1 i
= √ + √
2π y − ix 2π y + ix
1 2y
= √ . 2
2π y + x2
r
2 y
f (x) = .
π y2 + x2
q R ∞ 2
. Hence deduce that 0 sint t dt = π2 .
2 1−cos as
is π s2
1 h a
Z Z a i
F [ f (x)] = √ (a − |x|) cos pxdx + i (a − |x|) sin pxdx
2π Z −a −a
a
2
= √ (a − |x|) cos pxdx
2π Z0
a
2
= √ (a − x) cos pxdx since a − |x| = a − x in (0, a)
2π 0
2 h cos ap 1 i
= √ 0− − 0 −
2π p2 p2
2 h 1 − cos ap i
= √
2π p2
r
2 h 1 − cos ap i
F [ f (x)] = f (p) = .
π p2
Put x = 0, we have
1 ∞ 1 − cos ap
Z
f (0) = dp
π −∞ p2
1 ∞ 1 − cos ap
Z
a = dp
π −∞ p2
1 ∞
Z ap 1
a = 2 sin2 . dp
π −∞ 2 p2
2 ∞ 2 ap 1
Z
a = sin . dp
π −∞ 2 p2
4 ∞ 2 ap 1
Z
a = sin . dp
π 0 2 p2
Put ap
2
= t. Then a2 d p = dt ⇒ d p = 2dt
a
Z ∞
4 1 2dt
a = sin2 t. 4t2
π 0 2
a
a
Z ∞ 2
4 a sin t
a = . dt
π 2 0 t2
∞
sin2 t π
Z
2
dt = .
0 t 2
1 + ax , for − a < x < 0
f (x) =
1 − ax , for 0 < x < a
0,
otherwise
√
2 π
F [ f (x)] = √ cos
p 4
√
2 1
= √ .√
p 2
h 1 i 1
F √ = √ .
|x| p
Solution. Z ∞
1
F [ f (x)] = √ f (x)eipx dx
2π −∞
1 h −a
Z Z a Z ∞ i
F [ f (x)] = √ f (x)e dx +
ipx
f (x)e dx +
ipx
f (x)eipx dx
2π −∞ −a a
Z −a Z a Z ∞
1 h i
= √ 0.eipx dx + x.eipx dx + 0.eipx dx
2π Z −∞ −a a
a
1
= √ x.eipx dx
2π −a
Solution. Z ∞
1
F [ f (x)] = √ f (x)eipx dx
2π −∞
1 h −a
Z Z a Z ∞ i
F [ f (x)] = √ f (x)e dx +
ipx
f (x)e dx +
ipx
f (x)eipx dx
2π −∞ −a a
Z −a Z a Z ∞
1 h i
= √ 0.eipx dx + cos x.eipx dx + 0.eipx dx
2π Z −∞ −a a
a
1
= √ cos x.eipx dx
2π −a
1 h eipx ia
= √ (ip cos x + sin x)
2π i2 p2 + 1 −a
1 h eipa e−ipa i
= √ (ip cos a + sin a) − (ip cos a − sin a)
2π 1 − p2 1 − p2
1 1 h ipa i
= √ (e − e−ipa
)ip cos a + (eipa
− e −ipa
) sin a
2π 1 − p2
1 1 h i
= √ (2i sin pa)ip cos a + (2 cos pa) sin a
2π 1 − p2
Solution. Z ∞
1
F [ f (x)] = √ f (x)eipx dx
2π −∞
Z π Z ∞
1 h 0
Z i
F [ f (x)] = √ f (x)e dx +
ipx
f (x)e dx +
ipx
f (x)eipx dx
2π −∞ 0 π
Z 0 Z π Z ∞
1 h i
= √ 0.eipx dx + sin x.eipx dx + 0.eipx dx
2π Z −∞ 0 π
π
1
F [ f (x)] = √ sin x.eipx dx
2π 0
1 h eipx iπ
F [ f (x)] = √ (ip sin x − cos x)
2π i2 p2 + 1 0
1 h eipπ 1 i
= √ (0 − (−1)) − (0 − 1)
2π 1 − p2 1 − p2
1 h eipπ 1 i
= √ +
2π 1 − p2 1 − p2
1 1 + eipπ
F [ f (x)] = √ .
2π 1 − p2
Solution. Z ∞
1
F [ f (x)] = √ f (x)eipx dx
2π Z−∞
∞
1
= √ xe−x eipx dx
2π 0
x2 p2
Example 16.9.14. Show that the Fourier transform of e− 2 is e− 2 by finding the Fourier
transform of e−a x .
2 2
Solution. Z ∞
1
F [ f (x)] = √ f (x)eipx dx
2π Z−∞
∞
1 2 2
= √ e−x a eipx dx
2π Z−∞
∞
1
e−x a +ipx dx
2 2
F [ f (x)] = √
2π −∞
ip
−a2 x2 + ipx = −a2 x2 − 2 x
a
h ip 2 ip 2 i
= −a2 x − 2 −
2a 2a2
h ip 2 p2 i
= −a x − 2 + 4
2
2a 4a
2
ip 2 p
−a2 x2 + ipx = −a2 x − 2 − 2
2a 4a
Z ∞ 2 2
1 −a2 x− ip2 − p 2
F [ f (x)] = √ e 2a 4a
dx
2π −∞
2
1 − p22 ∞ −a2 x− 2aip2
Z
= √ e 4a e dx
2π −∞
1 − p22 ∞ −t2 dt
Z
∴ F [ f (x)] = √ e 4a e
2π −∞ a
Z ∞
1 − p2 √
2
2 √
= √ e 4a . π, since e−x dx = π
2π −∞
1 − p22
F [ f (x)] = √ e 4a
a 2
Put a = √1
2
in the above equation, we get
x2 p2
F [e− 2 ] = e− 2 .
Example 16.9.15. Find the Fourier transform of e−a|x| and hence show that
R ∞ px √
π −a|x|
(i) 0 acos
2 +p2 d p = 2a
e and (ii) F [xe−a|x|
] = √
2 2pai
π(p2 +a2 )2
.
Solution. Z ∞
1
F [ f (x)] = √ f (x)eipx dx
2π Z−∞
∞
1
F [e−a|x| ] = √ e−a|x| eipx dx
2π Z −∞
∞
1
F [e−a|x| ] = √ e−a|x| [cos px + i sin px]dx
2π −∞
1 h ∞ −a|x|
Z Z ∞ i
F [e −a|x|
] = √ e cos pxdx + i e−a|x| sin pxdx
2π Z−∞ −∞
∞
1
= √ 2. e−a|x| cos pxdx
2π Z0
∞
1
= √ 2. e−ax cos pxdx
2π 0
r
2 a
= .
π a2 + p2
a ∞
Z
1
= (cos px − i sin px)dx
π −∞ a + p2
2
ah ∞
Z Z ∞
1 1 i
= cos pxdx − i sin pxdx
π −∞ a2 + p2 −∞ a + p
2 2
Z ∞
a 1
e−a|x| = 2. cos pxdx
π 0 a + p
2 2
∞
π −a|x|
Z
1
cos pxdx = e .
0 a2 + p2 2a
r
d h 2 a i
F [xe −a|x|
] = −i
dp π a2 + p2
r
2h 1 i i
= −ai − 2 (2p)
π (a + p2 )2
√
2 2pai
F [xe−a|x| ] = √ 2 .
π(p + a2 )2
Example 16.9.16. Find the Fourier transform of e−|x| and hence show that
R ∞ xt √
π −|x|
(i) 0 cos
1+t2
d p = 2
e and (ii) F [xe−|x|
] = √
2 2pi
π(p2 +1)2
.
R∞
and hence show that 0
sin x−cos x
x3
dx = π4 .
2 h cos ap sin ap i
= √ 0 − 2a 2 + 2 3 − (0 − 0 + 0)
2π p p
4 sin ap − ap cos ap
h i
F [ f (x)] = f (p) = √
2π p3
2 ∞ h sin ap − ap cos ap i
Z
f (0) = dp
π −∞ p3
2 ∞ h sin ap − ap cos ap i
Z
a =
2
dp
π −∞ p3
Put ap = t. Then d p = dt
a
.
When p → −∞ ⇒ t → −∞ and p → ∞ ⇒ t → ∞.
Z ∞ h sin t − t cos t i dt
2
a =
2
π −∞
t3 a
a3
∞h
2a2
Z
sin t − t cos t i
= dt
π −∞ t3
π ∞h
Z
sin t − t cos t i
a2 . = 2. dt
2a2 0 t3
π
Z ∞h
sin t − t cos t i
∴ 3
dt = .
0 t 4
16.10 Exercises
(i)
x , |x| < x0
2
f (x) =
|x| > x0
0,
(ii)
2a , |x| ≤ a
1
f (x) =
0, |x| > a
is sin pa
pa
.
x2
3. Find the Fourier transform of e− 2 , −∞ < x < ∞.
4. Show that the Fourier transform of
0, forx < α
f (x) =
1, forα < x < β
0, forx > β.
1 iβp
is ip
(e − eiαp ).
Answers.
√ h i
1. (i) √ 2 3 (x2 p2
πp 0 − 2) sin px0 = 2px0 cos px0 .
(ii) π √1 2 sinaαaα .
p2
3. e− 2 .
Lesson 17
• solve integral equations by using infinite Fourier sine and cosine transform.
17.1 Introduction
I
n this Lesson, we present problems related to (infinite) Fourier Sine and Cosine Transforms.
Example 17.2.1. Find the Fourier sine and cosine transform of f (x) = x.
and r Z ∞ r Z ∞
2 2
f c (p) = f (x) cos pxdx = x cos pxdx
π 0 π 0
Let r Z ∞
2
f (p) = xe−ipx dx
π 0
Put ipx = t. Then ipdx = dt
r Z ∞
2 t −t dt
f (p) = e
π 0 ip ip
r Z ∞
1 2
= − 2 e−t t2−1 dt
p π 0
r Z ∞
1 2
= − 2 Γ(2), since Γ(n) = e−x xn−1 dx, n > 0
p π 0
r
1 2
= − 2 , Γ(n) = (n − 1)! i f n is a positive integer
p π
r Z ∞ r
2 1 2
∴ xe−ipx dx = − 2
π 0 p π
Z ∞
1
xe−ipx dx = − 2.
0 p
1
i.e., f s (p) = 0 and f c (p) = − .
p2
Example 17.2.2. Find the Fourier cosine transform of the function f (x) defined by
0<x<a
cos x,
f (x) =
0,
x≥a
Z a
1
f c (p) = √ (cos(1 + p)x + cos(1 − p)x)dx
2π 0
1 h sin(1 + p)x sin(1 − p)x ia
= √ +
2π 1+ p 1− p 0
Example 17.2.3. Find the Fourier cosine transform of the function f (x) defined by
0<x<a
sin x,
f (x) =
0,
x≥a
Solution. r Z ∞
2
f s (p) = f (x) sin pxdx
π 0
r Z ∞
2
= e−x sin pxdx
π 0
r
2 h e−x i∞
= (− sin px − p cos px)
π p2 + 1 0
r
2 p
∴ f s (e−x ) = .
π p2 + 1
Note. r Z ∞ r
2 2 1
f c (e−x ) = e−x cos pxdx = .
π 0 π p2 + 1
−ax
Example 17.2.5. Find the Fourier sine and cosine transforms of f (x) = e x , deduce that
R ∞ −ax −bx
(i) 0 e −e x
sin pxdx = tan −1 p
a
− tan −1 p
b
R ∞ h e−ax −e−bx i q p2 +b2
and (ii) 0 x
cos pxdx = log p2 +a2
.
e−ax 1
Fc [ ] = − √ log(p2 + a2 )
x 2π
We know that Fourier cosine transform of f (x) is given by
r Z ∞
2
Fc [ f (x)] = f c (p) = f (x) cos pxdx
π 0
r Z ∞
2 h e−ax − e−bx i h e−ax − e−bx i
cos pxdx = Fc
π x x
r Z0 ∞
2 h e−ax − e−bx i e−ax e−bx
cos pxdx = Fc − Fc
π 0 x x x
r Z ∞
2 h e−ax − e−bx i 1 1
cos pxdx = − √ log(p2 + a2 ) + √ log(p2 + b2 )
π 0 x 2π 2π
r Z ∞
2 h e−ax − e−bx i 1 p2 + b2
cos pxdx = √ log 2
π 0 x 2π p + a2
1 p2 + b2
= log 2
2 p + a2
s
Z ∞ h −ax
e − e−bx i p2 + b2
cos pxdx = log .
0 x p2 + a2
e−2x
Note. Find the Fourier sine transform of x
.
h i q p
−2x
Putting a = 2 in the above problem, we get Fs e x = 2
π
tan−1 2
.
Example 17.2.7. Find the Fourier cosine transform of f (x) defined by f (x) = 1
1+x2
and hence
find the Fourier sine transform of f (x) = x
1+x2
.
Solution. r Z ∞
2
Let I = Fc [ f (x)] = f (x) cos pxdx
π 0
r Z ∞
2 1
= cos pxdx
π 0 1 + x2
which gives (D2 − 1)Fc [ f (x)] = 0, whose general solution is Fc [ f (x)] = c1 e p + c2 e−p . When
p = 0, r Z ∞ r r
2 π π
r
2 1 2
Fc [ f (x)] = dx = (tan (x))0 =
−1 ∞
. =
π 0 1+x 2 π π 2 2
π
r
c1 + c2 = (17.2.1)
2
Also r
d 2
[Fc [ f (x)] = − , (when p = 0)
dp π
r
2 −p
∴ I = Fc [ f (x)] = e .
π
Now, r
dI 2 −p
=− e
dp π
Since r Z ∞
dI 2 x sin px h x i
=− dx = −F s
dp π 0 1 + x2 1 + x2
r r
2 −p h x i h x i 2 −p
∴− e = −F s ⇒ Fs = e .
π 1+x 2 1+x 2 π
Example 17.2.8. Find the Fourier sine transform of f (x) defined by f (x) = 1
x(x2 +a2 )
and hence
deduce Fourier cosine transform of f (x) = 1
x2 +a2
.
Solution. r Z ∞
2
Let F s [ f (x)] = f (x) sin pxdx
π 0
r Z ∞
2 1
= sin pxdx
π 0 x(x + a2 )
2
r Z ∞ r Z ∞Z ∞
2 cos px 2
cos px2ye−(x +a )y dydx
2 2 2
dx =
π 0 x +a
2 2 π
r Z0 ∞ 0
2 hZ ∞ 2 2
−a2 y2
i
= 2ye e−x y cos pxdx dy
π 0 0
Now, let Z ∞
2 2
S = e−x y cos pxdx
0
When p = 0, √
∞
π
Z
−x2 y2
S = e dx =
0 2y
√
π
∴ c1 = .
2y
Hence √
∞
π p2
Z
−x2 y2
S = e cos pxdx = . − 4y2 .
0 2y e
r Z ∞ r Z ∞ √
2 cos px 2 −a2 y2 π p2
∴ dx = 2ye . − dy
π 0 x 2 + a2 π 0 2y e4y2
√ Z ∞ −a2 y2 + p22 2
= 2 e 4a y
dy
0
√
√ π −2 2ap a2
= 2. e
r Z ∞ 2a
πe
r −pa
2 cos px
dx = .
π 0 x +a
2 2 2 a
π e−pa
r
d
∴ F s [ f (x)] = .
ds 2 a
Integrating both sides with respect to p, we get
√ Z √ −pa √
π π e π
F s [ f (x)] = √ e dp = √ .
−pa
+ c2 = − √ e−pa + c2
a 2 a 2 −a a2 2
When p = 0, we have r Z ∞
2
F s [ f (0)] = f (x).0dx = 0
π 0
√
π
∴ c2 = √
a2 2
Hence √ √
π −pa π π
r
F s [ f (x)] = − 2 √ e + √ = [1 − e−pa ].
a a 2
a 2 2
If f (x) = 1
x2 +a2
, then
r Z ∞ r
2 π −pa π −pa
r
2 cos px
Fc [ f (x)] = dx = . e = e
π 0 x +a
2 2 π 2a 2
Solution. Let r Z ∞
2 2
I = Fc [ f (x)] = e−x cos pxdx.
π 0
Differentiating both sides with respect to p, we get
r Z ∞
dI 2 2
= e−x (−x sin px)dx
dp π 0
r
2 1 ∞
Z
dI 2
= . (−2xe−x ) sin pxdx
dp π 2 0
Z ∞
1 h −x2 2
= √ (e sin px)0 − ∞
pe−x cos pxdx
2π Z 0
∞
p 2
= −√ e−x cos pxdx
2π 0
dI p
= − √ .I
dp 2π
dI p
= − √ dp
I 2π
p
Integrating, log I = − √ + log c
2 2π
p2
−
I = ce .
√
2 2π
If p = o, then √
∞
π
Z
−x2
I= e dx = .
0 2
√
π
∴ =c
2 √
π − √p2
Hence I = e 2 2π
2
√
π − √p2
i, e., Fc [ f (x)] = e 2 2π .
2
Example 17.2.10. Find the Fourier cosine and sine transform of e−ax , a > 0 and hence evaluate
R ∞ px R ∞ p sin px
(i) 0 acos
2 +p2 d p and (ii) 0 a2 +p2
dp
Solution. By definition,
r Z ∞
2
Fc [ f (x)] = cos pxdx
π 0
r Z ∞
2
Fc [e )] =
−ax
e−ax cos pxdx
π 0
r
2 a
Fc [e−ax )] = .
π a2 + p2
r Z ∞
2
F s [ f (x)] = sin pxdx
π 0
r Z ∞
2
F s [e−ax )] = e−ax sin pxdx
π 0
r
2 p
F s [e−ax )] = .
π a2 + p2
2 ∞ p sin px
Z
e = −ax
dx
π 0 a2 + p2
∞
π −ax
Z
p sin px
i.e., dx = e .
0 a2 + p2 2
Note. Find the Fourier cosine and sine transform of f (x) = e−x . Put a = 1 in the above problems,
we get Fc [ f (x)] = 1
1+p2
and F s [ f (x)] = p
1+p2
Solution. We have r
2 p
F s [e −ax
)] = .
π a2 + p2
The inverse Fourier sine transform of e−ax is
r Z ∞
2
f (x) = F s [ f (x)] sin pxd p
π 0
r Z ∞
2
f (x) = F s [e−ax ] sin pxd p
π 0
r Z ∞r
2 2 p
f (x) = sin pxd p
π 0 π a2 + p2
2 ∞ p sin px
Z
e−ax = dp
π 0 a2 + p2
∞
π −ax
Z
p sin px
i.e., dp = e .
0 a2 + p2 2
∞
π −ap
Z
x sin xp
dx = e .
0 a2 + x 2 2
Hence
h x i π −ap
Fs = e .
x 2 + a2 2
Also we have r
2 a
Fc [e−ax )] = .
π a2 + p2
The inverse Fourier sine transform of e−ax is
r Z ∞
2
f (x) = Fc [ f (x)] cos pxd p
π 0
r Z ∞
2
f (x) = Fc [e−ax ] cos pxd p
π 0
r Z ∞r
2 2 1
f (x) = cos pxd p
π 0 π a2 + p2
2a ∞ cos px
Z
e−ax = dp
π 0 a2 + p2
∞
π −ax
Z
cos px
i.e., dp = e .
0 a2 + p2 2a
∞
π
Z
cos xp
dx = e−ap .
0 a +x
2 2 2a
Hence
h 1 i π
Fc = e−ap .
x +a
2 2 2a
Example 17.2.12. Find the Fourier sine transform of f (x) = xm−1 , 0 < m < 1.
Solution. r Z ∞
2
F s [ f (x)] = sin pxdx
π 0
r Z ∞
2
F s [xm−1 ] = xm−1 sin pxdx
π 0
We know that
Z ∞ h e−st i∞
e−xt sin pxdx =(−t sin px − p cos px)
0 t 2 + p2 0
p
= 2
t + p2
Z ∞
1h i
e−xt sin pxdx = (t − ip)−1 − (t + ip)−1
0 2i
1
Since r = (t2 + p2 ) 2 and p
t
= tan θ.
∞
π (m − 1)!
Z r
−1 p
∴ x m−1 −xt
e sin pxdx = sin m tan .
2 (t2 + p2 ) 2
m
0 t
∞
π (m − 1)!
Z r
mπ
xm−1 sin pxdx = sin
0 2 pm 2
. Thus
π Γ(m)
r
mπ
F s [ f (x)] = sin
2 pm 2
.
Example 17.2.13. Find the Fourier sine and cosine transform of 2e−5x + 5e−2x .
(i) We have r
2 a
Fc [e−ax ] = .
π a2 + p2
Example 17.2.14. Find the Fourier sine and cosine transform of (i) xn−1 , (ii) √1 ,
x
and (iii) x.
Solution. r Z ∞
2
F s [xn−1 ] = xn−1 sin pxdx
π 0
r Z ∞
2
Fc [x ] =
n−1
xn−1 cos pxdx
π 0
r Z ∞
2
Fc [x n−1
] + iF s [x n−1
] = xn−1 (cos px + i sin px)dx
π 0
r Z ∞
2
= xn−1 eipx dx
π 0
r Z ∞
2 −t n−1 dt
= e−t − , Puttingipx = −t
π 0 ip ip
(ii) Put n = 1
2
in the above result, we have
√
2 Γ( 2 )
r 1
r
h 1 i π 2 π 1
Fc √ = cos = √ = √ .
x π p2 1
4 π p p
√
2 Γ( 2 )
r 1
r
h 1 i π 2 π 1
Fs √ = sin = √ = √ .
x π p2 1
4 π p p
(iii) Putting n = 2 in the above result
r r
2 Γ(2) 2 1
Fc [x] = cos π = − .
π p2 π p2
and r
2 Γ(2)
F s [x] = sin π = 0.
π p2
Example 17.2.15. Show that (i) F s [x f (x)] = − ddp [Fc (p)] and (ii) Fc [x f (x)] = d
dp
[F s (p)].
d
∴ F s [x f (x)] = − [Fc (p)].
dp
(ii) We have r Z ∞
2
F s (p) = f (x) sin pxdx
π 0
Differentiating with respect to p, we get
r Z ∞ r Z ∞
d 2 2
[F s (p)] = f (x)(x cos px)dx = [x f (x)] cos pxdx = −Fc [x f (x)]
dp π 0 π 0
d
∴ Fc [x f (x)] = [F s (p)].
dp
2 x2
Example 17.2.16. Find the Fourier cosine transform of e−a and hence evaluate Fourier sine
transform of xe−a x .
2 2
Solution. r Z ∞
−a2 x2 2 2 2
Fc [e ] = e−a x cos pxdx
π 0
r Z ∞
2 2 2
= R.P o f e−a x eipx dx
π
r Z0 ∞
2
e−a x +ipx dx
2 2
= R.P o f
π 0
r √
2 π − p22
= . e 4a
π 2a
2 2 1 − p2
Fc [e−a x ] = √ e 4a2 .
a 2
We know that
d h i
F s [x f (x)] = − Fc [ f (x)]
dp
2 2 d h 2 2
∴ F s [xe−a x ] = − Fc [e−a x ]
dp
d h 1 − p22 i
= − √ e 4a
dp a 2
1 p2 p
= − √ e− 4a2 − 2
a 2 2a
2 2 p −p
2
∴ F s [xe−a x ] = √ e 4a2 .
a3 2 2
R∞ x∼mx
Example 17.2.17. Find the Fourier sine transform of e−|x| and hence evaluate 0 1+x2
dx.
Solution. r Z ∞
2
F s [e−|x| ] = e−|x| sin pxdx
π 0
r Z ∞
2
= e−x sin pxdx
π 0
r
2 h e−x i∞
= (− sin px − p cos px)
π 1 + p2 0
r
2 p
= .
π 1 + p2
for, 0 < x < 1
x,
f (x) =
for, 1 < x < 2
2 − x,
for, x > 2
0,
q R
2 ∞
is π 0
2 sin p(1−cos p)
p2
.
Solution. By definition,
r Z ∞
2
F s [ f (x)] = f (x) sin pxdx
π 0
r Z ∞
2h 1
Z Z 2 i
F s [ f (x)] = f (x) sin pxdx + f (x) sin pxdx + f (x) sin pxdx
π
r Z0 1 Z 2
1 2
2 h i
= x sin pxdx + (2 − x) sin pxdx
π 0 1
r
2 hh cos px sin px i1 h cos px sin px i2 i
= x − − 1. − + (2 − x) − − (−1). −
π p p2 0 p p2 1
r
2 h cos p sin p sin 2p cos p sin p i
= − + 2 − + + 2
π p p p2 p p
r
2 h 2 sin p sin 2p i
= −
π p2 p2
r
2 h 2 sin p 2 sin p cos p i
= −
π p2 p2
r
2 sin p
F s [ f (x)] = 2 (1 − cos p).
π p2
for, 0 < x < 1
x,
f (x) =
for, 1 < x < 2
2 − x,
for, x > 2
0,
Solution. By definition,
r Z ∞
2
Fc [ f (x)] = f (x) cos pxdx
π
r Z0 1 Z 2 Z ∞
2h i
= f (x) cos pxdx + f (x) cos pxdx + f (x) cos pxdx
π
r Z0 1 Z 2
1 2
2 h i
= x cos pxdx + (2 − x) cos pxdx
π 0 1
r
2 hh sin px cos px i1 h sin px cos px i2 i
= x − 1. − + (2 − x) − (−1). −
π p p2 0 p p2 1
r
2 h sin p cos p 1 cos 2p sin p cos i
= + 2 − 2− − + 2
π p p p p2 p p
r
2 2 cos p cos 2p
h 1 i
= − − 2
π p2 p 2 p
r
2 h 2 cos p − cos 2p − 1 i
Fc [ f (x)] =
π p2
r
2 h 2 cos p − (2 cos2 p − 1) − 1 i
Fc [ f (x)] =
π p2
r
2 h 2 cos p − 2 cos2 p i
=
π p2
r
2 cos p
Fc [ f (x)] = 2 (1 − cos p).
π p2
Example 17.2.20. Find the Fourier cosine transform of e−ax cos ax, a > 0 and Fourier sine
transform of e−ax sin ax.
Solution. (i)
r Z ∞
2
Fc [ f (x)] = f (x) cos pxdx
π 0
r Z ∞
2
= e−ax cos ax cos pxdx
π
r Z0 ∞
2 h cos(a + p)x + cos(a − p)x i
= e−ax dx
π 0 2
r Z ∞
2 1h a a i a
= . + 2 , since eax cos bxdx = 2
π 2 a + (p + a)
2 2 a + (p − a) 2
0 a + b2
r
2 a(p2 + 2a2 )
= .
π (p2 + 2ap + 2a2 )(p2 − 2ap + 2a2 )
(i)
r Z ∞
2
F s [ f (x)] = f (x) sin pxdx
π 0
r Z ∞
2
= e−ax sin ax sin pxdx
π
r Z0 ∞
2 h cos(a − p)x − cos(a + p)x i
= e−ax dx
π 0 2
r
2 1h a a i
= . −
π 2 a2 + (p − a)2 a2 + (p + a)2
1 4a2 p
= √ .
2π (p− 2ap + 2a2 )(p2 + 2ap + 2a2 )
Solution. r Z ∞
2
F s [ f (x)] = f (x) sin pxdx
π 0
r Z a Z ∞
2h i
= k sin pxdx + 0. sin pxdx
π 0 a
r
2 cos px a
= k −
π p 0
r
2
= k (− cos ap + 1)
π
r
2 1 − cos ap
F s [ f (x)] = k
π p
r Z ∞
2
Fc [ f (x)] = f (x) cos pxdx
π 0
r Z a Z ∞
2h i
= k cos pxdx + 0. cos pxdx
π 0 a
r
2 sin px a
= k
π p 0
r
2 sin ap − 0
= k
π p
r
2 sin ap
Fc [ f (x)] = k .
π p
d
F s [x f (x)] = − [Fc [ f (x)]]
dp
d
F s [xe−ax ] = − [Fc [e−ax ]]
dp
r
d h 2 a i
= −
dp π a2 + p2
r
2 1
F s [x f (x)] = − a − 2 .2p
π (a + p )
2 2
d
Fc [x f (x)] = [F s [ f (x)]]
dp
d
Fc [xe−ax ] = [F s [e−ax ]]
dp
r
d h 2 p i
=
dp π a2 + p2
r
2 (a2 + p2 ).1 − p.2p
=
π (a2 + p2 )2
r
2 a2 − p2
Fc [xe−ax ] = . .
π (a2 + p2 )2
Example 17.2.23. Find the inverse Fourier sine transform f (x) of f s (p) = pn e−ap .
We know that Z ∞
x
e−ap sin pxdx = .
0 a2 + x2
Differentiating both sides with respect to a, n times, we get
∞
dn x
Z
(−1) n
e−ap pn sin pxd p =
0 dan a2 + x2
dn h 1 1 1 i
= −
dan 2i a − ix a + ix
1h i
= (−1)n n!(a − ix)−n−1 − (−1)n n!(a + ix)−n−1
2i
(−1)n n! h i
= (a − ix)−n−1 − (a + ix)−n−1
2i
Put x = r sin θ and a = r cos θ. Then r2 = x2 + a2 , and θ = tan−1 x
a
Similarly,
(a + ix)−(n+1) = r−(n+1) [cos(n + 1)θ − i sin(n + 1)θ]
∞
(−1)n n! −(n+1)
Z
(−1)n
e−ap pn sin pxd p = 2ir sin(n + 1)θ
0 2i
= (−1)n n!r−(n+1) sin(n + 1)θ
Z ∞
1
(−1)n
e−ap pn sin pxd p = (−1)n n! n+1
sin(n + 1)θ
0 (r2 ) 2
∞
n! sin(n + 1)θ
Z
∴ e−ap pn sin pxd p = .
(x2 + a2 )
n+1
0 2
Hence r
2 n! sin(n + 1)θ
f (x) = .
π (x2 + a2 ) n+1
2
Example 17.2.24. Find the inverse Fourier cosine transform f (x) of Fc [ f (x)] = pn e−ap .
We know that Z ∞
a
e−ap cos pxdx = .
0 a2 + x2
∞
dn a
Z
(−1) n
e−ap pn cos pxd p =
0 dan a2 + x2
dn h 1 1 1 i
= +
dan 2 a − ix a + ix
1h i
= (−1)n n!(a − ix)−n−1 + (−1)n n!(a + ix)−n−1
2
∞
(−1)n n!
Z
(−1)n e−ap pn cos pxd p = cos(n + 1)θ Re f er Example 2.1.23
rn+1
Z0 ∞
n!
∴ e−ap pn cos pxd p = cos(n + 1)θ
rn+1
Z0 ∞
n!
i.e., e−ap pn cos pxd p = cos(n + 1)θ
(a2 + x2 ) 2
n+1
0
r
2 n!
∴ f (x) = cos(n + 1)θ.
π (a2 + x2 ) n+1
2
Solution. r Z ∞
2
f (x) = f (p) cos pxd p
π 0 c
r Z 2a Z ∞
2h 1 p i
= a− cos pxd p + 0. cos pxd p
π 0 2a 2 2a
r Z 2a
2 1 p
= . a− cos pxdxp
π 2a 0 2
1 h p sin px 1 cos px i2a
= √ a− − − −
a 2π 2 x 2 x2 0
1 h 1 cos 2ax 1 i
= √ 0− +
a 2π 2 x2 2x2
1 1 − cos 2ax
= √
a 2π 2x2
1 2 sin2 ax
= √
a 2π 2x2
1 sin2 ax
f (x) = √ .
a 2π x2
e−ap
Example 17.2.26. Find the inverse Fourier cosine transform f (x) of F s (p) = p
and hence
deduce F s−1 1p
r Z ∞
df 2
= e−ap cos pxd p
dx π 0
r
2 h e−ap i∞
= (−a cos px + x sin px)
π a2 + x2 0
r
df 2 a
=
dx π a2 + x 2
Integrating,
r Z
12
f (x) = a dx
π
+ x2 a2
r
2 1 −1 x
= a. tan +c
π a a
(20.2.6) ⇒ f (0) = 0 + c ⇒ c = 0.
r
2 x
f (x) = tan−1 .
π a
Deduction. Putting a = 0 in the above equation, we get
r
2 π π
r
1
f (x) = F s−1 = . = .
p π 2 2
∞
sin(a + x)p ∞
Z Z
1 h sin(a − x)p i
f (x) = √ dp + dp
2π 0 p 0 p
π π
√2π . 2 + 2 , if a − x > 0
1
f (x) =
√12π . π2 − π2 ,
if a − x > 0
R∞
Since 0
sin ax
x
dx = π2 , when a > 0
Hence
√2π , if x > a
1
f (x) =
if x < a
0,
Solution. Z ∞
Let f (x) cos(λx)dx = e−λ (17.4.1)
0
r Z ∞ r
2 2 −λ
∴ f (x) cos(λx)dx = e .
π 0 π
r
2 −λ
∴ f c (λ) = e .
π
1, 0≤t≤1
Z ∞
F(x) sin xtdx =
2, 1≤t≤2
0
0,
t≥2
Solution. Let Z in f ty
F(x) sin pxdx = f (p).
0
1, 0≤t≤1
∴ f (t) =
2, 1≤t≤2
0,
t≥2
r Z ∞ r
2 2
F(x) sin xtdx = f (t) = F s (t).
π 0 π
By Fourier sine inversion formula, we have
r Z ∞
2
f (x) = F s (p) sin xtdt
π 0
r Z ∞r
2 2
= f (t) sin xtdt
π 0 π
2 ∞
Z
= f (t) sin xtdt
π 0
Z ∞
2h 1
Z Z 2 i
= 1. sin xtdt + 2. sin xtdt + 0. sin xtdt
π 0 1 2
2 h cos xt 1 cos xt 2 i
= − −2
π x 0 x 1
2 h cos x 1 2 cos 2x 2 cos x i
= − + − +
π x x x x
2 1 + cos x − 2 cos 2x
= .
π x
17.5 Exercises
3. Find the Fourier sine and cosine transform of the function f (x) = e−2x + 4e−3x .
eax +e−ax
4. Find the Fourier sine and cosine transform of the function f (x) = eπx −e−πx
.
5. Find the sine transform of f (x) defined by f (x) = 1
eπx −e−πx
and hence deduce that F s (csc hπx) =
p
1
2
tanh 2
.
6. Find the cosine transform of a function of x which is unity for 0 < x < a and zero for x > a.
q
What is the function whose cosine transform is 2 sin ap
π p
.
Answers.
1. π2 .
p
h i h cos(1+p) cos(p−1) i
2. (i) √12π sin(1+p)
1+p
+ sin(1−p)
1−p
; (ii) √1
2π
− 1+p + p−1 + p+1 1
− 1
p−1
.
q q
3. 2
π p2 +4
p
+ p4p2 +9 ;
2 2
− 12 .
q π pp +4− p p +9
2 2
p −p e 2 +e 2
4. √12π e p +ee−p−e+2 cos a ; 2
π 2 cos a+e p +e−p
cos a2 .
q
2 e p −1
5. .
π 4(e p +1)
.
q
6. π
. p ; f (x) = 1 when x < a and f (x) = 0, when x > a.
2 sin ap
Lesson 18
CONVOLUTION
Learning Objectives
Upon completion of this lesson, students will be able to
18.1 Introduction
I
n this Lesson, we discuss convolution concept and related results with illustrated worked out
examples.
R∞
Definition 18.1.1. The function H(x) = F ∗ G = √1
2π −∞
F(u)G(x − u)du is defined as the
Convolution or Falting of two integrable functions F and G over the interval (−∞, ∞).
Theorem 18.1.1. If F [ f (x)] and G [ f (x)] are the Fourier transforms of the functions f (x) and
g(x) respectively, then the Fourier transform of the convolution of f (x) and g(x) is the product
of their Fourier transform.
Proof. We have
h 1 Z ∞ i
F [ f (x) ∗ g(x)] = F √ f (u)g(x − u)du
2π −∞
Z ∞h Z ∞
1 1 i
F [ f (x) ∗ g(x)] = √ √ f (u)g(x − u)du eipx dx
2π
Z ∞−∞ h 2π Z ∞ −∞
1 i
= f (u) g(x − u)eipx dx du by changing the order of integration
2π −∞ −∞
Z ∞
1
= f (u)F [g(x − u)]du
2π −∞
Z ∞
1
= f (u)eiup F [g(x)]du, by using shifting theorem
2π −∞
Z ∞
1
= F [g(x)]. f (u)eiup du
2π −∞
Z ∞
1
= F [g(x)]. f (x)eixp dx, by changing the dummy variable u by x
2π −∞
= F [g(x)].F [ f (x)]
Theorem 18.2.1. Parseval’s Theorem or Rayleigh’s Theorem. If f (p) and g(p) are Fourier
transform of f (x) and g(x) respectively. Then
R∞ R∞
(i) −∞
f (p)g(p)d p = −∞
f (x)g(x)dx
R∞ R∞
(ii) −∞
| f (x)|2 dx = −∞
| f (p)|2 d p. where bar denotes the complex conjugate.
Z ∞ Z ∞ h 1 Z ∞
1 i
f (x)g(x)dx = √ g(p) √ f (x)eipx dx d p
−∞ 2π −∞ 2π −∞
or Z ∞ Z ∞
| f (p)| d p =
2
| f (x)|2 dx.
−∞ −∞
2. Similarly, we can prove the following Parseval’s identities for Fourier sine and cosine
transforms
R ∞ as above. R∞ R∞ R∞
(i) 0 f c (p)gc (p)d p = 0 f (x)g(x)dx; (ii) 0
f s (p)g s (p)d p = 0
f (x)g(x)dx;
R∞ R∞ R∞ R∞
(iii) 0 | f c (p)|2 d p = 0 | f (x)|2 dx; (iv) 0 | f s (p)|2 d p = 0 | f (x)|2 dx;
R∞ sin2 ax πa
and hence prove that 0 x2
dx = 2
.
Solution. r
2 sin ap
F [ f (x)] = f (p) = . Refer to Example 1.9.1.
π p
Using Parseval’s identity for Fourier transform, we get
Z ∞ Z ∞
| f (x)| dx = 2
| f (p)|2 d p
−∞ −∞
Z ∞
2 sin2 ap
Z a
(1)2 dx = dp
−a −∞ π p2
2 ∞ sin2 ap
Z
2a = dp
π −∞ p2
4 ∞ sin2 ap
Z
2a = dp
π 0 p2
πa
Z ∞ 2
sin ap
2
dp = .
0 p 2
R ∞ sin x 4
Hence deduce that 0 x
dt = π3 .
2 h cos p 1 i
= √ 0− 2 − 0− 2
2π p p
2 1 − cos p
h i
= √
2π p2
r
2 h 1 − cos p i
F [ f (x)] = f (p) = .
π p2
Put p
2
= x. Then d p = 2dx
∞
π
Z
1
4
sin4 x2dx =
0 (2x) 24
π
Z ∞ 4
1 sin x
dx =
8 0 x4 24
π
Z ∞ 4
sin x
4
dx = .
0 x 3
e−2ax ∞ Z ∞
2a2 1
= dp
−2a 0 π 0 (p + a )
2 2 2
Z ∞
1 2a2 1
= dp
2a π 0 (p + a )
2 2 2
∞
π
Z
1
dp = .
0 (p + a )
2 2 2 4a3
Solution.
4 sin p − p cos p
F [ f (x)] = f (p) = √ , Refer to Example 1.9.2.
2π p3
Using Parseval’s identity for Fourier transform, we get
Z ∞ Z ∞
| f (x)| dx =
2
| f (p)|2 d p
−∞ −∞
Z 1 Z ∞
16 sin p − p cos p 2
(1 − x2 )2 dx = dp
−1 −∞ 2π p3
1
8 ∞ sin p − p cos p 2
Z Z
2 (1 + x4 − 2x2 )dx = dp
0 π −∞ p3
x5 x 3 i1 16 ∞ sin p − p cos p 2
h Z
2 x+ −2 = dp
5 3 0 π 0 p3
16 ∞ sin p − p cos p 2
Z
16
= dp
15 π 0 p3
π
Z ∞
sin p − p cos p 2
dp =
p3 15
Z0 ∞
sin x − x cos x 2 π
i.e., dx = .
0 x3 15
R∞ π
Example 18.3.6. Using Parseval’s identity, show that 0 dx
(x2 +a2 )(x2 +b2 )
= 2ab(a+b)
.
R∞ π
(or) Evaluate 0 (x2 +a2dx
)(x2 +b2 )
= 2ab(a+b) using transform.
18.4 Exercises
Example 18.5.1. Find the finite Fourier sine and cosine transform of f (x) = 1 in 0 < x < π.
Solution. (i)
Z l pπx
F s [ f (x)] = f (x) sin
dx
l
Z0 π pπx
f s (p) = 1. sin dx
0 π
(ii)
Z l pπx
Fc [ f (x)] = f (x) cos
dx
0 l
Z π pπx
f c (p) = 1. cos dx
0 π
Z π
= cos pxdx
0
sin px π sin pπ
= =
p 0 p
Fc [ f (x)] = 0
Example 18.5.2. Find the finite Fourier sine and cosine transform of f (x), defined by f (x) = x,
where 0 < x < 4.
Solution. (i)
Z l pπx
F s [ f (x)] = f (x) sin dx
0 l
Z 4 pπx
f s (p) = x. sin dx
0 4
h cos pπx sin pπx i4
= x − pπ 4
− 1. − p2 π24
0
4 16
16 16
= − cos pπ + 2 2 sin pπ
pπ pπ
16
F s [ f (x)] = − cos pπ
pπ
(ii)
Z l pπx
Fc [ f (x)] = f (x) cos
dx
0 l
Z 4 pπx
f c (p) = x. cos dx
0 4
h sin pπx cos pπx i4
Fc [ f (x)] = x pπ 4
− 1. − p2 π24
0
4 16
16 16 16
Fc [ f (x)] = sin pπ + 2 2 cos pπ − 2 2
pπ pπ pπ
16
Fc [ f (x)] = 2 2 [(−1) p − 1], whenp > 0
pπ
and if p = 0, Z 4
Fc [ f (x)] = xdx = 8.
0
Example 18.5.3. Find the finite Fourier sine and cosine transforms of f (x), defined by f (x) = 2x,
where 0 < x < 2π
Solution. (i)
Z l pπx
F s [ f (x)] = f (x) sin
dx
0 l
Z 2π px
f s (p) = 2x. sin dx
0 2
h cos px sin px i2π
= 2x − p 2
− 1. − p2 2
0
2 4
h 4π 4 i
= 2 − cos pπ + 2 sin pπ
p p
8π
= (−1) p+1 , if p = 1, 2, 3, ...
p
(i)
Z l pπx
Fc [ f (x)] = f (x) cos
dx
0 l
Z 2π px
f c (p) = 2x. cos dx
0 2
h sin px cos px i2π
= 2x p 2
− 1. − p2 2
0
2 4
h 4π 4 4i
= 2 sin pπ + cos pπ − 0 +
p p2 p2
8
= 2 (cos pπ − 1)
p
8
= 2 ((−1)n − 1), if p = 1, 2, 3, ...
p
Example 18.5.4. Find the finite sine Fourier sine transform of f (x) = x3 in (0, π)
Solution. Z l pπx
F s [ f (x)] = f (x) sin dx
l
Z0 π
f s (p) = x3 . sin pxdx
0
h cos px sin px cos px sin px iπ
= x3 − − 3x2 − + 6x − 6
p p2 p3 p4 0
π 3
3π 2
6π 6
= − cos pπ + 2 sin pπ + 3 cos pπ − 4 sin pπ
p p p p
π3 6π
F s [ f (x)] = − + 3 (−1) p , i f p = 1, 2, 3, ..
p p
2
Example 18.5.5. (i) Find the finite Fourier sine transform of f (x) defined by f (x) = 1 − πx ,
where 0 < x < π.
(ii) Show that the finite sine transform of 1 − x
π
is 1p .
Solution. (i)
Z l pπx
F s [ f (x)] = f (x) sin dx
0 l
2 cos pπ 1 2 1
= − − + 2. 3
π p
2 3 p π p
2 1
= (cos π − 1) +
pπ
3 2 p
2 1
F s [ f (x)] = ((−1) n
− 1) +
p3 π2 p
Z l pπx
F s [ f (x)] = f (x) sin dx
l
Z0 π
x
f s (p) = 1 − . sin pxdx l = π
0 π
h x cos px 1 sin px iπ
= 1− − − − −
π p π p2 0
h 1 i
F s [ f (x)] = 0 − − − 0
p
1
F s [ f (x)] = .
p
Example 18.5.6. Find the finite Fourier sine and cosine transforms of f (x) = sin ax in (0, π) .
If a = p, then
Z π
F s [ f (x)] = sin2 xdx, since a = p
0
1 π
Z
= (1 − cos 2px)dx
2 0
1h sin 2px π
= x−
2 2p 0
1
= (π − 0).
2
0, if a , n and a, p are integers
∴ F s [sin ax] =
π2 , if a = p
π x2
Example 18.5.7. Find the Fourier cosine transform of f (x) defined by f (x) = 3
−x+ 2π
, where
0 < x < π.
Solution. Z l pπx
Fc [ f (x)] = f (x) cos dx
0 l
Z π
π x2
f c (p) = −x+ cos pxdx, l = π
0 3 2π
h π x2 sin px x cos px 1 sin px iπ
= −x+ − −1+ − + −
3 2π p π p2 π p3 0
1
= 0+0−0− − 2 +0
p
1
Fc [ f (x)] = , p > 0.
p2
If p = 0,
π π x2
Z
Fc [ f (x)] = −x+ dx = 0.
0 3 2π
Example 18.5.8. Find the finite Fourier sine transform of f (x), defined by
π
x, 0≤x≤
2
f (x) =
π
π − x, ≤x≤π
2
Solution. Z l pπx
F s [ f (x)] = f (x) sin dx
l
Z0 π
f s (p) = f (x) sin pxdx, l = π
0
π pπ 1 pπ π pπ 1 pπ
= − cos + 2 sin + cos + 2 sin
2p 2 p 2 2p 2 p 2
2 pπ
F s [ f (x)] = 2 sin .
p 2
Example 18.5.9. Find the finite Fourier sine and cosine transform of (i) f (x) = x2 , 0 < x < l
(ii)
π
0<x<
1,
2
f (x) =
π
<x<π
−1,
2
2l3
=
cos pπ
p2 π2
2l3
Fc [ f (x)] = 2 2 (−1) p , p , 0.
pπ
1 pπ 1 pπ
= − cos −1 + cos pπ − cos
p 2 p 2
1 pπ
F s [ f (x)] = − 2 cos − cos pπ − 1 , p , 0.
p 2
2 pπ
Fc [ f (x)] = sin .
p 2
(−1) p+1
Example 18.5.10. Show that the finite Fourier sine transform of x
π
is p
, 0<x<π
Solution. Z l pπx
F s [ f (x)] = f (x) sin dx
l
Z0 π
x
= sin pxdx
0 π
1 h π i
= − cos pπ
π p
p+1
(−1)
F s [ f (x)] = .
p
π4 ,
if p = 0
π
Solution. (i) Here f c (0) = 4
and l = 2π. From the inverse finite Fourier sine transform, we have
∞
1 2X pπx
f (x) = f c (0) + f c (p) cos
l l p=1 l
∞
1 π 2 X 1 pπ pπx
= . + sin cos
2π 4 2π p=1 2p 2 2π
∞
1 1 X1 pπ pπx
f (x) = + + sin cos .
8 2π p=1 p 2 2π
∞
1 2X 1 2pπ
f (x) = + cos cos pπx
l l p=1 (2p + 1) 2 3
∞
X 1 2pπ
f (x) = 1 + 2 cos cos pπx.
p=1
(2p + 1) 2 3
Solution.
π
π x2
Z
Fc [ f (x)] = cos pxdx −
0 2π 6
h x2 π sin px 2x − cos px 2 sin px iπ
= − ( )− −0 + −
2π 6 p 2π p2 2π p3 0
cos pπ
=
p2
(−1) p
Fc [ f (x)] = , p , 0.
p2
When p = 0,
π x2 π
Z
Fc [ f (x)] = − dx = 0.
0 2π 6
Example 18.5.13. Find the inverse finite sine transform f (x) if f s (p) = 1−cos pπ
p2 π2
, where 0 < x <
π.
∞
2X pπx
f (x) = f s (p) sin
l p=1 l
∞
2 X 1 − cos pπ
= sin px
π p=1 p2 π2
∞
2 X 1 − cos pπ
= 3 sin px.
π p=1 p2
2pπ
Example 18.5.14. Find the inverse finite cosine transform f (x) if f c (p) = 1
(2p+1)2
cos 3
, where
0 < x < 4.
∞
1 2X pπx
f (x) = f c (0) + f c (p) cos
l l p=1 l
∞
1 2 2X 1 2pπ pπx
= .1 + + cos cos
4 4 l p=1 (2p + 1) 2 3 4
∞
1 1 2X 1 2pπ pπx
f (x) = + + cos cos
4 2 l p=1 (2p + 1) 2 3 4
Example 18.5.15. Find the finite Fourier sine and cosine transforms of f (x) = eax , in (0, l).
1 pπ al
F s [ f (x)] = p2 π2
[e (−1) p+1 + 1]
a2 + l 2 l
pπl
= 22 [(−1) p+1 eal + 1].
a l + p2 π2
eal 1
= p2 π2
a cos pπ + 0 − p2 π2
(a + 0)
a2 + l2
a2 + l2
a
= p2 π2
[eal (−1) p − 1]
a2 + l2
2
al
= [(−1) p eal − 1].
a2 l2 + p2 π2
2
Example 18.5.16. Find the finite Fourier cosine transform of f (x) = 1 − πx in (0, π).
Solution. Z l pπx
Fc [ f (x)] = f (x) cos dx
0 l
Z π
x 2
f c (p) = 1− . cos pxdx l = π
0 π
h x 2 sin px x 1 cos px 2 sin px iπ
= 1− −2 1− − − + 2 −
π p π π p2 π p3 0
2
Fc [ f (x)] = , if p > 0
πp2
When p = 0,
Z π
f c (0) = f (x)dx
0
Z π x 2
= 1− dx
0 π
π
f c (0) = .
3
16(−1) p−1
Example 18.5.17. Find the inverse finite Fourier sine transform f (x) if f s (p) = p3
, where
p is a positive integer and 0 < x < 8.
∞
2X pπx
f (x) = f s (p) sin
l p=1 l
∞
2 X 16(−1) p−1 pπx
= 3
sin
8 p=1 p 8
∞
X (−1) p−1 pπx
f (x) = 4 sin .
p=1
p3 8
Example 18.5.18. Find the finite Fourier sine and cosine transforms of
π
if 0 < x <
1,
2
f (x) =
π
<x<π
−1,
if 2
1 pπ 1 pπ
= − cos +1 + cos pπ − cos
p 2 p 2
1 pπ
= 1 − 2 cos + (−1) p , p = 1, 2, 3, ...
p 2
If p = 0, then
F s [ f (x)] = 0.
If p = 0, then π
Z 2
Z π
Fc [ f (x)] = 1.dx + (−1)dx = 0.
π
0 2
18.6 Exercises
3. Find the finite Fourier sine and cosine transform of f (x) defined by
(i) f (x) = x, 0 < x < π (ii) f (x) = 2x, 0 < x < 4 (iii) f (x) = x2 , 0 < x < 4.
4. Find the finite Fourier sine and cosine transform of f (x) = x(π − x), 0 < x < π.
5. (a) Find the finite Fourier sine and cosine transform of cos ax in (i) (0, l) and (ii) (0, π).
(b) Find the finite Fourier sine transform of cos kx.
2π(−1) p−1
6. Find inverse finite sine transform of f (x) of p3
, p = 1, 2, 3, .. where 0 < x < π.
7. Find inverse finite cosine transform of f (x) of 6
π(2p+1)
sin pπ
2
−cos pπ . for p = 1, 2, 3, .. where
0 < x < 4.
8. Find inverse finite cosine transform of f (x) of 1
(2p+1)2
cos 2pπ
3
, if 0 < x < 1, p = 1, 2, 3, ...
Answers
p
1. π3 when p = 0 and 6π(−1) p2
, p = 1, 2, 3, ....
2. πp + p22 cos pπ − p12 πp + p12 cos pπ.
2
Lesson 19
APPLICATIONS TO ORDINARY
DIFFERENTIAL EQUATIONS
WITH CONSTANT COEFFICIENTS
Learning Objectives
Upon completion of this lesson, students will be able to
• use Laplace transform to solve ordinary differential equations with constant coefficients
19.1 Introduction
O
rdinary differential equations with constant coefficients can be easily solved by the Laplace
transform method, without the necessity of first finding the general solutions and then
evaluating the arbitrary constants. This method is, in general, shorter than our earlier methods
and is especially suitable to obtain the solution of linear non - homogeneous ordinary differential
equations with constant coefficients.
dn y dn−1 dn−2 dy
+ a1 + a2 + · · · + an−1 + an y = F(t), (19.1.1)
dtn dtn−1 dtn−2 dt
be the given initial or boundary conditions, where c0 , c1 , c2 , ..., cn−1 are constants.
If a1 , a2 , a3 , ...., an are constants, then we use
Taking Laplace transform of both sides of (19.1.1) and applying (19.1.3) and using conditions
(19.1.2) we obtain an algebraic equation known as subsidiary equation from which y(s) = L [y(t)]
is obtained. The required solution y(t) is obtained by taking inverse Laplace transform of y(s).
Working Rule to Solve Differential Equation by Laplace Transform Method.
1. We take the Laplace transforms of both sides of the given differential equation in y(t),
simultaneously using the given initial conditions. This gives an algebraic equation in y(s) =
L [y(t)].
2. We solve the algebraic equation to get y(s) as a function of s.
3. Finally we take L −1 [y(s)] to get y(t). The various methods we have discuss in UNIT -III will
enable us to find L −1 [y(s)].
The procedure is illustrated in the worked examples given below:
Example 19.2.1. Using Laplace transform, solve (D2 + 4D + 5)y = 5, given that y(0) = 0,
y0 (0) = 0.
y00 + 4y0 + 5y = 5.
i.e., p2 L [y] − p.y(0) − y0 (0) + 4[pL [y] − y(0)] + 5L [y] = 5.L [1].
Given
y(0) = 0 = y0 (0)
5
p2 L [y] + 4pL [y] + 5L [y] =
p
5
(p2 + 4p + 5)L [y] =
p
5
L [y] =
p(p2 + 4p + 5)
1 p+4
L [y] = − 2 (Resolving into partial f ractions)
p p + 4p + 5
h1 p+4 i
∴y = L −1 − 2
p p + 4p + 5
h1i h p+4 i
= L −1 − L −1 2
p p + 4p + 5
h p+2+2 i
= 1 − L −1
(p + 2)2 + 1
h p+2 i h 2 i
= 1 − L −1 + L −1
(p + 2)2 + 1 (p + 2)2 + 1
h p i h 1 i
= 1 − e−2t L −1 2 + 2e−2t L −1 2 ,
(p + 1 p +1
= 1 − e−2t cost + 2e−2t sin t
Example 19.2.2. Solve the following initial value problem by using Laplace transform
4y00 + π2 y = 0, y(0) = 2, y0 (0) = 0.
Solution. Given
4y00 + π2 y = 0,
4L [y00 ] + π2 L [y] = 0.
Given
y(0) = 2 and y0 (0) = 0
(4p2 + π2 )L [y] = 8p
8p
L [y] =
4p2 + π2
8p
=
π2
4 p2 + 4
h p i
∴ y = 2L −1 π2
p2 + 4
πt
y = 2 cos .
2
d3 y 2
Example 19.2.3. Solve by Laplace transform dt3
+2 ddt2y − dy
dt
−2y = 0, y(0) = 1, y0 (0) = y00 (0) = 2.
y000 + 2y00 − y0 − 2y = 0
p3 L [y] − p2 y(0) − py0 (0) − y00 (0) + 2[p2 L [y] − py(0) − y0 (0)] − [pL [y] − y(0)] − 2L [y] = 0.
Given
y(0) = 1 and y0 (0) = y00 (0) = 2.
p 2 1
= + 2 +
p − 1 p − 1 (p + 2)(p − 1)(p + 1)
2
p 2 1 1 1 1
L [y] = + 2 + − +
p − 1 p − 1 3(p + 2) p − 1 6 p − 1
2
h p 2 1 1 1 1 i
∴y = L −1 2 + 2 + − +
p − 1 p − 1 3(p + 2) p − 1 6 p − 1
1 1 1
y = cos ht + 2 sinh t + e−2t − e−t + et .
3 2 6
d2 y
Example 19.2.4. Using Laplace transform, solve dt2
+ 2 dy
dt
− 3y = sin t, y = dy
dt
= 0, when t = 0.
1
p2 L [y] − py(0) − y0 (0) + 2[pL [y] − y(0)] − 3L [y] = .
p2 +1
Given
y(0) = 0 and y0 (0) = 0.
1
∴ p2 L [y] + 2pL [y] − 3L [y] = .
p2 +1
1
L [y](p2 + 2p − 3) =
p2 +1
1
L [y] =
(p2 + 1)(p2 + 2p − 3)
1
=
(p2 + 1)(p + 3)(p − 1)
−p
− 15 1 1
= 10
− + , resolving into partial f ractions
p + 1 40(p + 3) 8(p − 1)
2
1 p 1 1 1 1
L [y] = − − − +
10 p + 1 5 p + 1 40(p + 3) 8(p − 1)
2 2
h 1 p 1 1 1 1 i
∴y = L −1 − − − +
10 p + 1 5 p + 1 40(p + 3) 8(p − 1)
2 2
1 1 1 1
y = − cos t − sin t − e−3t + et .
10 5 40 8
Example 19.2.5. Using Laplace transform method, solve (D2 + 1)y = 6 cos 2t, t > 0, if y = 3,
Dy = 1 when t = 0.
Solution. Given
y00 + y = 6 cos 2t
6p
p2 L [y] − py(0) − y0 (0) + L [y] = .
p2+4
Given
y(0) = 3 and y0 (0) = 1.
6p
∴ p2 L [y] − 3p − 1 + L [y] = .
p2 + 4
1
L [y](p2 + 1) = + 1 + 3p
p2 +1
6p 1 3p
L [y] = + 2 + 2
(p + 4)(p + 1) p + 1 p + 1
2 2
h p p i 1 3p
= 2 2 − 2 + 2 + 2
p +1 p +4 p +1 p +1
5p 2p 1
L [y] = − +
p2 + 1 p2 + 4 p2 + 1
h 5p 2p 1 i
y = L −1 2 − 2 + 2
p +1 p +4 p +1
y = 5 cos t − 2 cos 2t + sin t.
d2 x
Example 19.2.6. Solve the differential equation dt2
− 4 dx
dt
− 12x = e3t given that x(0) = 1 and
x0 (0) = −2 using Laplace transform.
1
p2 L [x] − px(0) − x0 (0) − 4[pL [x] − x(0)] − 12L [x] = .
p−3
Given
x(0) = 1 and x0 (0) = −2.
1
∴ p2 L [x] − p + 2 − 4[L [x] − 1] − 12L [x] = .
p−3
1
L [x](p2 − 4p − 12) = + p−6
p−3
1 p−6
L [x] = 2
+ 2
(p − 2p − 12)(p − 3) p − 2p − 12
1 p−6
L [x] = +
(p + 2)(p − 6)(p − 3) (p + 2)(p − 6)
1 1 1 1 1 1 1
L [x] = − + + ,
40 p + 2 15 p − 3 24 p − 6 p + 2
by resolving into partial f raction
h1 1 1 1 1 1 1 i
x = L −1 − + +
40 p + 2 15 p − 3 24 p − 6 p + 2
1 −2t 1 3t 1 6t
x = e − e + e + e−2t
40 15 24
41 −2t 1 3t 1 6t
x = e − e + e .
40 15 24
d2 x
Example 19.2.7. Solve the differential equation using Laplace transform dt2
+ 3 dx
dt
+ 2x = e−t
given that x(0) = 0 and x0 (0) = 1.
1
L [x](p2 + 3p + 2) = +1
p+1
p+2
L [x](p2 + 3p + 2) =
p+1
p+2
L [x] =
(p + 1)(p2 + 3p + 2)
p+2
=
(p + 1)(p + 1)(p + 2)
1
L [x] =
(p + 1)2
h 1 i
x = L −1
(p + 1)2
x = te−t , by shi f ting theorem.
d2 x
Example 19.2.8. Solve the differential equation dt2
+ 9x = sin t using Laplace transform given
that x(0) = 1 and x π2 = 1.
x00 + 9x = sin t.
1
p2 L [x] − px(0) − x0 (0) + 9L [x] = .
p2 +1
π
x(0) = 1 and x = 1.
2
1
∴ p2 L [x] − p − A + 9L [x] = .
p2 +1
1
L [x](p2 + 9) = + p+A
p2 +1
1 p A
L [x] = + 2 + 2
(p + 1)(p + 9) p + 9 p + 9
2 2
1h 1 1 p A
= − + +
8 p2 + 1 p2 + 9 p2 + 9 p2 + 9
h1h 1 1 p A i
x = L −1 − + +
8 p2 + 1 p2 + 9 p2 + 9 p2 + 9
1h sin 3t i A
x = sin t − + cos 3t + sin 3t
8 3 3
π
Given x 2
= 1.
sin 3 π2 i
1h π π A π
1 = sin − + cos 3 + sin 3
8 2 3 2 3 2
1 1 A
1 = [1 + ]+0−
8 3 3
5
A = −
2
1h sin 3t i 5 1 7
∴x= sin t − + cos 3t − sin 3t = sin t + cos 3t − sin 3t.
8 3 6 8 6
Example 19.2.9. Solve the differential equation y00 + n2 y = a sin(nt + 2) given that y(0) = 0 and
y0 (0) = 0 using Laplace transform.
n p
p2 L [y] − py(0) − y0 (0) + n2 L [y] = a cos 2 + a sin 2 2 .
p2 +n2 p + n2
Given y(0) = 0 andy0 (0) = 0.
n p
∴ p2 L [y] + n2 L [y] = a cos 2 + a sin 2 2 .
p2 +n 2 p + n2
n p
L [y](p2 + n2 ) = a cos 2 + a sin 2 2 .
+n 2p2 p + n2
n p
L [x] = a cos 2 2 + a sin 2 2
(p + n )2 2 (p + n2 )2
h n i h p i
x = an cos 2L −1 2 + a sin 2L −1
(p + n2 )2 (p2 + n2 )2
We have
h p i t
L −1 = sin nt.
(p + n )
2 2 2 2n
Since
h f (p) i Z t
L −1
= F(t)dt
p 0
Z t
h 1 i
−1 1
h p i 1 1
∴L −1
= L = t sin ntdt = [−nt cos nt + sin nt]
(p + n )
2 2 2 p (p + n )
2 2 2 2n 0 2n3
1 t
x = an cos 2. 3
[−nt cos nt + sin nt] + a sin 2. sin nt
2n 2n
a
= [−nt cos 2 cos nt + cos 2 sin nt + nt sin 2 sin nt]
2n2
a
= [cos 2 sin nt − nt(cos nt cos 2 − sin nt sin 2]
2n2
a
x = [cos 2 sin nt − nt cos(nt + 2)]
2n2
Example 19.2.10. Using Laplace transform method solve the differential equation (D2 + 1)y =
cos t − cos 3t
y00 + y = .
2
1 1
L [y00 ] + L [y] = L [cos t] − L [cos 3t].
2 2
1 p 1 p
p2 L [y] − py(0) − y0 (0) + L [y] = − .
2 p + 1 2 p2 + 9
2
1 p 1 p
L [y](p2 + 1) = − +p
2 p2 + 1 2 p2 + 9
1 p 1 p p
L [y] = − + 2
2 (p + 1)
2 2 2 (p + 1)(p + 9) p + 1
2 2
1 p 1h p p i p
= − − + 2
2 (p + 1)
2 2 16 (p + 1) p + 9
2 2 p +1
h1 p 1 h p p i p i
y = L −1 − − +
2 (p2 + 1)2 16 (p2 + 1) p2 + 9 p2 + 1
1 t 1 h p i t
= . sin t − [cos t − cos 3t] + cos t, since L −1 2 = sin at
2 2 16 (p + a )
2 2 2a
t 15 1
y = sin t + cos t + cos 3t.
4 16 16
d2 y
Example 19.2.11. Solve the differential equation using Laplace transform dt2
+2 dy
dt
+5y = e−t sin t
given that y(0) = 0 and y0 (0) = 1.
1
p2 L [y] − py(0) − y0 (0) + 2[pL [y] − y(0)] + 5L [y] = .
(p + 1)2 + 1
Given
y(0) = 0 and y0 (0) = 1.
1
∴ p2 L [y] − 1 + 2[pL [y] − 0] + 5L [y] = .
(p + 1)2 + 1
1
L [y](p2 + 2p + 5) = +1
p2 + 2p + 2
p2 + 2p + 3
L [y] =
(p2 + 2p + 2)(p2 + 2p + 5)
1 1 2 1
L [y] = + . 2
3 p + 2p + 2 3 p + 2p + 5
2
h1 1 2 1 i
y = L −1 + .
3 p2 + 2p + 2 3 p2 + 2p + 5
1 −1 h 1 i 2 h 1 i
= L + L −1
3 (p + 1)2 + 1 3 (p + 1)2 + 4
1 −t −1 1 h i 2 −t −1 1 ih
= e L + e L
3 p2 + 1 3 p2 + 4
1 −t 2 sin 2t
= e sin t + e−t
3 3 2
e−t
y = (sin t + sin 2t).
3
Example 19.2.12. Using Laplace transform method solve the differential equation (D2 + 1)x =
t cos 2t, given that x = 0 and dx
dt
= 0 at t = 0.
d p
p2 L [x] − px(0) − x0 (0) + L [x] = −
dp p + 4
2
p2 − 4
L [x](p2 + 1) =
(p2 + 4)2
p2 − 4
L [x] =
(p2 + 1)(p2 + 4)2
5 1 8 1 5 1
= + − , by resolving into partial f ractions
9 p + 4 3 (p + 4)
2 2 2 9 p2 + 1
h5 1 8 1 5 1 i
x = L −1 + −
9 p2 + 4 3 (p2 + 4)2 9 p2 + 1
5 1 8 h1 p i 5
= . sin 2t + L −1 . 2 − sin t
9 2 3 p (p + 4)2 9
Z t
5 8 u 5 h p i t
= sin 2t + sin 2udu − sin t, L −1 2 = sin at
18 3 0 4 9 (p + a )
2 2 2a
5 2 h − cos 2u sin 2u it 5
= sin 2t + u − 1. − − sin t
18 3 2 4 0 9
5 2h t sin 2t i 5
= sin 2t + − cos 2t + − sin t
18 3 2 4 9
4 5 t
x = sin 2t − sin t − cos 2t.
9 9 3
d2 x
Example 19.2.13. Solve dt2
+ 2 dx
dt
+ x = 3te−t given that x(0) = 4 and dx
dt
= 0 at t = 0.
3
p2 L [x] − px(0) − x0 (0) + 2[pL [x] − x(0)] + L [x] = .
(p + 1)2
Given
x(0) = 4 and x0 (0) = 0.
3
∴ p2 L [x] − 4p + 2[pL [x] − 4] + L [x] = .
(p + 1)2
3
L [x](p2 + 2p + 1) = + 4p + 8
(p + 1)2
3
L [x]((p + 1)2 ) = + 4p + 8
(p + 1)2
3 p 8
L [x] = +4 +
(p + 1) 4 (p + 1) 2 (p + 1)2
h 3 p+1−1 8 i
x = L −1 + 4 +
(p + 1)4 (p + 1)2 (p + 1)2
h1i h 1 1 i −1 1
h i
x = 3e−t L −1 + 4L −1
− + 8e−t
L
p4 p + 1 (p + 1)2 p2
t3
= 3e−t . + 4e−t (1 − t) + 8e−t t
3!
h t3 i
x = e−t + 4t + 4 .
2
Solution. Given
y0 + y = 0
L [y0 ] + L [y] = 0
pL [y] − y0 + L [y] = 0
Given y(0) = y0
L [y](p + 1) = y0
y0
=
p+1
1
y = y0 L −1
p+1
y = y0 e−t .
1 1
p2 L [y00 ] − py(0) − y0 (0) − 3[pL [y] − y(0)] + 2L [y] = − .
p p−2
Given
y(0) = 1 and y0 (0) = 0.
1 1
∴ p2 L [y] − p − 3[pL [y] − 1] + 2L [y] = − .
p p−2
1 1
L [y](p2 − 3p + 2) = − + p−3
p p−2
1 1
L [y]((p − 1)(p − 2)) = − + p−3
p p−2
y00 + y0 = t2 + 2t.
2 2
p2 L [y00 ] − py(0) − y0 (0) + pL [y] − y(0) = 3
+ 2.
p p
Given
y(0) = 4 and y0 (0) = −2.
2 2
∴ p2 L [y] − 4p + 2 + pL [y] − 4 = 3
+ 2.
p p
2 2
L [y](p2 + p) = + + 4p + 2
p3 p2
2 + 2p + 4p4 + 2p3
L [y](p + p)) =
2
p3
2 + 2p + 4p4 + 2p3
L [y] =
p4 (p + 1)
2 2 2
L [y] = 4 + +
p p p+1
h2 2 2 i
y = L −1 + +
p4 p p + 1
2t3
y = + 2 + 2e−t
3!
t3
y = + 2 + 2e−t .
3
y00 + 2y0 + 2y = 0.
Given
y(0) = 1 and y0 (0) = 1.
L [y](p2 − 2p + 2) = p − 1
p−1
L [y] =
p2− 2p + 2
p−1
L [y] =
(p − 1)2 + 1
h p−1 i
y = L −1
(p − 1)2 + 1
h p i
y = et L −1 2
p +1
y = et cos t, by using f irst shi f ting theorem.
y00 + 2y0 + y = t.
1
p2 L [y00 ] − py(0) − y0 (0) + 2[pL [y] − y(0)] + L [y] = .
p2
Given
y(0) = −3 and y(1) = −1.
Let y0 (0) = A
1
∴ p2 L [y] + 3p − A + 2pL [y] + 6 + L [y] = .
p2
1
L [y](p2 + 2p + 1) = + A − 3p − 6
p2
1
L [y](p + 1)2 = 2 + A − 3p − 6
p
1 A 3p + 6
L [y] = + −
p2 (p + 1)2 (p + 1)2 (p + 1)2
2 1 A 3 3
= − + 2+ − − ,
p p (p + 1) 2 p + 1 (p + 1)2
2 1 1 A−2
L [y] = − + 2 − +
p p p + 1 (p + 1)2
h 2 1 1 A−2 i
y = L −1 − + 2 − +
p p p + 1 (p + 1)2
y = −2 + t − e−t + (A − 2)e−t t
∴ y = −2 + t − e−t + te−t .
1
p2 L [y00 ] − py(0) − y0 (0) + 6[pL [y] − y(0)] + 9L [y] = .
1 + p2
Given
y(0) = 1 and y0 (0) = 0.
1
∴ p2 L [y] − p + 6pL [y] − 6 + 9L [y] = .
p2 + 1
2
p2 L [y00 ] − py(0) − y0 (0) − [pL [y] − y(0)] − 2L [y] = 20. .
p2 +4
Given
y(0) = −1 and y0 (0) = 2.
40
∴ p2 L [y] + p − 2 − pL [y] − 1 − 2L [y] = .
p2 +4
40
L [y](p2 − p − 2) = +3− p
+4
p2
40
L [y](p − 2)(p + 1) = 2 +3− p
p +4
40 3− p
L [y] = +
(p − 2)(p + 1)(p2 + 4) (p − 2)(p + 1)
3 1 8 1 p−6 1 1 4 1
= − + 2 + . −
5p−2 3p+1 p +4 3 p−2 3p+1
by resolving into partial fractions
2 4 p 6
L [y] = − + 2 − 2
p−2 p+1 p +4 p +4
h 2 4 p 6 i
y = L −1 − + 2 − 2
p−2 p+1 p +4 p +4
y = 2e2t − 4e−t + cos 2t − 3 sin 2t
y0 + y = 1.
1
L [y0 ] + L [y] = .
p
1
pL [y] − y(0) + L [y] = .
p
Given y(0) = 2
1
∴ pL [y] − 2 + L [y] = .
p
1
L [y](p + 1) = +2
p
1 2
L [y] = +
p(p + 1) p + 1
dy
Example 19.2.22. Solve dt2
+ y = 0, under the conditions that y = 1, dy
dt
= 0, when t = 0
y00 + y = 0.
L [y00 ] + L [y] = 0.
Given
y(0) = 1 and y0 (0) = 0.
∴ p2 L [y] − p + L [y] = 0.
L [y](p2 + 1) = p
p
L [y] =
+1
p2
h p2 i
y = L −1 2
p +1
y = cos t
y00 + 9y = 18t.
18
p2 L [y00 ] − py(0) − y0 (0) + 9L [y] =
p2
Given
y(0) = 0 and let y0 (0) = A.
18
∴ p2 L [y] − A + 9L [y] =
p2
18
L [y](p2 + 9) = +A
p2
18 A
L [y] = 2 2 + 2
p (p + 9) p + 9
1 2 A
L [y] = − 2 + 2
p 2 p +9 p +9
h1 2 A i
y = L −1 2 − 2 + 2
p p +9 p +9
1
y = 2t + (A − 2) sin 3t
3
Given y π2 = 0
π 1
0 = 2 + (A − 2) (−1) ⇒ A = 2 + 3π
2 3
∴ y = 2t + π sin 3t.
y00 − y0 − 6y = 2.
2
p2 L [y00 ] − py(0) − y0 (0) − [pL [y] − y(0)] − 6L [y] = .
p
Given
y(0) = 1 and let y0 (0) = 0.
2
∴ p2 L [y] − p − pL [y] + 1 − 6L [y] = .
p
2
L [y](p2 − p − 6) = + p−1
p
1
L [y]((p − 3)(p + 2)) = + p−1
p
1 p−2
L [y] = +
p(p − 3)(p + 2) (p − 3)(p + 2)
p2 − p + 2
=
p(p − 3)(p + 2)
11 8 1 4 1
= − + +
3 p 15 p − 3 5 p + 2
h 11 8 1 4 1 i
y = L −1 − + +
3 p 15 p − 3 5 p + 2
1 8 4
y = − + e3t + e−2t .
3 15 5
d2 x
Example 19.2.25. Solve dt2
+ x = t, x0 = 1, x1 = 2.
x00 + x = t.
1
p2 L [x00 ] − px(0) − x0 (0) + L [x] = .
p2
Given
x(0) = 1, and x0 (0) = 2.
1
∴ p2 L [x] − p − 2 + L [x] = .
p2
1
L [x](p2 + 1) = + p+2
p2
1 p+2
L [x] = + 2
p (p + 1) p + 1
2 2
1 1 p+2
L [x] = − +
p2 p2 + 1 p2 + 1
1 1 p
= + 2 + 2
p 2 p +1 p +1
h1 1 p i
x = L −1 2 + 2 + 2
p p +1 p +1
x = t + sin t + cos t
x00 + x0 = 2.
2
p2 L [x00 ] − px(0) − x0 (0) + pL [x] − x(0) = .
p
Given
x(0) = 3 and let x0 (0) = 1.
2
∴ p2 L [x] − 3p − 1 + pL [x] − 3 = .
p
2
L [x](p2 + p) = + 3p + 4
p
2 3p 4
L [x] = + +
p (p + 1) p(p + 1) p(p + 1)
2
2 + 3p2 + 4p
L [x] =
p2 (p + 1)
2 1 1
= + 2+
p 2 p p+1
h2 1 1 i
x = L −1 2 + 2 +
p p p+1
x = 2 + 2t + e−t .
5
p2 L [y00 ] − py(0) − y0 (0) + 5pL [y] − 5y(0) + 6L [y] = .
p−1
Given
y(0) = 2 and y0 (0) = 1.
5
∴ p2 L [y] − 2p − 1 + 5pL [y] − 10 + 6L [y] = .
p−1
5
L [y](p2 + 5p + 6) = + 2p + 11
p−1
5
L [y]((p + 3)(p + 2)) = + 2p + 11
p−1
5 2p + 11
L [y] = +
(p − 1)(p + 2)(p + 3) (p + 3)(p + 2)
5 1 1 1 5 1 5 7
L [y] = + + − +
12 p − 1 3 p + 2 4 p + 3 p + 3 p + 2
5 1 22 1 15 1
= + −
12 p − 1 3 p+2 4 p+3
h5 1 22 1 15 1 i
y = L −1 + −
12 p − 1 3 p+2 4 p+3
5 t 22 −2t 15 −3t
y = e + e − e .
12 3 4
Y 00 + 4Y 0 + 4Y = 4e−2t .
L [Y 00 ] + 4L [Y 0 ] + 4L [Y] = 4L [e−2t ].
4
p2 L [Y 00 ] − pY(0) − Y 0 (0) + 4pL [y] − 4Y(0) + 4L [Y] = .
p+2
Given
Y(0) = −1 and Y 0 (0) = 4.
4
∴ p2 L [Y] + p − 4 + 5pL [y] + 4 + 4L [Y] = .
p+2
4
L [Y](p2 + 5p + 4) = −p
p+2
4
L [Y]((p + 2)2 ) = −p
p+2
4 −p
L [Y] = +
(p + 2) 3 (p + 2)2
4 h p+2 2 i
= − −
(p + 2)3 (p + 2)2 (p + 2)2
4 1 2
= − +
(p + 2) 3 (p + 2) (p + 2)2
h 4 1 2 i
Y = L −1 − +
(p + 2)3 (p + 2) (p + 2)2
h1i h1i
= 4e−2t L −1 3 − e−2t + 2e−2t L −1 2
p p
2
t
Y = 4e−2t − e−2t + 2e−2t t
2
π
d2 Y
Example 19.2.29. Solve dt2
+ 9Y = cos 2t, if Y(0) = 1; Y 2
= 1.
Y 00 + 9Y = cos 2t.
p
p2 L [Y 00 ] − pY(0) − Y 0 (0) + 9L [Y] = .
p2 +4
Given
Y(0) = 1, and Y 0 (0) = A.
p
∴ p2 L [Y] − p − A + 9L [Y] = .
p2 +4
p
L [Y](p2 + 9) = + p+A
p2 + 4
p p A
L [Y] = + 2 + 2
(p + 9)(p + 4) p + 9 p + 9
2 2
1h p p i p A
L [y] = − + 2 + 2
5 p +4 p +9
2 2 p +9 p +9
4 p 1 p A
= + +
5 p2 + 9 5 p2 + 4 p2 + 9
h4 p 1 p A i
Y = L −1 + +
5 p2 + 9 5 p2 + 4 p2 + 9
4 1 A
Y = cos 3t + cos 2t + sin 3t
5 5 3
π
Given Y 2
= 1.
4 3π 1 2π A 3π 18
∴1= cos + cos + sin ⇒A=− .
5 2 5 2 3 2 5
Hence
4 1 6
Y= cos 3t + cos 2t − sin 3t.
5 5 5
d2 Y
Example 19.2.30. Solve dt2
+ Y = t cos 2t, if Y = 0 = dY
dt
= 0 t = 0.
Y 00 + Y = t cos 2t.
1 8
L [Y](p2 + 1) = −
p2 + 4 (p2 + 4)2
1 8
L [Y] = − 2
(p + 4)(p + 1) (p + 4)2 (p2 + 1)
2 2
1 1 1 1 8h 1 1 3 i
L [y] = − − − −
3 p2 + 1 3 p2 + 4 9 p2 + 1 p2 + 4 (p2 + 4)2
5 1 5 1 8 1
= − 2 + +
9 p + 1 9 p2 + 4 3 (p2 + 4)2
h 5 1 5 1 8 1 i
Y = L −1 − + +
9 p2 + 1 9 p2 + 4 3 (p2 + 4)2
5 5 8h 1 i
= − sin t + sin 2t + (sin 2t − 2t cos 2t)
9 18 3 16
5 4 16
= − sin t + sin 2t − t cos 2t,
9 9 3
h i
since L −1 1
(p2 +a2 )2
= 1
2a3
(sin at − at cos at).
In the solution of simultaneous linear differential equations, the Laplace transform method can
also be used in the same way as we have done earlier.
Solution. Given
dx dy
− 2x + 3y = 0 and + 2x = 0.
dt dt
Taking Laplace transform on both sides, we get
and
p − 2 8
2 3 3p − 22 3p − 22
L [y] = = 2 =
p − 2 p − 3p − 4 (p + 1)(p − 4)
3
2 p − 1
5 3
∴ L [x] = +
p+1 p−4
h 5 3 i
x = L −1 +
p+1 p−4
x = 5e−t + 3e4t .
5 2
and L [y] = −
p+1 p−4
h 5 2 i
x = L −1 −
p+1 p−4
x = 5e−t − 2e4t .
Solution. Given
dx dy
+ y = sin t and + x = cos t.
dt dt
Taking Laplace transform on both sides, we get
1 p
pL [x] − 2 + L [y] = and pL [y] + L [x] =
p2 + 1 p2 + 1
1
pL [x] + L [y] = 2 +
p2 +1
p
L [x] + pL [y] =
p2 +1
h 2p i
x = L −1 = 2 cosh t.
p2 − 1
p 2 + 1
p2 +1
p p2 1
1 p +1
2
p2 +1
−2− p2 +1 1 2
L [y] = = = − 2
p2 −1 p2 +1 p −1
p 1
1 p
h 1 2 i
y = L −1 − = sin t − 2 sinh t.
p2 + 1 p2 − 1
Note. While solving simultaneous equations using Laplace transform, one can find one of the
unknown functions using Laplace transform and the other one directly using given equations.
Example 19.4.3. Solve by Laplace transform technique, (D+2)x−y = −6t; −2x+(D+1)y = −30t
if x(0) = 2 and y(0) = 3.
Solution. Given
x0 + 2x − y = −6t (19.4.1)
and
y0 − 2x + y = −30t (19.4.2)
6
pL [x] − x(0) + 2L [x] − L [y] = −
p2
Given x(0) = 2.
6 6
pL [x] − 2 + 2L [x] − L [y] = − 2
⇒ (p + 2)L [x] − L [y] = − 2 + 2
p p
2p2 − 6
(p + 2)L [x] − L [y] = (19.4.3)
p2
Now taking Laplace transform of (19.4.2), we have
1
pL [y] − y(0) − 2L [x] + L [y] = −30
p2
Given y(0) = 3.
30 30
pL [y] − 3 − 2L [x] + L [y] = − 2
⇒ −2L [x] + (p + 1)L [y] = − 2 + 3
p p
3p2 − 30
−2L [x] + (p + 1)L [y] = (19.4.4)
p2
2p3 + 5p2 − 6p − 36
L [x] =
p3 (p + 3)
1 2 12 1
= + 2− 3+
p p p p+3
h1 2 12 1 i
x = L −1 + 2 − 3 +
p p p p+3
2
t
= 1 + 2t − 12. + e−3t
2!
x = 1 + 2t − 6t2 + e−3t
dx
= 2 − 12t − 3e−3t .
dt
dx
Substituting the values of x and dt
in (19.4.1), we get
Example 19.4.4. Solve by Laplace transform method (D−2)x−(D−2)y = 1−2t; (D2 +1)x+2Dy =
0 if x(0) = 0, y(0) = 0 and x0 (0) = 0.
Solution. Given
x0 − y0 − 2x + 2y = 1 − 2t; x00 + 2y0 + x = 0.
and
p2 L [x] − px(0) − x0 (0) + 2[pL [y] − y(0)] + L [x] = 0.
1 2
pL [x] − pL [y] − 2L [x] + 2L [y] = − 2; p2 L [x] + 2pL [y] + L [x] = 0
p p
p−2
(p − 2)L [x] − (p − 2)L [y] =
p2
(p2 + 1)L [x] + 2pL [y] = 0
h1 1 1 i
L [x] = 2 −
−
p p + 1 (p + 1)2
h1 1 1 i
x = 2L −1 − −
p p + 1 (p + 1)2
x = 2(1 − e−t − te−t ).
p − 2 p−2
p2
p + 1
2
0 p2 + 1
L [y] = = − 2
p − 2 −(p − 2) p (p + 1)2
p2 + 1 2p
h p2 + 1 i
y = −L −1
p2 (p + 1)2
y = −t(1 + 2e−t ) + 2(1 − e−t )
Solution. Given
x00 − 3x − 4y = 0; x + y00 + y = 0.
p2 L [x] − px(0) − x0 (0) − 3L [x] − 4L [y] = 0; L [x] + p2 L [y00 ] − py(0) − y0 (0) + L [y] = 0.
2(p2 + 1)
L [x] =
(p2 − 1)2
1 1
= +
(p − 1) 2 (p + 1)2
h 1 1 i
x = L −1 +
(p − 1)2 (p + 1)2
h1i h1i
x = et L −1 2 + e−t L −1 2
p p
x = et t + e−t t
x = 2t cos ht
dx d2 x
= 2[t sinh t + cosh t] and = 2[t cosh t + sinh t + sinh t] = 2[t cosh t + 2 sinh t].
dt dt2
d2 x
Using the values of x, and dt2
in x00 − 3x − 4y = 0
y = sinh t − t cosh t.
Example 19.4.6. Solve (D − 2)x − (D + 1)y = 6e3t ; (2D − 3)x + (D − 3)y = 6e3t if x = 3, y = 0
when t = 0.
Solution. Given
x0 − 2x − y0 − y = 6e3t ; 2x0 − 3x + y0 − 3y = 6e3t .
6
pL [x] − x(0) − 2L [x] − pL [y] + y(0) − L [y] =
p−3
and
6
2pL [x] − 2x(0) − 3L [x] + pL [y] − y(0) − 3L [y] = .
p−3
Given initial conditions are x(0) = 3, y(0) = 0,
6 6
pL [x] − 3 − 2L [x] − pL [y] − L [y] = ; 2pL [x] − 6 − 3L [x] + pL [y] − 3L [y] = .
p−3 p−3
6 6 6p − 12
(p − 3)L [x] − (p + 1)L [y] = 3 + ; (2p − 3)L [x] + (p − 3)L [y] = 6 + = .
p−3 p−3 p−3
3p − 3
(p − 3)L [x] − (p + 1)L [y] = (19.4.5)
p−3
6p − 12
(2p − 3)L [x] + (p − 3)L [y] = (19.4.6)
p−3
From (19.4.5) and (19.4.6), we get
3p2 − 6p − 1 −3p + 5
L [x] = ; L [y] = .
(p − 3)(p − 1)2 (p − 3)(p − 1)2
1 2 2
L [x] = + 2
+
p − 1 (p − 1) p−3
h 1 2 2 i
x = L −1 + +
p − 1 (p − 1)2 p − 3
x = et + 2tet + 2e3t
Solution. Given
x00 + 2x − y0 = 1; x0 + y00 + 2y = 0.
1
p2 L [x]−px(0)−x0 (0)+2L [x]−pL [y]+y(0) = ; pL [x]−x(0)+[p2 L [y]−py(0)−y0 (0)]+2L [y] = 0.
p
1
p2 L [x] + 2L [x] − pL [y] = ; pL [x] + p2 L [y] + 2L [y] = 0.
p
1
(p2 + 2)L [x] − pL [y] = ; pL [x] + (p2 + 2)L [y] = 0.
p
From these equations, we have
p2 + 2 1
L [x] = ; L [y] =
p(p2 + 1)(p2 + 4) (p2 + 1)(p2 + 4)
1 p p
L [x] = − −
2p 3(p2 + 1) 6(p2 + 4)
h1 p p i
x = L −1 − −
2p 3(p2 + 1) 6(p2 + 4)
1 1 1
x = − cos t − cos 2t
2 3 6
−1 h 1 1 i
L [y] = −
3 p2 + 1 p2 + 4
1 h 1 1 i
y = − L −1 2 − 2
3 p +1 p +4
1 1
y = − sin t + sin 2t.
3 6
1 1 1 1 1
x= − cos t − cos 2t; y = − sin t + sin 2t.
2 3 6 3 6
19.5 Exercises
Answers h i
1. y = et − 3e−t + 2e−2t ; 2. y = 1
10
et (sin 2t − 2 cos 2t) + 2 ;
3. y = t + cos t − 3 sin t; 4. y = 50
1
53e−3t + 155e−3t + 4 sin t − 3 cos t ;
2
5. x = 2et − 3tet + t2 et ; 6. y = 74 e−t + 2t e−t − 43 e−3t ;
7. y = cos 3t + 1
24
(3 sin t − sin 3t; 8. y = 15 (cos 2t + 4 sin 3t + 4 cos 3t);
9. y = 2t + π sin 3t; 10. y = 2t + 3 + 21 (e3t−e ) − 2e2t ;
t
3 2
t5
11. y = e−t t2 + 6t + 4 ; 12. y = et 1 − t − t2 + 60 ;
√
e−t t
13. y = 1 + 3
− 23 e 2 cos 2
3
t; 14. y = −1 + 12 (cos t + cosh t);
15. y = t(sin t + cos t); 16. y = t;
17. y = 1 = 2t.
Lesson 20
APPLICATION OF LAPLACE
TRANSFORMS TO PARTIAL
DIFFERENTIAL EQUATIONS
Learning Objectives
Upon completion of this lesson, students will be able to
20.1 Introduction
T
he Laplace transform is especially well - suited for solving initial boundary value problem
for which some auxiliary conditions are prescribed at t = 0. Such problems arise naturally
in the solution of the heat equation or wave equation. We will now prove some theorems.
∂Y
where L [Y(x, t)] = y(x, p), Yt (x, 0) = ∂t t=0
.
Proof. (i)
∂Y Z
∂Y ∞
L = e−pt dt
∂t 0 ∂t
∂Y
Z a
= lim e−pt dt
a→∞ 0 ∂t
Z a i
= lim [e −pt
Y(x, t)at=0 +p e−pt Y(x, t)dt
a→∞ 0
Z ∞
= −Y(x, 0) + p e−pt Ydt
0
∂Y
(ii) Let V = ∂t
= Y, then
∂2 Y ∂V
L = L
∂t2 ∂t
= pV(x, p) − V(x, 0), by (i)
= pL [V] − Yt (x, 0)
∂Y
= pL − Yt (x, 0)
∂t
= p[py(x, p) − Y(x, 0)] − Yt (x, 0), by (i)
∂2 Y
L = p2 y(x, p) − pY(x, 0) − Yt (x, 0).
∂t2
(iii)
∂Y
−pt ∂Y
Z ∞ Z ∞
d d dy
L = e dt = e−pt Ydt = L [Y] = .
∂x 0 ∂t dx 0 dx dx
(iv)
∂2 Y ∂U ∂Y
L = L , where U =
∂x2 ∂x ∂x
d
=
L [U]
dx
d ∂Y
= L
dx ∂x
d dy
= by (iii)
dx dx
∂2 Y d2 y
L = .
∂x2 dx2
∂Y
Example 20.2.1. Solve ∂x
= 2 ∂Y
∂t
+ Y, Y(x, 0) = 6e−3x which is bounded for x > 0, t > 0.
∂Y ∂Y
L = L 2 +Y
∂x ∂t
dy
= 2[py(x, p) − Y(x, 0)] + y
dx
= 2py(x, p) − 12e−3x + y
dy
− (1 + 2p)y = −12e−3x
dx
Z Z
IntegratingFactor = exp Pdx = exp − (1 + 2p)dx = exp(−(1 + 2p)x).
The solution is
Z
ye −(1+2p)x
= −12e−3x .e−(1+2p)x dx + c
Z
= −12 e−(2p+4)x dx + c
e−(2p+4)x
= −12 +c
−(2p + 4)
e−(2p+4)x
= 6 +c
p+2
6e−3x
y = + ce(1+2p)x
p+2
6e−3x
∴y=
p+2
6e−3x h 1 i
i.e., L [Y] = ⇒ Y = 6e−3x L −1
p+2 p+2
Y = 6e−3x e−2t = 6e−(2t+3x) .
∂2 U ∂2 U ∂U
Example 20.2.2. Solve ∂x2
− ∂t2
= xt, where U = 0 = ∂t
at t = 0.
Solution.
∂2 U ∂2 U
− 2 = xt
∂x2 ∂t
h ∂2 U i h ∂2 U i
L −L = xL [t]
∂x2 ∂t2
d2 u x
2
− [p2 u − pU(x, 0) − Ut (x, 0)] = 2
dx p
C.F = Ae−xp + Be xp
u0p = B u00p = 0.
d2 u x
2
− p2 u = 2
dx p
x
0 − p2 (A + Bx) = 2
p
1
A = 0 and B=− .
p4
x
∴ P.I = u p = −
p4
x
∴ u = Ae−px + Be px − (20.2.2)
p4
U = 0 for t = 0 ⇒ U = 0, ∀x, p
⇒u=0
x
∴ u = Ae−xp −
p4
x −1 1
h i xt3
∴u=− ⇒ U = −xL = −
p4 p4 3!
xt3
U=− .
6
∂Y
= 2 ∂∂xY2 , where Y(0, t) = 0 = Y(5, t) and Y(x, 0) = 10 sin 4πx.
2
Example 20.2.3. Solve ∂t
Solution.
∂Y ∂2 Y
=2 2
∂t ∂x
h ∂Y i ∂2 Y i
L = 2L [
∂t ∂x2
d2 y
py − Y(x, 0) = 2
dx2
d2 y
2 = py − 10 sin 4πx
dx2
d2 y p
− y = −5 sin 4πx (20.2.3)
dx2 2
The general solution is
y = C.F + P.I
√p √p
C.F = Ae x 2 + Be−x 2
p
−16π2 A sin 4πx − 16π2 cos 4πx − (A sin 4πx + B cos 4πx) = −5 sin 4πx
2
10
A= and B = 0.
32π2 + p
10
∴ yp = sin 4πx
32π2 + p
∴ the general solution is
√p √p 10
y = Ae x 2 + Be−x 2 + sin 4πx.
32π2 + p
∴ A+B = 0
√p √p 10 sin 20π
Ae5 2 + Be−5 2 + = 0
32π2 + p
10 h 10 i
∴y= sin 4πx ⇒ Y = L −1
sin 4πx
32π2 + p 32π2 + p
2
Y = 10e−32π t sin 4πx.
∂Y ∂2 Y
Example 20.2.4. Solve ∂t
= ∂x2
, x > 0, t > 0 where Y(0, t) = 1, Y(x, 0) = 0.
Solution.
∂Y ∂2 Y
= 2
∂t ∂x
h ∂Y i ∂2 Y i
L = 2L [
∂t ∂x2
d2 y
py − Y(x, 0) = 2
dx2
d2 y
= py, since Y(x, 0) = 0.
dx2
d2 y
− py = 0 (20.2.4)
dx2
The general solution is
√ √
y = Ae x p
+ Be−x p
(20.2.5)
− √xp
∴ y = Be .
∂Y ∂Y
Example 20.2.5. Solve ∂x
− ∂t
= 1 − e−t , 0 < x < 1, t > 0 given that Y(x, 0) = x.
Solution.
∂Y ∂Y
− = 1 − e−t
∂x ∂t
h ∂Y
∂Y i
L = L [1 − e−t ].
−
∂x ∂t
dy 1 1
− [py − Y(x, 0)] = −
dx p p+1
dy 1 1
− py + x = − , since Y(x, 0) = x
dx p p+1
dy 1 1
− py = − −x
dx p p+1
Z Z Z
y exp Pdx = Q exp pdx dx + c.
Z Z
I.F = exp Pdx = exp −pdx = exp − px .
Z h
1 i
ye −px
= − x e−px dx + c
p(p + 1)
hx 1 1i
= − 2 + 2 e−px + c
p p (p + 1) p
x 1
y = + + ce px .
p p(p + 1)
x 1
∴y= + .
p p(p + 1)
x 1 hx 1 1 i
L [Y] = + ⇒ Y = L −1 + − = x + 1 − e−t .
p p(p + 1) p p p+1
∂Y
= 2 ∂∂xY2 , where Y(0, t) = 0 = Y(5, t) and Y(x, 0) = 10 sin 4πx −
2
Example 20.2.6. Solve ∂t
5 sin 6πx.
Solution.
∂Y ∂2 Y
=2 2
∂t ∂x
h ∂Y i ∂2 Y i
L = 2L [
∂t ∂x2
d2 y
py − Y(x, 0) = 2
dx2
d2 y
2 = py − 10 sin 4πx + 5 sin 6πx
dx2
d2 y p 5
2
− y = −5 sin 4πx + sin 6πx (20.2.6)
dx 2 2
The general solution is
y = C.F + P.I
√p √p
C.F = Ae x 2 + Be−x 2
p
−16π2 A sin 4πx − 16π2 cos 4πx − [A sin 4πx + B cos 4πx] = −5 sin 4πx
2
10 sin 4πx
A= .
32π2 + p
p 5
−36π2C sin 6πx − 36π2 D cos 6πx − (C sin 6πx + D cos 6πx) = sin 6πx
2 2
−5
D= and C = 0.
72π2 + p
10 5
∴ y p = y p1 + y p2 = sin 4πx − sin 6πx
32π + p
2 72π + p
2
√p √p 10 5
y = Ae x 2 + Be−x 2 + sin 4πx − sin 6πx.
32π + p
2 72π + p
2
∴ A+B = 0
√p √p 10 sin 20π 5
Ae5 2 + Be−5 2 + − sin 30π = 0
32π + p
2 72π + p
2
10 5 h 10 5 i
∴y= sin 4πx − sin 6πx ⇒ Y = L −1
sin 4πx − sin 6πx
32π2 + p 72π2 + p 32π2 + p 72π2 + p
2 2
Y = 10e−32π t sin 4πx − 5e−72π t sin 6πx.
∂y ∂2 y
∂y
π
Example 20.2.7. Solve ∂t
= ∂x 2 , where Y 2 , t = 0, ∂x
= 0 and y(x, 0) = cos 3x.
x=0
Solution.
∂y ∂2 y
=
∂t ∂x2
h ∂y i h ∂2 y i
L = L
∂t ∂x2
d2 y
py(x, p) − y(x, 0) =
dx2
d2 y
= py − cos 3x
dx2
d2 y
− py = − cos 3x (20.2.7)
dx2
The general solution is
y = C.F + P.I
√ √
C.F = Ae x p
+ Be−x p
y p = A sin 3x + B cos 3x
1
A = 0. B=
9+ p
1
∴ yp = cos 3x
9+ p
∴ the general solution is
√ √ 1
y = Ae x p
+ Be−x p
+ cos 3x. (20.2.8)
9+ p
Given y π2 , t) = 0 ⇒ y π2 , p) = 0
From (20.2.8),
√ √
p π2 p π2
Ae + Be− = 0
Also,
∂y dy
=0⇒ = 0,
∂x dx
when x = 0.
dy √ √ √ √ 3
= A pe x p − pBe−x p + sin 3x
dx 9+ p
√ √ dy
0 = A p − B p, since = 0, when x = 0
dx
1 h 1 i
∴y= cos 3x ⇒ Y = L −1 cos 3x
9+ p 9+ p
∂y ∂2 y
Example 20.2.8. Solve ∂t
= ∂x2
, y(x, 0) = 3 sin 2πx, y(0, t) = 0 = y(1, t).
Solution.
∂y ∂2 y
=
∂t ∂x2
h ∂y i h ∂2 y i
L = L
∂t ∂x2
2
d y
py(x, p) − y(x, 0) =
dx2
d2 y
= py − 3 sin 2πx
dx2
d2 y
− py = −3 sin 2πx (20.2.9)
dx2
The general solution is
y = C.F + P.I
√ √
C.F = Ae x p
+ Be−x p
−4π2 A sin 2πx − 4π2 B cos 2πx − p(A sin 2πx + B cos 2πx) = −3 sin 2πx
3
A= , B=0
4π2 +p
3
∴ yp = sin 2πx
4π2 +p
∴ the general solution is
√ √ 3
y = Ae x p
+ Be−x p
+ sin 2πx. (20.2.10)
4π2 +p
y(1, t) = 0 ⇒ y(1, p) = 0.
From (20.2.9),
∴ A+B = 0
√ √
Ae p
+ Be− p
= 0
3 h 3 i
∴y= sin 2πx ⇒ Y = L −1
sin 2πx
4π2 + p 4π2 + p
2
Y = 3e−4π t sin 2πx.
Partial differential equations are frequently occurring in solving many problems in Physics and
Engineering. If the partial differential equation is satisfied within a certain region and certain
condition are satisfied on the boundary region, the problem is said to be a boundary value problem.
Laplace transform is used to solve such type of problems.
Heat Conduction Equation. The heat flow in a homogeneous body is given by the heat equation
∂u ∂2 u ∂2 u ∂2 u
= c2 + + ,
∂t ∂x2 ∂y2 ∂z2
c2 = k
cρ
, where u(x, y, z, t) is the temperature of the body, k is the thermal conductivity, σ is the
specific heat, ρ the density of the material of the body and c2 the constant is called the diffusivity
of the body.
If we consider the heat flow in along thin bar on wire of constant cross section and homogeneous
material which along x - axis (is insulated so that the heat flows in x - direction only u depends
only) on x and t. The heat equation becomes
∂u ∂2 u
= c2 2 .
∂t ∂x
Wave Equation. The vibration of an elastic string are governed by the one dimensional wave
equation
∂2 u 2∂ u
2
= c ,
∂t2 ∂x2
where c2 = Tρ . Here u(x, t) is the deflection of the string stretched between the fixed points (0, 0)
and (l, 0).
Laplace Equation. When the temperatures in a homogeneous material is in steady state then the
∂2 u ∂2 u ∂2 u
+ + = 0.
∂x2 ∂y2 ∂z2
∂u
Example 20.4.1. Solve ∂t
+ x ∂u
∂x
= x, x > 0, t > 0, u(x, 0) = 0 = u(0, t).
Solution. Given
∂u ∂u
+ x = x.
∂t ∂x
Taking Laplace transform on both sides, we get
h ∂u i h ∂u i
L + xL= xL [1]
∂t ∂x
du x
pu(x, p) − u(x, 0) + x =
dx p
du x
pu(x, p) + x = , since u(x, 0) = 0
dx p
du p 1
+ u(x, p) =
dx x p
Z p
I.F = exp dx = exp p log x = exp(log x p ) = x p .
x
The general solution is
Z
1 p
ux p
= x dx + c
p
1 x p+1
= +c
pp+1
Here
u(0, t) = 0 ⇒ L [u(0, t)] = u(0, p) = 0.
x 1 1
u= =x −
p(p + 1) p p+1
1 1
∴ u = xL −1 − = x(1 − e−t ).
p p+1
Example 20.4.2. Solve x ∂u
∂t
+ ∂u
∂x
= x, u(x, 0) = 0 = u(0, t).
Solution. Given
∂u ∂u
x + = x.
∂t ∂x
Taking Laplace transform on both sides, we get
h ∂u i h ∂u i
xL +L = xL [1]
∂t ∂x
du x
x[pu(x, p) − u(x, 0)] + =
dx p
du x
xpu(x, p) + = , since u(x, 0) = 0
dx p
du x
+ xpu(x, p) =
dx p
Z px2
I.F = exp pxdx = exp .
2
px2
i.e., I.F = e 2 .
Here
u(0, t) = 0 ⇒ L [u(0, t)] = u(0, p) = 0.
x2 x2
∴u=t− t− H t− .
2 2
∂2 u
= a2 ∂∂xu2 , (t > 0, x > 0) where
2
Example 20.4.3. Solve the boundary value problem ∂t2
Solution. Given
∂2 u 2∂ u
2
= a .
∂t2 ∂x2
Taking Laplace transform on both sides, we get
h ∂2 u i h ∂2 u i
L = a2 L
∂t2 ∂x2
2
d u
p2 u(x, p) − pu(x, 0) − ut (x, 0) = a2
dx2
d2 u
p2 u(x, p) = a2 2 , since u(x, 0) = 0 and ut (x, 0) = 0
dx
d 2 u p2
− u=0
dx2 a2
px px
u(x, p) = Ae a + Be− a
Here
1
u(0, t) = t ⇒ L [u(0, t)] = L [t] ⇒ u(0, p) = .
p2
lim u(x, t) = 0 ⇒ lim L [u(x, t)] = 0 ⇒ lim u(x, p) = 0
x→∞ x→ x→∞
As x → ∞, u(x, p) → 0, we have A = 0.
−px
u(x, p) = Be p
1
u(0, p) = B ⇒ = B.
p2
1 −px
∴ u(x, p) = e a .
p2
Taking inverse Laplace transform,
h 1 px i
∴ u(x, t) = L −1 e− a ,
p2
t − a , t>
x x
x x
a
u(x, t) = = t− H t−
a a
0, t < x
a
∂2 u
= a2 ∂∂xu2 , (t > 0, x > 0) where
2
Example 20.4.4. Solve the boundary value problem ∂t2
Solution. Given
∂2 u 2∂ u
2
= a .
∂t2 ∂x2
Taking Laplace transform on both sides, we get
h ∂2 u i h ∂2 u i
L = a2 L
∂t2 ∂x2
2
d u
p2 u(x, p) − pu(x, 0) − ut (x, 0) = a2
dx2
d2 u
p2 u(x, p) = a2 2 , since u(x, 0) = 0 and ut (x, 0) = 0
dx
d 2 u p2
− u=0
dx2 a2
px px
u(x, p) = Ae a + Be− a
Here Z ∞
u(0, t) = F(t) ⇒ L [u(0, t)] = F(t)e−pt dt = F(p).
0
As x → ∞, u(x, p) → 0, we have A = 0.
−px
u(x, p) = Be p
u(0, p) = B ⇒ F(p) = B.
−px
∴ u(x, p) = F(p)e a .
h px i x x
∴ u(x, t) = L −1 F(p)e− a = F t − H t − .
a a
F t − a , t >
x x
a
u(x, t) =
t< x
0,
a
∂2 u
= a2 ∂∂xu2 − g, (t > 0, x > 0) where
2
Example 20.4.5. Solve the boundary value problem ∂t2
∂2 u 2∂ u
2
= a − g.
∂t2 ∂x2
h ∂2 u i h ∂2 u i
L = a2 L − gL [1]
∂t2 ∂x2
d2 u g
p2 u(x, p) − pu(x, 0) − ut (x, 0) = a2 2 −
dx p
2
d u
p2 u(x, p) = a2 2 , since u(x, 0) = 0 and ut (x, 0) = 0
dx
d 2 u p2 g
− u= 2 .
dx2 a2 a p
px px g
u(x, p) = Ae a + Be− a −
p3
Here
u(0, t) = 0 ⇒ L [u(0, t)] = 0 ⇒ u(0, p) = 0.
du
lim = 0.
p→∞ dx
du
∴ lim = 0 ⇒ A = 0.
p→∞ dx
−px g
u(x, p) = Be p − 3
p
g g
u(0, p) = 0 ⇒ B − 3
= 0 ⇒ B = 3.
p p
g −px g
∴ u(x, p) = e a − .
p3 p2
Taking inverse Laplace transform,
h g −px gi
∴ u(x, t) = L −1 e a −
p3 p2
− 2ag2 (2axt − x2 ),
x ≤ at
u(x, t) =
2
− gt 0,
x ≥ at
2
∂u
= k ∂∂xu2 which tends to zero as x → ∞ and
2
Example 20.4.6. Find the solution of the equation ∂t
which satisfies the conditions u = f (t) when x = 0, t > 0, and u = 0, where x > 0, t = 0.
∂u ∂2 u
= k 2.
∂t ∂x
h ∂u i h ∂2 u i
L = kL
∂t ∂x2
2
d u
pu(x, p) − u(x, 0) = k
dx2
d2 u
pu(x, p) = k 2 , since u(x, 0) = 0
dx
2
d u p
− u = 0.
dx2 k
√p √p
u(x, p) = Ae k x
+ Be− k x
As x → ∞ we have u → 0. As x → ∞, we have u → 0. ∴ A = 0.
Since u = f (t), we have f (p) = B
√p
∴ u = f (p)e− k x
h √p i 1
Z r+∞ √p
∴ u(x, t) = L −1
f (p)e − kx = e pt f (p)e− k x
d p.
2πi r−i∞
Example 20.4.7. A string is stretched between two fixed points (0, 0) and (c, 0). If it is displaced
into the curve y = b sin πxc and released from rest in that position at time t = 0, find its
∂2 y 2∂ y
2
= a .
∂t2 ∂x2
πx
with boundary conditions y(0, t) = 0, y(c, t) = 0 and initial condition y(x, 0) = b sin c
and
yt (x, 0) = 0.
h ∂2 y i h ∂2 y i
L = a2 L
∂t2 ∂x2
2
d y
p2 y(x, p) − py(x, 0) − yt (x, 0) = a2
dx2
πx d2 y
p2 y(x, p) − pb sin = a2 2 , since yt (x, 0) = 0
2 dx
2
d y p 2
bp πx
2
− 2 y = − 2 sin
dx a a c
px px b πx
u(x, p) = Ae a + Be− a + π2 a2
p sin
c2
+ p2 c
Here
y(0, t) = 0 ⇒ L [y(0, t)] = 0 ⇒ y(0, p) = 0.
Also
y(0, p) = 0 ⇒ A + B = 0.
pc −pc
y(c, p) = 0 ⇒ Ae a + Be a
h p i πx πx πat
∴ y(x, t) = bL −1 π2 a2
sin = b sin cos .
c2
+ p2 c c c
Example 20.4.8. An infinite long string having one end at x = 0 is initially at rest on the x−
axis. The end x = 0 undergoes a periodic transverse displacement given by A0 sin(nt), t > 0.
Find the displacement of any point on the string at t > 0.
Solution. The displacement at any point of the string is given by the partial differential equation
is
∂2 u 2∂ u
2
= a
∂t2 ∂x2
with boundary and initial condition u(0, t) = A0 sin(nt), t > 0 u(x, 0) = 0, and ut (x, 0) = 0,
x ≥ 0. The displacement is finite.
h ∂2 u i h ∂2 u i
L = a2 L
∂t2 ∂x2
2
d u
p2 u(x, p) − pu(x, 0) − ut (x, 0) = a2
dx2
d2 u
p2 u(x, p) = a2 2 , since u(x, 0) = 0 and ut (x, 0) = 0
dx
d 2 u p2
− u = 0.
dx2 a2
px px
u(x, p) = Ae a + Be− a
Here
n
u(0, t) = A0 sin(nt) ⇒ L [u(0, t)] = A0 L [sin nt] ⇒ u(0, p) = A0 .
p2 + n2
u(x, p) is finite for x ≥ 0.
∴ A = 0.
n n
u(0, p) = A0 ⇒ B = A0 2
p2 +n 2 p + n2
n −px
∴ u(x, p) = A0 e a .
p2 +n 2
h 1 −px i
∴ u(x, t) = nA0 L −1 e a
p2 + n2
, t>
x x
A sin n t −
0 a a
u(x, t) =
t< x
0,
a
20.5 Exercises
∂u ∂2 u
1. Solve the boundary value problem ∂t
= ∂x2
, 0 < x < 1, t > 0 where u(0, t) = 1 = u(1, t),
t > 0 and u(x, 0) = 1 + sin πx, 0 < x < 1.
∂2 u ∂2 u
2. Solve the boundary value problem ∂t2
= ∂x2
, 0 < x < 1, t > 0 where u(0, t) = 0 = u(1, t),
t > 0, u(x, 0) = sin πx and ut (x, 0) = − sin πx, 0 < x < 1.
Answers
1. u(x, t) = 1 + e−π t sin πx.
2
5. u(x, t) = 1
π2
(1 − cos πt) sin πx.
Lesson 21
APPLICATIONS OF FOURIER
TRANSFORMS TO INITIAL AND
BOUNDARY VALUE PROBLEMS
Learning Objectives
Upon completion of this lesson, students will be able to
• solve heat equation by using finite Fourier transform and also give physical
interpretation of the problem
21.1 Introduction
W
e will use Fourier transform to solve initial and boundary value problems. If the Fourier
transform is applied with respect to one of the variables in the partial differential equation,
then we obtain an ordinary differential equation in terms of the other variable. We solve this
differential equation. The solution to the boundary value problem (BVP)is then obtained by taking
inverse Fourier transform.
∞
∂2 V ∂V ∂V
Z i∞ Z ∞
sin px 2 dx = sin px − p cos px. dx
0 ∂x ∂x 0
0 ∂x
∂V ∂V
Z ∞
= −p cos px. dx, i f → 0 as x → ∞
0 ∂x ∂x
∞ Z ∞
= −p V cos px + p V sin pxdx
0
0
= p(V) x=0 − p2 V s .
∞
∂2 V ∂V ∂V
Z i∞ Z ∞
cos px 2 dx = cos px + p sin px. dx
0 ∂x ∂x 0
0 ∂x
∂V Z ∞
∂V
= − + p[V sin px] − p 2
V cos px.dx, i f → 0 as x → ∞
∂x x=0 0 ∂x
∂V
= − p2 V c .
∂X x=0
if V → 0 as x → ∞.
Thus, we observe that for the exclusion of ∂∂xV2 from a differential equation, we require (V) x=0 in
2
Example 21.2.1. Use the complex form of the Fourier transform to show that V =
R∞
f (u)e− 4t du is the solution of the boundary value problem ∂V = ∂∂xV2 , −∞ < x < ∞,
(x−u) 2
1√
2π t −∞ ∂x
∞
∂V ipx ∂ V ipx
Z Z ∞ 2
1 1
√ e dx = √ e dx
2π −∞ ∂t 2π −∞ ∂x2
d
V(p) = (ip)2 V(p)
dt
d
V(p) = −p2 V(p)
dt
Z ∞
1
V(p) = √ f (u)eipu du,
2π −∞
Z ∞ h −p2 t Z ∞
1 e i
V(x, t) = √ √ f (u)eipu du e−ipx d p
2π
Z ∞−∞ h 2π −∞
1 2
i
= f (u) e−p t−ip(x−u) d p du
2π −∞
Z ∞ h i2
1 h Z ∞ −t p+i (x−u)2 − (x−u)2 i
V(x, t) = f (u) e 2t 4t
d p du
2π −∞ −∞
Z ∞ ∞
h 2
i2
1 hZ −t p+i (x−u) i (x−u)2
V(x, t) = f (u) e 2t
d p e− 4t du
2π −∞ −∞
√ i(x − u) dy
Put t p+ =y so that d p = √ .
2t t
Z ∞ h 1 Z ∞ 2 i (x−u)2
1
V(x, t) = f (u) √ e−y dy e− 4t du
2π −∞ t −∞
Z ∞ Z ∞
1 1 √ − (x−u) 2
2 √
= f (u) √ πe 4t du, since e−y dy = π
2π −∞ t −∞
Z ∞
1 (x−u)2
∴V = √ f (u)e− 4t du.
2 πt −∞
∂U ∂2 U
Example 21.2.2. Solve ∂t
= ∂x2
, x > 0, t > 0 subject to the conditions
(i) U = 0, when x = 0, t > 0
(ii) U = 1, 0 < x < 1, when t = 0
(iii) U = 0, x ≥ 1.
(iv) U(x, t) is bounded.
Solution. Given that U = 0, when x = 0. Thus we will consider Fourier sine transform. Given
where A is a constant.
When t = 0, we have the Fourier sine transform of U(x, t) is
r Z ∞
2
U s (p, 0) = U(x, 0) sin pxdx
π 0
r Z 1 r Z ∞
2 2
= 1. sin pxdx + 0. sin pxdx
π 0 π 1
r
2 cos px 1
= −
π p 0
r
2 1 − cos p
=
π p
Since r
2 1 − cos p
U s (p, 0) = A ⇒ A =
π p
r
2 1 − cos p −p2 t
∴ Us = e .
π p
Applying inverse Fourier sine transform, we have
Z ∞
2 1 − cos p −p2 t
U(x, t) = e sin pxd p
π 0 p
∂U
= 2 ∂∂xU2 , x > 0, t > 0 if U(0, t) = 0, U(x, 0) = e−x , x > 0, U(x, t)
2
Example 21.2.3. Solve ∂t
Solution. Given that U = 0, when x = 0. Thus we will consider Fourier sine transform. Given
partial differential equation
∂U ∂2 U
=2 2
∂t ∂x
Taking Fourier sine transform on both sides, we get
r Z ∞ r Z ∞
2 ∂U 2 ∂2 U
sin pxdx = 2 sin pxdx
π 0 ∂t π 0 ∂x2
r r r Z ∞
2 h ∂U ∂U
Z ∞
2d i∞ 2
U sin pxdx = 2 sin px − 2p cos pxdx
π dt 0 π ∂x 0 π 0 ∂x
r Z ∞
∂U s 2 ∂U ∂U
= −2p cos pxdx, i f → ∞ as x → ∞
∂t π 0 ∂x ∂x
r r Z ∞
∂U s 2h i∞ 2
= −2p U(x, t) cos px − 2p2 U sin pxdx
∂t π 0 π 0
r
2
= − pU(0, t) − p2 U s , assuming U → 0 as x → 0
π
where A is a constant.
But U(x, 0) = e−x
r Z ∞
2
U s (p, 0) = U(x, 0) sin pxdx
π 0
r Z ∞
2
= e−x . sin pxdx
π 0
r
2 h e−x iin f ty
= (− sin px − p cos px)
π 1 + p2 0
r
2 p
=
π 1 + p2
Since r
2 p
U s (p, 0) = A ⇒ A =
π 1 + p2
r
2 p 2
∴ Us = e−2p t .
π 1 + p62
Applying inverse Fourier sine transform, we have
Z ∞
2 p 2
U(x, t) = e−2p t sin pxd p
π 0 1+ p2
Example 21.2.4. The temperature U in the semi infinite rod 0 ≤ x ≤ ∞ is determined by the
∂U
= k ∂∂xU2 subject to the conditions (i) U(x, 0) = 0 when t = 0, x ≥ 0; (ii)
2
differential equation ∂t
∂U
∂x
= −u (a constant) when x = 0, t > 0. Making use of cosine transform show that
Z ∞
2u cos px −kp2 t
U(x, t) = (1 − e )d p
π 0 p2
∂U ∂2 U
=k 2
∂t ∂x
Z
I.F = exp kp2 dt = exp kp2 t .
2
i.e., I.F = ekp t
Z r
kp2 t k kp2 t
U ce = e dt + A
u
r
2 u kp2 t
= e +A
π p2
r
2 u −kp2
i.e., U c = 1 − e
π p2
Applying inverse Fourier cosine transform, we have
r Z ∞ 2
2u ∞ 1 − e−kp t
Z
2
Uc = U c cos pxd p = cos pxd p
π 0 π 0 p2
If in the given problems,range of one of the variables is finite, we will apply finite Fourier
transform.
Finite Fourier Transforms of the Partial Derivatives
∂U
(i) The finite Fourier sine and cosine transforms of ∂x
where U is a function of x and t for
∂U ∂U
Z l
pπx
Fs = sin dx
∂x 0 ∂x l
pπx il pπ l
Z
h pπx
= U(x, t) sin − U cos dx
l 0 l 0 l
∂U pπ
Fs = − Fc (U)
∂x l
Again
∂U ∂U
Z l
pπx
Fc = cos dx
∂x 0 ∂x l
pπx il pπ l
Z
h pπx
= U(x, t) cos + U sin dx
l 0 l 0 l
∂U pπ
Fc = F s (U)
∂x l
= U(0, t) − U(l, t) cos pπ
∂2 U
(ii) The finite Fourier sine and cosine transforms of ∂x2
where U is a function of x and t for
0 < x < l, t > 0
∂2 U Z l
∂2 U pπx
Fs = sin dx
∂x2 0 ∂x
2 l
h ∂U pπx il pπ l ∂U
Z
pπx
= sin − cos dx
∂x l 0 l 0 ∂x l
pπ l ∂U
Z
pπx
= − cos dx
l 0 ∂x l
pπx il pπ l
Z
pπ h pπx
= − U cos + U sin dx
l l 0 l 0 l
pπ h pπ i
= − F s (U) − U(0, t) − U(l, t) cos px
l l
∂2 U p2 π2 pπ
∴ Fs = − 2 F s (U) + [U(0, t) − U(l, t) cos pπ]
∂x2 l l
and
∂2 U ∂2 U
Z l
pπx
Fc = cos dx
∂x2 0 ∂x
2 l
h ∂U pπx il pπ l ∂U
Z
pπx
= cos + sin dx
∂x l 0 l 0 ∂x l
pπ ∂U
= Fs − [U x (0, t) − U x (l, t) cos pπ]
l ∂x
∂2 U p2 π2
∴ Fc = − Fc (U) − [U x (0, t) − U x (l, t) cos pπ].
∂x2 l2
Choice of finite sine or cosine transform The choice of finite or cosine transform is decided by
form of the boundary conditions.
If U(0, t) and U(l, t) are given we apply finite Fourier sine transform.
If U x (0, t) and U x (l, t) are given we apply finite Fourier cosine transform.
∂u ∂2 u
Example 21.4.1. Use finite Fourier transforms to solve the equation ∂t
= ∂x2
, u(0, t) = 0 =
u(4, t), u(x, 0) = 2x where 0 < x < 4, t > 0. Give the physical interpretation of the problem.
4
∂u ∂u
Z Z 4 2
pπx pπx
sin dx = sin dx
0 ∂t 4 0 ∂x
2 4
d p2 π2 pπ
(u s ) = − us + [u(0, t) − u(4, t) cos pπ],
dt 4 4
d p2 π2
us = − us
dt 16
The solution is
p2 π2 t
u s = Ae− 16
Since u(x, 0) = 2x, 0 < x < 4 Taking finite Fourier sine transform, we have at t = 0,
Z 4
pπx
us = 2x sin
dx
0 4
h cos pπx sin pπx i4
= 2x − pπ 4
− 2 − p2 π24
0
4 16
32
= − cos pπ
pπ
32
us = (−1) p+1 .
pπ
32
∴ u(x, 0) = u s = A ⇒ A = (−1) p+1 .
pπ
32 −p2 π2 t
∴ us = (−1) p+1 e 16
pπ
Taking inverse finite Fourier sine transform, we have
∞
2 X 32 −p2 π2 t pπx
u(x, t) = (−1) p+1 e 16 sin
4 p=1 pπ 4
∞
16 X 1 −p2 π2 t pπx
u(x, t) = (−1) p+1 e 16 sin .
π p=1 p 4
Physical Interpretation Here u(x, t) represents the temperature at any point x at any time to in
the solid bounded by the planes x = 0, and x = 4 (or) a bar on the x− axis with the ends x = 0
and x = 4 whose surfaces insulated latterly. The conditions u(0, t) = 0 and u(4, t) = 0 imply
that the ends are kept at zero temperature. The condition u(x, 0) = 2x implies that the initial
temperature is a function of x.
∂U ∂2 U
Example 21.4.2. Solve the equation ∂t
= ∂x2
, 0 < x < 6, t > 0 subject to the conditions
U(0, t) = 0, U(6, t) = 0,
1, 0 < x < 3
U(x, 0) =
0, 3 < x < 6
6
∂U ∂U
Z Z 6 2
pπx pπx
sin dx = sin dx
0 ∂t 6 0 ∂x
2 6
d p2 π2 pπ
(U s ) = − Us + [U(0, t) − U(6, t) cos pπ],
dt 36 6
d p2 π2
Us = − Us
dt 36
The solution is
p2 π2 t
U s = Ae− 36
6 pπ
∴ U(x, 0) = U s = A ⇒ A = 1 − cos .
pπ 2
6 pπ −p362 π2 t
∴ Us = 1 − cos e
pπ 2
Taking inverse finite Fourier sine transform, we have
∞
2X 6 pπ −p362 π2 t pπx
U(x, t) = 1 − cos e sin
6 p=1 pπ 2 6
∞
2 X 1 pπ −p362 π2 t pπx
U(x, t) = 1 − cos e sin
π p=1 p 2 6
Physical Interpretation
Here U(x, t) represents the temperature at any point x at any time to in the solid bounded by
the planes x = 0, and x = 6 kept at zero temperature which is insulated latterly. Initially the
temperature in the half bar from x = 0 to x = 3 is constant equal to one unit. The half bar from
x = 3 to x = 6 is at zero temperature.
∂U ∂2 U
Example 21.4.3. Solve the boundary value problem ∂t
= ∂x2
, U(0, t) = 1, U(π, t) = 3,
U(x, 0) = 1, 0 < x < π, t > 0.
π Z π 2
∂U ∂U
Z
sin pxdx = sin pxdx
0 ∂t 0 ∂x
2
d
(U s ) = −p2 U s + p[U(0, t) − U(π, t) cos pπ],
dt
d
U s = −p2 U s + p(1 − cos pπ)
dt
Z
I.F = exp p2 dt = exp(p2 t)
2
i.e., I.F = e p t .
Z
p2 t 2
∴ U se = p(1 − cos πp)e p t dt + c
1 − cos 3π p2 t
= e +c
p
1 − cos 3π 2
Us = + ce−p t
p
At t = 0, c = 2 cos pπ
p
1 − cos pπ 2 cos pπ −p2 t
∴ Us = + e
p p
∞ ∞
2 X 1 − 3 cos pπ 2 X 2 cos pπ −p2 t
U(x, t) = sin px + e sin px
π p=1 p π p=1 p
∞ ∞
2 X 1 − 3(−1) p 4 X (−1) p −p2 t
U(x, t) = sin px + e sin px
π p=1 p π p=1 p
Note.1 If a, b are constants or functions of t then (21.5.1) is called Fredholm integral equation.
2. If a = 0, b = t then (21.5.1) is called Volterra’s equation.
The Laplace transform provides a useful technique for the solution of equation (21.5.2) in which
F(t), K(u, t) are known and Y(t) is to be determined.
Note.1 It is possible to convert every linear differential equation into integral equation. However
not every integral equation can be converted into a differential equation.
Example. Z t
Y(t) = cos t + ln(u + t)Y(u)du.
0
2. Volterra Integral Equation arises from linear differential equation where conditions are
specified at one point.
3. Fredholm Integral Equation arises from linear differential equation where the condition are
specified at two points.
Example. Z t
Y (t) = Y(t) + sin t +
00
cos(t − u)Y(u)du.
0
is called Abel’s integral eqaution where G(t) is known and n is constant such that 0 < n < 1.
21.6 Exercises
∂u
= k ∂∂xu2 , −∞ < x < ∞, t > 0 subject to the conditions u(x, 0) = f (x), ∂u
2
1. Solve ∂t ∂x
and u
tends to zero as x tends to ±∞.
∂u
= k ∂∂xu2 , for x ≥ 0, t ≥ 0 subject to the conditions u(x, 0) = u0 , t > 0 with initial
2
2. Solve ∂t
condition u(x, 0) = 0, x ≥ 0.
∂u ∂2 u ∂u ∂u
3. Solve ∂t
= ∂x2
, 0 < x < 6, t > 0, given ∂x
(0, t) = 0, ∂x
(6, t) = 0 and u(x, 0) = 2x.
∂u ∂2 u
4. Solve ∂t
= ∂x2
, 0 < x < π, t > 0, given that u(0, t) = 0, u(π, t) = 0 for t > 0 and
u(x, 0) = sin3 x.
∂u
= 2 ∂∂xu2 , 0 < x < 1, t > 0, given that u(0, t) = 0, u(1, t) = 0 for t > 0 and
2
5. Solve ∂t
Answers
R∞ √
1. u(x, t) = f (x − 2 kt φ)e−φ dφ.
2
√1
π −∞
R ∞ 1−e−ks2 t
2. u(x, t) = 2u0
π 0 s
sin pxd p.
2 2
(cos pπ−1) − p π t
pπx
3. u(x, t) = 6 + .
24 P∞
π2 p=1 p2
e 36 cos 6
B.Sc. Mathematics
Second Year - Fourth Semester
Core Paper-VII
TRANSFORM TECHNIQUES
Time: Three Hours Maximum: 75 Marks
PART-A ( 10 × 2 = 20 Marks)
Answer any TEN out of TWELVE questions
Each question carries TWO marks
t .
sin t
2. Find the Laplace transform of
2p+1
3. Find the inverse Laplace transform of p(p+1) .
h i
4. Find L −1 1
p2 +2p+5
.
PART - B (5 × 5 = 25Marks)
Answer any FIVE of SEVEN Questions.
Each question carries SIX marks.
h Rt i
13. Find L e−3t 0 1−cos at
t dt .
15. Find the Fourier series to represent the function f (x) = | sin x|, in −π < x < π.
x2
16. Find the Fourier transform of f (x) defined by f (x) = e− 2 .
∂Y ∂Y
17. Solve ∂x − ∂t = 1 − e−t , 0 < x < 1, t > 0 given that Y(x, 0) = x.
p2
h i
18. Find L −1 (p+2)3 ] .
19. Find the Fourier cosine transform of the function f (x) defined by
0<x<a
(
cos x,
f (x) =
0, x ≥ a.
PART - C (3 × 10 = 30 Marks)
Answer any THREE out of FIVE questions.
Each question carries TEN marks.
0<t<b
(
t,
F(t) =
2b − t, b < t < 2b,
22. Find the Fourier series of periodicity 3 for f (x) = 2x − x3 in 0 < x < 3.
d2 x
dt − 12x = e
− 4 dx given that x(0) = 1, and x0 (0) = −2 using
3t
24. Solve the differential equation dt2
Laplace transform.
zzzzz